eMedici- medicine and acute care Flashcards

1
Q

Adrenal insufficiency states: describe px based on biochem

A

addison’s disease-

Adrenal insufficiency is a failure of the adrenal glands to produce adequate amounts of adrenocortical hormones. It can be primary, secondary, or tertiary.

Primary adrenal insufficiency (Addison disease)

  • Primary adrenal insufficiency can be caused by abrupt destruction of the adrenal gland (acute adrenal insufficiency; e.g., due to massive adrenal hemorrhage) or by its gradual progressive destruction or atrophy (chronic adrenal insufficiency; e.g., due to autoimmune conditions, infection).
  • Autoimmune adrenalitis: Most common cause
  • Infectious adrenalitis
  • Adrenal hemorrhage [2][3]
  • Infitrates
  • Impaired activity of enzymes that are responsible for cortisol synthesis
  • Secondary adrenal insufficiency
    • Secondary adrenal insufficiency is caused by conditions that decrease ACTH production (impaired hypothalamic-pituitary-adrenal axis).
    • Sudden discontinuation of chronic glucocorticoid therapy or stress (e.g., infection, trauma, surgery) during prolonged glucocorticoid therapy
  • Tertiary adrenal insufficiency
    • Tertiary adrenal insufficiency is caused by conditions that decrease CRH production.

adrenal insufficiency ( loss of corticol hormones- androgens, glucocorticoids, mineralocorticoids

Hypocortisolism
A state of decreased production of glucocorticoids (e.g., due to adrenal insufficiency). Can cause weakness, fatigue, depression, decreased appetite, weight loss, nausea, vomiting, diarrhea, and abdominal pain.

Hypoaldosteronism
A state of either decreased aldosterone production (e.g., adrenal insufficiency, hyporeninemic hypoaldosteronism) or resistance to aldosterone (e.g., administration of K+ sparing diuretics). Since aldosterone normally increases the reabsorption of Na+ and water and decreases the reabsorption of K+ and the excretion of H+, patients manifest with hyperkalemia, hypovolemic hyponatremia, and mild hyperchloremic metabolic acidosis

Hypoandrogenism

How well did you know this?
1
Not at all
2
3
4
5
Perfectly
2
Q

Adrenal hyperplasia ddx

A

The idiopathic enlargement of adrenal gland tissue with associated hormone overproduction.

Most commonly occurs bilaterally and manifests with increased serum concentrations of adrenal hormones.

  • THE THREE C’s- conns, cushings,chromo
    • Can result in hyperaldosteronism (Conn syndrome) and
    • hypercortisolism (Cushing syndrome).
    • pheochromo

How would Conns present? Hyperaldosteronism

  • Since aldosterone normally increases the reabsorption of Na+ and water and decreases the reabsorption of K+ and the excretion of H+, patients manifest with
    • hypokalemia,
    • hypervolemic hypernatremia, and
    • mild hypochloremic metabolic alklosis

How would cushings present- HyperCortisol state

  • cushings syndrome ( primary ACTH independent adernal hypreplasia)
  • cushing disease (secondary ACTH dependent hyperplasia-pituitary microadenoma)

Cortisol causes-

  • Mobilization of energy reserves, encourages gluconeogenesis: hypoglyceamia
  • tonic dis-inhibition of ADH: free water retention-

“CUSHINGOID” is the acronym for side effects of corticosteroids: Cataracts, Ulcers, Striae/Skin thinning, Hypertension/Hirsutism/Hyperglycemia, Infections, Necrosis (of the femoral head), Glucose elevation, Osteoporosis/Obesity, Immunosuppression, Depression/Diabetes.

Patients with secondary hypercortisolism due to ectopic ACTH production may present with rapid onset of hypertension and hypokalemia without other typical features of Cushing syndrome.

Consider a diagnosis of hypercortisolism in patients who present with proximal muscle weakness, central obesity, thinning skin, weight gain, sleep disturbance, and/or depression.

How well did you know this?
1
Not at all
2
3
4
5
Perfectly
3
Q

SGL2-I - dapaglitlozin and warfarin. Which should be stopped preoperatively?

A

SGL2-I should be stopped. I’m stressed states, ie surgery/illness. Can cause euglyceamic ketoacidisis. Warfarin can be stopped 5 days prior to surgery. Need for Bridging therapy is dependent on VTE risk.

How well did you know this?
1
Not at all
2
3
4
5
Perfectly
4
Q

Adrenal hormones

A
How well did you know this?
1
Not at all
2
3
4
5
Perfectly
6
Q

disorientated and mumbling. recently been started on sertraline for depression. Apart from her disorientation in time and space, the patient does not have any abnormal signs on physical examination. Her blood pressure is 140/80 mmHg and pulse rate 90/min. Her laboratory results are shown:

Serum:

Sodium 112 mmol/L (135 - 145 mmol/L)
Potassium 2.7 mmol/L (3.5 - 5.5 mmol/L)
Chloride 90 mmol/L (95 - 110 mmol/L)
Urine:

Osmolality 900 mOsm/kg (500-850 mOsm/kg)
Sodium 31 mmol/L (<20 mmol/L)
Which one of the following is the most likely diagnosis?

A

SSRIs can cause syndrome of inappropriate anti-diuretic hormone secretion.

syndrome of inappropriate anti-diuretic hormone secondary (SIADH) to recent commencement of sertraline; given the low serum sodium, confusion, altered mental state, high urine osmolality and high urine sodium.

In SIADH, anti-diuretic hormone is released in excess. This leads to increased AQP-2 in the apical membrane of the distal convoluted tubule and collecting duct of the nephron in the kidney, and increased resorption of water. This leads to high urine sodium and osmolality, and dilution of serum electrolytes (particularly sodium) with the reabsorbed water. This leads to hyponatremia, which is the cause of this patient’s confusion and altered mental state.

How well did you know this?
1
Not at all
2
3
4
5
Perfectly
7
Q

disorientated and mumbling. recently been started on sertraline for depression. Apart from her disorientation in time and space, the patient does not have any abnormal signs on physical examination. Her blood pressure is 140/80 mmHg and pulse rate 90/min. Her laboratory results are shown:

Serum:

Sodium 112 mmol/L (135 - 145 mmol/L)
Potassium 2.7 mmol/L (3.5 - 5.5 mmol/L)
Chloride 90 mmol/L (95 - 110 mmol/L)
Urine:

Osmolality 900 mOsm/kg (500-850 mOsm/kg)
Sodium 31 mmol/L (<20 mmol/L)
Which one of the following is the most likely diagnosis?

A

Although Addison’s disease can cause increased urine sodium and decreased serum sodium; this presents as an “Addisonian crisis” which results in severe dehydration, vomiting, and muscle pain. This patient has a normal blood pressure, and no history of Addison’s disease, thus, it is an unlikely cause of her presentation.

Signs and symptoms [19]

Diagnosis [12][19][32]

Consider adrenal crisis in patients with severe hypotension refractory to fluid resuscitation and/or vasopressors.

Adrenal crisis can be life-threatening, so treatment with high doses of hydrocortisone should be started immediately, without waiting for diagnostic confirmation of hypocortisolism!

Management [1][19]

The 5 S’s of adrenal crisis treatment are Salt (0.9% saline), Sugar (50% dextrose), Steroids (100 mg hydrocortisone IV once, then 200 mg over 24 hours), Support (normal saline to correct hypotension and electrolyte abnormalities), and Search (for the underlying disorder).

How well did you know this?
1
Not at all
2
3
4
5
Perfectly
8
Q

Best option for smoking cessation?

A

behavioral counseling, nicotine replacement therapy NRT : vareniciline most effective —: nicotine receptor Agonist Varenicline is the most effective, single-form pharmacotherapy when used in combination with behavioural support, and has been shown to more than double the chances of long term cessation. It is a nicotinic receptor partial agonist that relieves symptoms of craving and withdrawal. It is contraindicated in patients who are breastfeeding and pregnant. Varenicline has been associated with mood changes, depression, behavioural and sleep disturbance, abnormal dreams and suicidal ideation, however, no causal link has been established and it is safe to use in patients with stable mental illness, although monitoring should occur. For people with a creatinine clearance of less than 30 mL/min, start with varenicline 0.5 mg daily for 3 days and then a maximum of 1 mg daily. Avoid varenicline in patients with end-stage kidney disease (eTG). Buproprion was originally developed for use as an antidepressant, it significantly increases cessation rates, however is less effective than varenicline. It carries a risk of lowering the seizure threshold and thus is contraindicated in patients with a history of seizures, eating disorders and those taking monoamine oxidase inhibitors; while caution should be taken with other medications that may lower the seizure threshold.

How well did you know this?
1
Not at all
2
3
4
5
Perfectly
9
Q

Pre-op drugs. Amlidipine Atorvostatin Metformin/sitagliptin Which need to be stopped before and when

A

Amcodipine: can be continued if normotensive- not hypotensive.
Atorvostatin can be continued

metformin/sitagliptin- stopped on day of surgery when fasting starts

How well did you know this?
1
Not at all
2
3
4
5
Perfectly
10
Q

what is the detail of ASA scores- what is the utility in practise?

A

general idea of the functional status of a comorbid patient and thus the risk posed to them undergoing surgery. This allows a risk benefit discussion to be had regarding the surgery, especially when it is an elective procedure and may not be absolutely necessary. One way to help conceptualise the risk is based on a patient’s ASA score.

ASA 1: healthy patient, non-smoking and minimal alcohol

ASA 2: mild disease without functional limitation (smoker, social drinker, pregnant, BMI 30-40, well controlled DM/hypertension, mild lung disease)

ASA 3: systemic disease with some functional limitation (e.g. poorly controlled DM or hypertension, COPD, morbid obesity (>40), heart failure, end stage kidney disease on dialysis, MI >3 months ago)

ASA 4: severe systemic disease that is a threat to life (e.g. MI <3months ago, CVA, severe valve disease, sepsis, ARDS, end stage kidney disease without dialysis)

ASA 5: patient will not survive without surgery (ruptured aortic aneurysm, massive trauma, intracranial bleed, ischaemic bowel)

ASA 6: brain dead patient for organ donation

How well did you know this?
1
Not at all
2
3
4
5
Perfectly
11
Q

GCS

A

ABC- First secure the airway if GCS is less than or equal to 8.

How well did you know this?
1
Not at all
2
3
4
5
Perfectly
12
Q

dexamethasone for head injury?

A

While effective in decreasing intracranial pressure due to malignancy, dexamethasone is contraindicated in traumatic brain injury due to an association between increased all-cause mortality and corticosteroid administration post-traumatic brain injury.

How well did you know this?
1
Not at all
2
3
4
5
Perfectly
13
Q

large city hospital after a building site accident in which he fell head-first onto a concrete floor. His pulse is 50/min and blood pressure 160/80mmHg. He is breathing spontaneously but irregularly. In response to pain stimulus, he opens his eyes, mumbles incoherently, and flexes the stimulated limb. He has a haematoma over the right temple. His right pupil is larger than the left, and the direct and consensual light reflexes are normal in the left eye and absent in the right eye.

A

extradural haemorrhage that will require emergency neurosurgical intervention, especially since he exhibits all features of Cushing’s triad - bradycardia, hypertension, and irregular breathing.

However, the most pressing concern is his conscious state: his GCS is 8 (E2V2M4), and his airway is now at risk of compromise. As such, he requires immediate intubation to secure his airway, and to ensure adequate ventilation and oxygenation in order to prevent secondary brain injury due to hypoxia.

A burr hole is not the preferred operation for this patient. A craniectomy or craniotomy is more effective due to the size of the hole and higher efficacy in evacuating the bleed. Although urgent surgical intervention will likely be required in this case, it is not the immediate priority here.

How well did you know this?
1
Not at all
2
3
4
5
Perfectly
14
Q

ACD vs IDA

A

NB- key features
ACD: ferittin is high/N, transferin is low N

IDA: ferritin is LOW, transferrin is high/N

How well did you know this?
1
Not at all
2
3
4
5
Perfectly
15
Q

In IDA- key features?

A

Hypochromic (MCH) microcytic anaemia

Hepciden- low (in ACD high)

Low ferritn, high transferrin.

How well did you know this?
1
Not at all
2
3
4
5
Perfectly
16
Q

Haemoglobin 102g/L (115-165)
Mean Cell Volume 75fL (80-100)
Mean Cell Haemoglobin Concentration 24pg (27-32)

DDX for IDA?

A

Thalassaemia minor- hb normally dx at younger age
Carcinoma of the caecum- occult blood loss, few syptoms. expect in the elderly
Abnormal uterine bleeding- young women
Dietary deficiency of iron- look for well balanced/vegitarian/vegan
Hookworm infestation- more likely malabsorption. look for histamine/IgE

Carcinoma of the uterus

EG answers

Iron deficiency anaemia which is caused by either

  • inadequate intake of iron,
  • mal-absorption of iron, or
  • loss or iron through blood loss or
  • excessive use of iron.

Iron loss through abnormal, heavy, uterine bleeding and/or inadequate iron intake through diet are the two most common causes of iron deficiency in women of childbearing age group.

A detailed diet and gynaecological history should be obtained to evaluate iron deficiency in this age group.

She is less likely to have inadequate iron intake as she eats a balanced diet with meat and vegetables.

Carcinoma of the caecum (leading to gastrointestinal blood loss) is uncommon in young women but needs to be considered as a cause of iron deficiency if she complains of gastrointestinal symptoms, has a family history of early gastrointestinal cancer or there is no other good cause for iron deficiency identified.

Hookworm infestation, causing malabsorption, is possible but less likely.

Thalassemia minor is possible in this ethnic background but would have been diagnosed in prior blood tests.

eg 2 old women non specific fatigue

most likely, based on her age and lack of signs, is caecal cancer. With no visible bleeding, asymptomatic ‘occult’ chronic blood loss is common in gastrointestinal tract cancers especially right sided large bowel cancer.

Gastric cancer can and does lead to iron deficiency anaemia, but is less common than bowel cancer and is associated with more symptoms even if they are vague such as early satiety, unintentional weight loss, nausea or ‘dyspepsia’.

Uterine cancer could also cause iron deficiency anaemia, but again is more commonly associated with some signs such as visible loss of blood - spotting.

Thalassaemia is an important differential, especially considering her Greek ethnicity, but is unlikely if she has no history of anaemia previously and there is evidence of iron deficiency on her bloods.

Dietary iron deficiency could be contributing, but is unlikely to result in iron deficiency anaemia in a woman who is no longer menstruating and hence has no sources of regular blood loss.

This woman should urgently undergo endoscopy and colonoscopy, as well as receive treatment for her iron deficiency anaemia.

Learning Points

An older individual presenting with iron deficiency anaemia needs to be investigated for bowel cancer with endoscopy and colonoscopy

Iron deficiency anaemia causes a hypochromic microcytic anaemia

How well did you know this?
1
Not at all
2
3
4
5
Perfectly
17
Q

three month history of difficulty in swallowing. He has a history of gastro-oesophageal reflux disease (GORD) treated with 20mg pantoprazole daily.

He denies weight loss, regurgitation, odynophagia, heartburn or dyspnoea. He has a 40 pack-year smoking history with a smoker’s cough and consumes about three standard drinks of alcohol a day. The physical examination is unremarkable.

An endoscopy is performed, which shows extensive and circumferential ulcerative oesophagitis immediately above a small (3 cm) hiatus hernia. Biopsies confirm severe inflammatory changes only.

A

MX of hiatis hernis:

Sliding hiatal hernia

A condition in which the gastroesophageal junction and part of the stomach temporarily protrude through the esophageal hiatus in the diaphragm into the chest during swallowing. The gastric fundus remains below the diaphragm. Most common type of hiatal hernia.

Management of patients with sliding hiatal hernia
Conservative management

  • Lifestyle modifications
  • Proton pump inhibitors (PPIs) if symptoms of GERD occur

Surgery: laparoscopic/open fundoplication and hiatoplasty [2]

  • Indications
  • Persistence of symptoms despite conservative management
  • Refusal or inability to take long-term PPIs
  • Severe symptoms/complications of gastroesophageal reflux disease: bleeding, strictures, ulcerations

next point- any despesia, consider

Red flag features of dyspepsia

Patients ≥ 60 years of age [1]

The following is applicable to patients ≥ 60 years of age with or without red flag features for dyspepsia.

  • Refer for EGD to exclude neoplasia.
  • Test for the presence of H. pylori during the endoscopy using biopsies and/or a rapid urease test.
  • If a clear etiology is identified using EGD (e.g., visualization of peptic ulcer, gastritis), manage accordingly.
  • If the etiology remains unclear after EGD, consider further testing based on clinical suspicion.

How well did you know this?
1
Not at all
2
3
4
5
Perfectly
18
Q

EGD- short and long segmenet. length? Mx of each

EGJ biopsie- low grade vs high grade dysplasia. Mx?

A

GERD MANAGEMENT

Biopsie:

  • Short-segment (< 3 cm of columnar epithelium between Z line and GEJ)
  • Long-segment (> 3 cm of columnar epithelium between Z line and GEJ): higher cancer risk
  • Complications: esophageal adenocarcinoma (see “Esophageal cancer”)
  • Management and surveillance
    • PPI therapy [27]
      • Consider if asymptomatic.
      • Continue maintenance therapy long-term if symptomatic.
    • Endoscopy with four-quadrant biopsies at every 2 cm of the suspicious area (salmon-colored mucosa)
      • If no dysplasia: Repeat endoscopy every 3–5 years.
      • If indefinite for dysplasia: Repeat endoscopy with biopsies after 3–6 months of optimized PPI therapy.
      • If low-grade dysplasia:
        • Endoscopic therapy of mucosal irregularities-Usually including endoscopic mucosal resection and radiofrequency ablation
        • Alternatively: surveillance every 6-12 months with biopsies every 1 cm
      • If high-grade dysplasia: endoscopic treatment of mucosal irregularities, e.g., radiofrequency ablation
    • Consider antireflux surgery or resection of the segment based on a specialist’s evaluation
How well did you know this?
1
Not at all
2
3
4
5
Perfectly
19
Q

three month history of difficulty in swallowing.

He has a 20-year history of gastro-oesophageal reflux disease treated with 40mg pantoprazole daily. He denies weight loss, regurgitation, odynophagia, heartburn or dyspnoea. He has a 40 pack-year smoking history, with a smoker’s cough and consumes about three standard drinks of alcohol a day.

The physical examination is unremarkable apart a husky voice and a non-tender 2cm mass in the left supraclavicular fossa.

Which one of the following is the most appropriate diagnostic investigation?

vs

72 y/o three-week history of a painless swelling in her neck. Upon further questioning, she describes feeling generally unwell over the last few months.

She has lost her appetite and unintentionally lost 10kg of weight. She reports a long-standing “smoker’s” cough with no changes in sputum. She does not report a medical history although admits to a forty-pack-year smoking history.

On examination, there is a non-tender 2 cm smooth swelling deep to the skin in the left supraclavicular fossa. There are no abnormalities detected on examination of her chest and abdomen.

Which one of the following is the most appropriate next step in management?

A

Endoscopy is a first-line investigation for the diagnosis and staging of Oesophageal Carcinoma

Long-term, chronic GORD can cause metaplastic changes in the lower oesophageal epithelium (known as Barrett’s Oesphagus), predisposing patients to oesophageal adenocarcinoma

carcinoma of the oesophagus, given his presenting complaint, history of GORD and smoking, and enlargement of Virchow’s node on physical examination.

An endoscopy is the most appropriate initial investigation, both in terms of a visual inspection of the upper digestive tract and the ability to obtain tissue for histological examination.

Although there is an increasing tendency for the initial examination in these patients to be a CT scan, this is probably a matter of expediency rather than logical clinical practice. In other words, although CT scan is necessary for staging of the cancer, and in the real world it is often easier to get a CT scan performed than wait for an endoscopy, endoscopy is the most appropriate diagnostic investigation.

Barium swallow may indicate the presence of an obstruction, but will not specifically diagnose oesophageal carcinoma, and will not allow for pathological diagnosis.

-→

eg 2 The correct answer is “chest x-ray”. The most likely diagnosis to fit this clinical scenario is a carcinoma of the lung. This is due to the combination of supraclavicular lump, significant systemic symptoms, cough, and smoking history. The swelling in her neck could well be a metastatic deposit. The easiest and most accessible investigation to confirm these clinical suspicions is to perform a chest X-ray. Depending on what this shows a CT of the chest and ultrasound-guided fine needle aspiration of the neck lump can be considered.

CT neck, chest, and abdomen is a reasonable investigation to consider following a chest x-ray.

Indirect laryngoscopy is not an appropriate initial investigation for this patient. Indirect laryngoscopies are useful for the diagnosis of head and neck cancers such as those of the larynx. This patient’s presentation is most likely explained by a carcinoma of the lung.

An ultrasound-guided biopsy may be considered following a chest x-ray.

Gastroscopy may be useful for diagnosing cancers of the stomach which may metastasise to the left supraclavicular lymph node. (Also known as Virchow’s node). This is a possible differential, however, carcinoma of the lung is more likely and a chest x-ray is a less invasive first investigation.

Learning Points

  • Carcinomas of the lung may metastasise to the supraclavicular lymph nodes.
  • Chest x-rays are an accessible and appropriate first-line investigation for suspected carcinoma of the lung.
How well did you know this?
1
Not at all
2
3
4
5
Perfectly
20
Q

Andexanet during surgery?

A

Andexanet is a reversal agent for factor Xa inhibitors (eg, rivaroxaban, apixaban), currently not available in Australia, and is intended for the reversal of acutely severe bleeding caused due to factor Xa inhibitors. This patient is currently not on any factor Xa inhibitors and does not require emergency anticoagulation reversal.

How well did you know this?
1
Not at all
2
3
4
5
Perfectly
20
Q

Andexanet during surgery?

A

Andexanet is a reversal agent for factor Xa inhibitors (eg, rivaroxaban, apixaban), currently not available in Australia, and is intended for the reversal of acutely severe bleeding caused due to factor Xa inhibitors. This patient is currently not on any factor Xa inhibitors and does not require emergency anticoagulation reversal.

How well did you know this?
1
Not at all
2
3
4
5
Perfectly
21
Q

when is Prothrombin complex concentrate (PCC) used?

A

Prothrombin complex concentrate (PCC) and fresh frozen plasma (FFP) before surgery are indicated for rapid warfarin reversal in patients who require emergency surgery or in patients with severe ongoing bleeding. PCC contains vitamin K-dependent clotting factors and helps reverse the anticoagulant effect caused by warfarin due to the inhibition of vitamin K-dependent factors (eg, factor II, VII, IX, and X). FFP is basically plasma that is acquired by removing the cellular components of the blood. Between the two, PCC is preferred for warfarin reversal since FFP is associated with an increased risk of volume overload.

How well did you know this?
1
Not at all
2
3
4
5
Perfectly
22
Q

Rx with Ticagrelor. Can patelet transfusion be used?

A

Platelets transfusion for patients taking ticagrelor is not useful since ticagrelor is a reversible ADP-receptor inhibitor and will likely bind to the receptors of the transfused platelets and render them inefficient.

How well did you know this?
1
Not at all
2
3
4
5
Perfectly
23
Q

PMR and GCA

This patient has a severe temporal headache and also has had stiffness in the shoulders and systemic symptoms (eg, fatigue) for the last few weeks.

GCA should be highly suspected in this patient.

A

Rx; Steroids immediately!

Giant cell arteritis (GCA) is systemic vasculitis that presents in patients aged >50 years and is characterized by systemic symptoms (eg, fever, weight loss, fatigue), temporal headache, jaw claudication, and visual pathologies (eg, amaurosis fugax, anterior ischaemic optic neuropathy). GCA is also associated with polymyalgia rheumatica that presents with pain and stiffness in the shoulders and the hips.

Generally, inflammatory markers (eg, ESR, CRP) are elevated in such patients. If GCA is highly suspected, patients should immediately be started on high-dose oral prednisolone (40-60 mg daily) without waiting for diagnostic confirmation because GCA is associated with a high risk of permanent vision loss and should be managed as soon as possible. Determining the route and dosage of glucocorticoids is shown in the table below.

Transfer to regional centre is not needed since GCA can be managed with glucocorticoids alone which are easily available in small rural hospitals. Although temporal artery biopsy is the confirmatory test for GCA, treatment should not be delayed for diagnostic confirmation because of the high risk of permanent vision loss.

How well did you know this?
1
Not at all
2
3
4
5
Perfectly
24
Q

history of numbness between his thighs and an inability to pass urine.

His recent medical history includes carcinoma of the prostate which is being treated with goserelin.

On examination there is an area of numbness in the inner thighs, extending into the perineum. Percussion of the abdomen shows an area of dullness extending three finger breadths above the pubis.

Which of the following is the most appropriate diagnostic investigation

A

Sudden onset of urinary retention and associated neurological symptoms must always raise the possibility of spinal cord compression.

diagnosis in this patient is cauda equina syndrome and this must be confirmed or excluded promptly. Cauda equina syndrome is a serious complication of severe compression of the descending lumbar and sacral nerve roots.
It generally presents with:

  • bilateral radicular pain,
  • asymmetric motor weakness,
  • hyporeflexia, and bladder dysfunction.

This patient has sensory loss extending to the perineum and neurogenic bladder (evidence by dull percussion note above the pubis).

If the diagnosis is confirmed, the patient will require emergency surgical intervention and decompression. An MRI is the diagnostic modality of choice for the diagnosis of cauda equina syndrome. An MRI can be used to define the possible changes of spinal epidural haematoma, disc protrusion, degenerative canal stenosis, and/or epidural abscess.

Plain X-ray of the spine may be indicated when there is a suspicion of degenerative bone disease, fracture, or other bone pathologies. It is not a good diagnostic investigation to look for soft tissue pathologies or nerve compression pathologies.

Non-contrast CT scan is not good diagnostic imaging to visualise the nerve roots. It may be helpful in the diagnosis of bone disease. However, this patient has clinical features suggestive of radicular disease and non-contrast CT would be less yielding than an MRI.

CT myelogram is a more invasive test than MRI. It can identify nerve roots but MRI is preferred since an MRI can also rule out other possible differential diagnoses (eg, epidural abscess). A CT myelogram may be indicated if the patient has some contraindication to MRI (eg, electrical implanted devices).

A PET scan shows the metabolic activity at a cellular level. It may be helpful for assessing metastases due to cancer. However, a PET scan is not useful for diagnosing radicular diseases (eg, cauda equina syndrome).

How well did you know this?
1
Not at all
2
3
4
5
Perfectly
25
Q

cauda equina vs conus medullaris

A
How well did you know this?
1
Not at all
2
3
4
5
Perfectly
26
Q

Straight leg test

A

Straight leg raise tests

Straight leg raise tests are used to differentiate between neurological and musculoskeletal causes of lower limb pain.

Positive test (SLT- nn impingement– correclate with reflex weakness to localize)

Lasègue test (top): With the patient supine and the knee in extension, the examiner lifts the patient’s leg. The test is considered positive when lifting to an angle 30º–45º elicits radicular pain, i.e., shooting pain that radiates from the back along the posterior surface of the elevated leg (some sources quote other ranges, e.g., 30º–70º). A positive test suggests lumbosacral nerve root impingement.

Bragard test/sign (middle): Following a positive Lasègue test, the leg is slightly lowered (just enough for the pain to disappear) and the ankle dorsiflexed. If dorsiflexion produces pain, the Bragard test is considered positive. A positive Bragard strengthens the diagnosis of lumbosacral nerve root impingement.

Pseudo-Lasègue sign (bottom): A straight leg raise produces an aching pain along the back of the leg (rather than typical radicular pain). This indicates that the pain likely originates from the hamstring muscles (usually occurs with the leg flexed to > 60º). The Lasègue test is then interpreted as negative. A negative Bragard test helps differentiate the Pseudo-Lasègue sign from a true positive Lasègue test and further strengthens the suspicion of a muscular etiology.

Positive Lasègue test + positive Bragard test: neurological etiology likely
Pseudo-Lasègue sign + negative Bragard test: muscular etiology likely

How well did you know this?
1
Not at all
2
3
4
5
Perfectly
27
Q

reflex levels for each radiculopathy level

(L2 - S4)

A
How well did you know this?
1
Not at all
2
3
4
5
Perfectly
28
Q

X ray shoulder- dislocations

A

How well did you know this?
1
Not at all
2
3
4
5
Perfectly
29
Q

unconscious collapse at home. She has been unwell with a fever, rigors, and confusion over the last few days. On presentation today, her blood pressure has varied from 90/50 to 150/70mmHg, pulse rate 110/min, respiratory rate 22/min, and temperature 38.9 centigrade. She has rigidity in all four limbs, but normal reflexes and downgoing plantars. She is not rousable to voice. Her past medical history includes schizophrenia and bipolar disorder. She has taken risperidone for many years and was started on lithium nearly two weeks ago.

A

presenting with the distinctive clinical picture of neuroleptic malignant syndrome (NMS),

  • characterised by muscle rigidity (often “lead-pipe” rigidity),
  • normal or depressed reflexes,
  • mental status change,
  • and protracted onset (typically days as opposed to hours).
  • Autonomic instability (fever, tachycardia, diaphoresis, labile BP) is also seen, however, is not specific to NMS.
    • NMS will occur in the context of recent dopamine antagonist use e.g. antipsychotics or particular antiemetics.
  • Investigations will also show an elevated CK, which can be used to distinguish NMS from similar clinical syndromes.

Her presentation could be solely related to her risperidone, as NMS can occur at any time during treatment (though typically the first week), however in the context of recently commencing lithium - a drug that may potentiate NMS with concurrent antipsychotic use - the new medication is the likely trigger. This is also consistent with lithium dosing as lithium typically takes 4 - 7 days to reach steady state concentrations, and her symptoms started during the second week of lithium therapy.

The most similar clinical syndromes to NMS are serotonin syndrome and lithium toxicity.

Serotonin syndrome can be distinguished by current use of serotonin agonist(s), more rapid onset within 24 hours, clonus, and hyperreflexia. It can also present with autonomic dysfunction, thus the fever, tachycardia, diaphoresis and labile blood pressure that can be seen in serotonin syndrome.

Lithium toxicity can be distinguished by supratherapeutic lithium intake or elevated serum levels, prominent gastrointestinal symptoms, and tremor.

Malignant hyperthermia is a condition that can result from the use of various anaesthetic agents and is associated with a high fever, tachycardia and muscle rigidity.

Whilst drug hypersensitivity can produce a similar picture of high fever and tachycardia, there is usually some form of cutaneous reaction (rash). Whilst all these conditions must be considered in the given clinical scenario, NMS is the most likely diagnosis and would explain all the given features.

It is important to note that in clinical practice multiple medications are often being taken by patients at varying dosages. As such, it is possible that patients may be experiencing manifestations from multiple clinical syndromes (eg. serotonin syndrome and NMS) concurrently.

Learning Points
Neuroleptic malignant syndrome (NMS) is caused by antipsychotics and characterised by fever, “lead-pipe” muscle rigidity, altered conscious state, autonomic dysfunction and elevated CK

How well did you know this?
1
Not at all
2
3
4
5
Perfectly
30
Q

drug of choice in the setting of head injuries and control of any subsequent seizures.

A

Traumatic brain injuries can be associated with the risk of subsequent seizures. Post-traumatic seizures can occur both in the early period (up to a week post-injury) and late period (after a week post-injury). The patient has risk factors for developing early PTS: Glasgow Coma Scale = 10, depressed skull fracture and penetrating head injury.

There has been a shift towards using levetiracetam for prophylactic pharmacological treatment due to its suitability for extended used and decreased side effect profile in comparison with phenytoin.

How well did you know this?
1
Not at all
2
3
4
5
Perfectly
31
Q

choice of beta blocker in an elderly patient with HFrEF

carvidalol, bisoprolol, misoprolol, Nebivolol, Atenolol, sotolo, Labetalol, Propranolol

A

four beta-blockers with evidence for use in congestive cardiac failure in terms of providing a mortality benefit are:

  1. metoprolol.
  2. nebivolol,
  3. bisoprolol,
  4. carvedilol,

These agents can reduce the mortality rate by about 30% over one year. Nebivolol has been studied for use in the elderly population with heart failure (SENIORS study).

The other three beta-blockers (ie, carvedilol, bisoprolol, metoprolol) provide significant mortality benefit but have not been studied extensively for use in the elderly population.

(AS) Atenolol and sotalol are associated with increased mortality in patients with congestive heart failure.

Labetalol is labeled for use in the acute management of severe hypertension or in hypertension during pregnancy. It is not indicated for use in congestive heart failure.

Although several studies have shown the mortality benefit of propranolol, it is not extensively studied enough to be considered a recommended choice of beta-blocker for heart failure.

SUMMARY

Use in HFrEF :Bisoprolol, Carvidalol, Misoprolol, Nebivolol. only Nebivolol in elderly

Atenolol, Sotolo (AS!)- increase mortality in HF

Labetalol, severe hyptns/ hypt in pregnancy (lullaby)

Propanalol- mortality- yes. HF?? not sure

How well did you know this?
1
Not at all
2
3
4
5
Perfectly
32
Q

X ray shoulder- dislocations

A
How well did you know this?
1
Not at all
2
3
4
5
Perfectly
33
Q

X ray shoulder- dislocations

A
How well did you know this?
1
Not at all
2
3
4
5
Perfectly
34
Q

A 50-year-old , reports five-month history of progressively worsening ankle oedema. exertional dyspnoea, accompanied by palpitations and a feeling of pressure in his chest when exerting himself.

drinks at least one bottle of wine daily. past medical history - type 2 diabetes and hypertension. currently taking metformin and perindopril.

he is haemodynamically stable.

On examination, the abdomen is not distended, soft and non-tender with no palpable masses.

His heart sounds are dual, with no murmurs and his pulse is regular. There is marked non-tender, pitting oedema of bilateral ankles. On auscultation of the chest, bibasilar crepitations.

Some investigations are performed.

Which of the following investigations would be most appropriate in determining the underlying cause of his peripheral oedema?

  • Venous duplex ultrasound
  • Echocardiography
  • Liver ultrasound
  • ECG
  • 24-hour urine protein
A

Learning Points
Common causes of bilateral ankle oedema include heart failure, cirrhosis, renal failure and chronic venous insufficiency

This patient’s peripheral oedema has multiple possible aetiologies, including heart failure, renal failure and cirrhosis.

However congestive heart failure is most likely based on the combination of respiratory fluid overload as well as peripheral oedema. Therefore an echocardiogram would be most appropriate in determining the underlying aetiology and severity of the disease.

An ECG is a mainstay of assessment of any patient with shortness of breath, palpitations or chest pain, however, while arrhythmia may be a cause of cardiac failure, it is unlikely in this case to determine the underlying aetiology given that this patient’s pulse was regular. Furthermore, in this case the exertional palpitations are more likely to be secondary to heart failure.

Severe hypoalbuminaemia may contribute to oedema in context of liver cirrhosis or nephrotic syndrome, however given these disorders are less likely to induce pulmonary oedema than heart failure, a liver ultrasound or 24-urine protein are not likely to determine the cause in this case.

Peripheral venous insufficiency is the most common cause of chronic peripheral oedema, however, it does not explain the respiratory or cardiac symptoms this patient is experiencing.
A venous duplex ultrasound may be considered if there was a suspicion of venous insufficiency of DVT, although it is not appropriate in this case.

Causes of acute bilateral peripheral oedema include medication side effects (calcium channel blockers), or bilateral DVT.

While other chronic causes of bilateral oedema include lymphoedema or myxoedema - although neither are true oedematous states.

How well did you know this?
1
Not at all
2
3
4
5
Perfectly
35
Q
  • Erythema induratum
  • Erythema annulare centrifugum
  • Erythema marginatum
  • Erythema ab igne
  • Livedo reticularis

Describe each

A

Erythema ab igne is a cutaneous reaction to repeated exposure of direct heat or infrared radiation to a person’s skin.

Heat sources that can cause this condition if there is repeated exposure include:

  • Hot water bottles, heating blankets or heat pads to treat chronic pain
  • Heated car seats, space heaters or fireplaces.
  • Occupational hazards of silversmiths and jewellers (face exposed), bakers and chefs (arms, face exposed)
  • Laptop computers rested on the anterior thighs
  • Erythema ab igne has been reported in people with chronic pancreatitis, and is associated with hypothyroidism and
  • lymphoedema.

Primary treatment of erythema ag igne is the removal of the causative heat source. Investigation of underlying causes of pain and excluding hypothyroidism should be considered.

Mild cases will settle spontaneously. In severe cases, abnormal pigmentation, atrophy or scarring may persist. Rarely, these lesions may develop into cutaneous malignancy. If there is any clinical suspicion of epidermal atypia, regular skin examination ± biopsies are recommended.

An important clinical differential for erythema ab igne is livedo reticularis. In livedo reticularis however, the skin changes are symmetrical (not focal to the point of heat source) and telangiectatic (rather than pigmented).

Erythema induratum (Bazin disease) is a panniculitis that typically presents on the calves, associated with a Mycobacterium tuberculosis infection.

Erythema annulare centrifugum is thought to be a chronic hypersensitivity reaction characterised by annular erythematous lesions, with a pathognomonic delicate scale behind the advancing edge (‘trailing scale’).

Erythema marginatum is a non-pruritic macular annular rash that typically occurs on the limbs which is associated with acute rheumatic fever.

Learning Points
Erythema ab igne is a cutaneous reaction to chronic direct heat exposure

How well did you know this?
1
Not at all
2
3
4
5
Perfectly
36
Q

Crohn’s disease
Pyoderma gangrenosum
Squamous cell carcinoma
Diabetic ulcers
Pseudomonas infection

systemically well.

On examination there are two large full-thickness ulcers across the right scalp. The ulcers have purple margins with an undermined edge. The surrounding skin appears normal. On the leg at the site of the previous ulcer, a cribriform scar was noted.

Swab mcs was unremarkable. The edge of one of the ulcers was biopsied and revealed a neutrophilic infiltrate with no evidence of infection or malignancy.

A

Pyoderma gangrenosum (PG) is a painful cutaneous ulcerative condition that is included in the group of neutrophilic dermatoses. It may affect any body site but does have a predilection for the lower limbs. It often develops at a site of minor tauma.

Characteristically the full-thickness ulcer has an undermined edge which is purple in colour. PG is not an infection, nor does it involve gangrene. Aetiology is poorly understood but is believed to be due to immune system dysregulation and genetic variations. Other neutrophilic dermatoses include Sweets syndrome, neutrophilic dermatosis of the dorsal hands, Behcet disease and bowel-associated-dermatitis-arthritis syndrome. PG is a diagnosis of exclusion - other causes of ulceration including infection, malignancy, vasculitis, diabetes and trauma need to be excluded.

Crohn’s disease can manifest as extra-gastrointestinal ulceration, however sites affected are commonly the trunk, arms and legs. Histology of cutaneous Crohn’s would reveala granulomatous infiltrate. Inflammatory bowel disease is associated with PG.

Cutaneous malignancy like squamous cell carcinoma is an important differential for chronic ulceration, particularly on the scalp.

Diabetes is a common cause of ulcers, typically occurring on the lower legs. Infection may be a cause of ulcers or occur within chronic ulceration and then impair healing. In this case, the clinical scenario points to the diagnosis of PG, with the histopathology excluding malignancy and wound swabs negative. If it was felt these conditions were clinically more likely, consider sampling error of investigations and repeat with representative samples.

Learning Points
Pyoderma gangrenosum is a painful cutaneous ulcerative condition that is a diagnosis of exclusion.

How well did you know this?
1
Not at all
2
3
4
5
Perfectly
37
Q

GLP 1 agonists and SGLT-2

GLP-1 agonists ( “Aglutide”) DOC with which co-existing Cardiovascular concern ()?

SGLT-2 DOC in (“flozin”)?

A
  • GLP-1 agonists and SGLT2 inhibitors have the best evidence base for cardiovascular benefits in patients with atherosclerotic ) cardiovascular disease
    • (GLP-1, esp vascular problems)
    • (SGLT-2, esp heart failure).
  • Considering renal function is important when selecting oral diabetic medications
    • SGLT2 inhibitors require decent renal function to adequately cause a glucose lowering effect, and so in this patient with ESRF, this would not be an appropriate choice.
  • SGLT-2 “iglaflozin”
  • Dapagliflozin
  • Empagliflozin
  • Canagliflozin

S/E of keto acidosis- must be stopped prior to surgery

From stages 3–4 of chronic kidney disease, the side effects of the drugs outweigh the benefits.

Active agents

How well did you know this?
1
Not at all
2
3
4
5
Perfectly
38
Q

thiazolidinedione.- “glitzones” ass with advesre cardiovascular effects in HF

linagliptin

Gliclizide-Sulphomylurea, last line oral for T2DM

A

Gliclizide’ is a sulfonylurea, and would not be the best option for this patient. Although it is able to be used in atherosclerotic cardiovascular disease, it is not preferable to a GLP1-agonist, and is usually a last line oral option for management of T2DM.

‘Continuing with linagliptin’ is a potential option. The patient was already on this oral medication and so likely tolerates it well. However, the literature suggests that GLP-1 agonists are preferable in patients with atherosclerotic cardiovascular disease, due to their cardiovascular benefit.

DPP4 ingibitors “agliptin”

Active agents

  • Sitagliptin
  • Saxagliptin
  • Linagliptin

Clinical profile [15][19][20]

‘Rosiglitazone’ is not the best option in this patient. Rosiglitazone is a thiazolidinedione. It is associated with adverse cardiovascular outcomes, particularly heart failure, in susceptible patients. This would not be an ideal option for this patient.

How well did you know this?
1
Not at all
2
3
4
5
Perfectly
39
Q

GLP-1 antagonist (incretins have “-aglutide): atherosclerotic HD, with renal impairment

DPP4-I (‘-agliptins”)

SGLT-I (“-agliflozin”): cardiovascular dx with HF- NOT in renal imp

Metformin - DOC monotherapy, dual therap in T2DM- lactic acidosis. Careful with SGLT-1 and DPP4-I (both need decent renal function)- can precipitate severe lactic acidosis. Cant prescribe with SUs

Sulphonylureas (SUs)- (“mides,rides,nides”) last choice oral agents. Insulin secreation. Hypoglycemia

SU analogues -(a/eglinedes), Insulin secreation. Hypoglycemia

Thiazolidinediones (‘Glitzones”)- insulin sensitizers-adverse cardiovascular effects: precipitate HF

Alpha-glucosidase inhibitors- acrabose and maglitol

A

Glucagon-like peptide-1 receptor agonists (incretin mimetics)

Active agents

  • Exenatide
  • Liraglutide
  • Albiglutide
  • Dulaglutide

Dipeptidyl peptidase-4 inhibitors (gliptins)

Active agents

  • Sitagliptin
  • Saxagliptin
  • Linagliptin

Sodium-glucose cotransporter 2 inhibitors (gliflozins)

Active agents

  • Dapagliflozin
  • Empagliflozin
  • Canagliflozin

Sulfonylureas- (“amide, zide, ride”)

Active agents

  • First generation
    • Chlorpropamide
    • Tolbutamide
  • Second generation
    • Glyburide (long-acting agent)
    • Glipizide (short-acting agent)
    • Glimepiride

insulin sectreation! The use of sulfonylureas in early stages of disease can lead to a premature loss of β-cell function, potentially impairing the effectiveness of antidiabetic treatment

Meglitinides (sulfonylurea analogue)

Active agents

  • Repaglinide
  • Nateglinide

Thiazolidinediones (glitazones, insulin sensitizers)

Active agents

  • Pioglitazone
  • Rosiglitazone

Alpha-glucosidase inhibitors

Active agents

How well did you know this?
1
Not at all
2
3
4
5
Perfectly
40
Q

Localisation of a parathyroid adenoma. imaging of choice-

A

Technetium-99m sestamibi scintigraphy has the highest positive predictive value and is the preferred primary investigation for the localisation of the adenoma.

Most common cause for primary hyperparathyroidism is a single parathyroid adenoma

Once the decision to operate is made, pre-operative radiological localisation is performed to facilitate minimally invasive surgery if the imaging is concordnant.

How well did you know this?
1
Not at all
2
3
4
5
Perfectly
41
Q

patient symptoms of generalised fatigue, weight gain and muscle weakness, in addition to insomnia and depressive symptoms are suggestive of Cushing syndrome, further supported by physical examination findings of high blood pressure, facial plethora, central obesity, easy bruising, and abdominal striae. Hypernatraemia and hypokalaemia are also commonly observed.

most appropriate test for cushings syndrome

midnight cortisol level
24-hour urinary free cortisol
High-dose dexamethasone suppression test
Plasma ACTH level
CT adrenal glands
ACTH stimulation test

A

Hypercortisolism has been demonstrated by at least two tests,

at least two cross-confirmatory results among three commonly-used tests:

  • low-dose dexamethasone suppression test,
  • 24-hour urinary free cortisol (ideally 3 separate collections), and/or
  • midnight salivary cortisol.

Determine ACTH dependence (is ACTH-pituitary/PNPS, driving the hypercortisolism) -
If ACTH levels high -pituitary/ectopic ACTH production

If adrenal pathology (low ACTH- primary hypercortisolism (independent of ACTH))

A high-dose dexamethasone suppression test helps localise ACTH-dependent disease to the pituitary gland (Cushings disease).

CT adrenal glands is performed to characterise ACTH-independent disease (primary Cushings Sydrome) , which typically reflects either adrenal hyperplasia or a unilateral adrenal mass

ACTH stimulation TEST is used for ADDISONS DISEASE (adrenal insufficiency)

How well did you know this?
1
Not at all
2
3
4
5
Perfectly
42
Q

What is orlistat? When is it prescribed

A

Orlistat inhibits pancreatic lipases and prevents absorption of dietary fat, thus potentially resulting in steatorrhoea. This side effect may be improved by reducing dietary fat intake.

Reduced absorption of fat-soluble vitamins and steatorrhoea are common sequela to treatment with orlistat.

How well did you know this?
1
Not at all
2
3
4
5
Perfectly
43
Q

Otherwise well, A CT scan is performed and this is normal apart from an enhancing, homogenous 5cm mass in the region of the left adrenal gland. The margins are well defined and the lesion appears to be encapsulated.

Which one of the following is the most appropriate management plan?
Choose the single best answer.

  • Repeat CT in six months
    Urinary and serum hormone levels
    CT-guided needle biopsy
    Reassurance and no follow-up
    MRI scan
A

‘adrenal incidentaloma.’ In otherwords, an adrenal lesion which is identified on imaging studies performed for other reasons, in the absence of directly suspected adrenal-related symptoms. The size of this lesion may indicate that intervention is necessary depending on further information. This is an adrenal adenoma and on size alone, needs to be removed to exclude malignancy. Any tumour greater than 4cm should be considered for excision.

A first step is to determine whether the lesion is functional or not. Hence, urinary and serum hormone levels is most appropriate at this stage, considering the possible diagnoses of

  • Conn’s,
  • Cushing’s or
  • phaeochromocytoma.

Based on these results, if biopsy or surgery is indicated then further work-up may involve MRI to delineate soft tissue anatomy, or an image-guided biopsy.

It is crucial to know if the tumour is hormonally active before biopsy or surgery since certain adrenal lesions (e.g. pheochromocytoma) has very significant perioperative and intraoperative considerations for the surgeon and anaesthetist.

Reassurance alone is not appropriate, nor is a CT follow-up in 6 months.

How well did you know this?
1
Not at all
2
3
4
5
Perfectly
44
Q

What are the distingusging factors of Conns syndrome?

A

Primary hyperaldosteronism is characterized by hypokalemia and drug-resistant hypertension.

Primary hyperaldosteronism, sometimes referred to as Conn syndrome, is an excess of aldosterone caused by autonomous overproduction. It is typically due to adrenal hyperplasia (most commonly bilateral) or adrenal adenoma (typically unilateral).

Primary hyperaldosteronism is a common cause of secondary hypertension, occurring in > 5–12% of hypertensive patients.

High systemic aldosterone levels result in increased renal sodium reabsorption and potassium secretion, which lead to water retention and hypokalemia.

Patients are often asymptomatic and found to have hypertension at routine health checks. Hypertension due to primary hyperaldosteronism is often resistant to pharmacotherapy, and patients may have other signs suggestive of secondary hypertension, such as onset before the age of 30 or after the age of 55.

If symptoms are present, these are usually manifestations of hypokalemia (e.g., headache, muscle weakness, and polyuria).

Features of hypokalemia

Initial laboratory values in primary hyperaldosteronism classically show hypokalemia, metabolic alkalosis, high plasma aldosterone concentration (PAC), and low plasma renin activity (PRA).

The plasma aldosterone-to-renin ratio is the initial screening test, followed by confirmatory testing.

Further subtyping with imaging and/or adrenal venous sampling can determine whether aldosterone hypersecretion is unilateral or bilateral, which guides management. Treatment of unilateral disease consists of surgical resection, whereas bilateral disease is managed medically with aldosterone antagonists (e.g., spironolactone, eplerenone).

How well did you know this?
1
Not at all
2
3
4
5
Perfectly
45
Q

when to suspect pheochromo. Diagnostic test

A

Clinical presentation is related to fluctuating levels of excess epinephrine, norepinephrine, and dopamine secretion, therefore, symptoms are relapsing and remitting (episodic hyperadrenergic syndrome).

Best initial test: plasma free metanephrines test [4]- Metanephrines are metabolites of catecholamines

How well did you know this?
1
Not at all
2
3
4
5
Perfectly
46
Q

Iodine supplements? when to give hyper or hypo thyroidism?

A

Hypothyroidism

How well did you know this?
1
Not at all
2
3
4
5
Perfectly
47
Q

. Grave’s disease that proves to be ineffective to antithyroid drugs. In addition, the Grave’s ophthalmopathy seems to be worsening with severe exophthalmos, new onset of diplopia and lid retraction. How to manage? ATD hasnt helped..

Options:

Total thyroidectomy/Radioiodine ablation/ ATD (Propylthiouricil, Carbimazole)/ B-blockers

A

RIA- option for graves. but can worsen enopthalmos. Total thyroidectomy- best option

A new ATD wont change enopthalmos

B-lockers just control hypthyroid sympathetic symptoms

The most definitive treatment in Grave’s disease, in this case, is a thyroidectomy. This patient is a young female that has no co-morbidities. She would be a fit candidate for surgery.

Radioactive iodine is also another definitive treatment of Grave’s disease. This treatment is useful for patients with high surgical risk or previous surgery irradiation of the neck. However, radioactive iodine may initially worsen the appearance and symptoms of her Grave’s ophthalmopathy. Hence, radioactive iodine is contraindicated in patients with severe graves ophthalmopathy.

Her antithyroid drug (carbimazole) has failed to achieve euthyroidism in this individual’s Grave’s disease. Switching one antithyroid drug to another (propylthiouracil) will most likely achieve similar ineffective results.

Beta-blockers only provide symptomatic relief for Grave’s disease and is not a definitive treatment.

Iodine supplements are used in hypothyroidism instead.

How well did you know this?
1
Not at all
2
3
4
5
Perfectly
48
Q

Fragility fracture with gynaecomastia in a tall man, ex reveals small testes with normal penis.

Diagnosis?

A
  • Cirrhosis
  • Klinefelter syndrome
  • Kallmann syndrome
  • Testicular malignancy
  • Prolactinoma

Fragility fracture with gynaecomastia in a tall man is suggestive of Klinefelter syndrome. Klinefelter syndrome is the most frequent chromosome disorder in males, characterised by the phenotype if the tall man with relatively long legs, small testes and gynaecomastia. Hypogonadism usually first presents in early adulthood and progresses with ageing. Other important differentials for gynaecomastia include prolactinoma, which may present with headaches and visual changes, cirrhosis, and Kallmann syndrome.

Kinefelter

How well did you know this?
1
Not at all
2
3
4
5
Perfectly
49
Q

Gonadal dysgenesis- female phenotype

A

Gonadal dysgenesis

Results in different degrees of hypogonadism and sex development

Typically results in the development of malfunctioning gonads (streak gonads) with fibrous tissue instead of normal germ cells, increasing the risk of malignancy (e.g., dysgerminoma, seminoma)

  • Turner syndrome: 45,XO
  • 46,XY gonadal dysgenesis
  • 46,XX gonadal dysgenesis

Turners: Karyotype

How well did you know this?
1
Not at all
2
3
4
5
Perfectly
50
Q

Male phenotype D/O sexual development

A

Phenotypically male

  • Kinefelter
  • 46,XX testicular disorder of sex development
  • Kallmann syndrome

Kallmann Syndrome (hypogonadotropic hypogonadism with hyposmia/anosmia)

How well did you know this?
1
Not at all
2
3
4
5
Perfectly
51
Q

Hyperthyroid states

A
  • Thyrotoxicosis: a hypermetabolic condition caused by an inappropriately high level of circulating thyroid hormones irrespective of the source.
  • Hyperthyroidism: a condition characterized by the overproduction of thyroid hormones by the thyroid gland; can cause thyrotoxicosis
    • Overt hyperthyroidism
    • Subclinical hyperthyroidism
      • ↓ Serum TSH levels with normal serum free T4 and T3 levels
      • Patients are normally asymptomatic or mildly symptomatic.
      • May progress to overt hyperthyroidism

Thyrotoxicosis: Can be due to overproduction by either the thyroid gland (i.e., hyperthyroidism) or an ectopic source, inappropriate release of thyroid hormone (e.g., in thyroiditis), or ingestion of exogenous thyroid hormone (accidental or intentional)

How well did you know this?
1
Not at all
2
3
4
5
Perfectly
52
Q

Hypothyroid States

A
How well did you know this?
1
Not at all
2
3
4
5
Perfectly
53
Q

Hyperthyroid (low TSH)- most common ddx

A

Graves

An autoimmune condition of the thyroid gland in which circulating TSH receptor autoantibodies lead to overstimulation of the thyroid gland and excess thyroid hormone production. Classic clinical signs include diffuse goiter, ophthalmopathy, and pretibial myxedema.

TMNG

Toxic multinodular goiter: Abbreviation: TMNG, Plummer disease

A multinodular enlargement of the thyroid gland with TSH-independent functioning of some nodules. It is the second most common cause of hyperthyroidism.

Toxic adenoma

A benign, solitary nodule in an otherwise nonpalpable thyroid gland. Caused by a gain-of-function mutation of the TSH receptor gene in a single precursor cell, which results in autonomous production of thyroid hormones

Postpartum thyroiditis

A lymphocytic thyroiditis that typically occurs 1-12 months after delivery and manifests with a period of transient thyrotoxicosis followed by hypothyroidism without thyroid enlargement or tenderness. Histologically characterized by lymphocytic infiltration and formation of germinal centers. Usually resolves spontaneously.

Subacute granulomatous thyroiditis Giant cell thyroiditis, De Quervain thyroiditis

A transient and self-resolving patchy inflammation of the thyroid gland that is associated with granuloma formation. Often occurs after a viral upper respiratory infection and is more common among women. The clinical course is typically triphasic, beginning with hyperthyroidism, followed by hypothyroidism, and finally a return to the euthyroid state. Classically presents with tender goiter, elevated ESR, and jaw pain.

How well did you know this?
1
Not at all
2
3
4
5
Perfectly
54
Q

Hyperthyroid (low TSH) approach to investigations

A

Anti-TRab, Throid scintography (RAI), US, MRI, TPO-ab

How well did you know this?
1
Not at all
2
3
4
5
Perfectly
55
Q

Subacute thyroiditis most commonly occurs after infection?

A

Viral infection. One of the only causes of hyperthyroidism to present with a sore t tender gland.

How well did you know this?
1
Not at all
2
3
4
5
Perfectly
56
Q
A

Anti-TRab, Throid scintography (RAI), US, MRI, TPO-ab

Thyroid scintigraphy [3][10]

How well did you know this?
1
Not at all
2
3
4
5
Perfectly
57
Q

18-year-old woman is brought into the Emergency Department. Over the past three days, she has noted progressive weakness in the lower limbs and is now unable to mobilise. She was recently unwell with a prolonged course of diarrhoea three weeks ago. On examination, there is decreased tone, decreased power, and absent reflexes in her lower limbs. There is also loss of sensation in both her legs in a stocking distribution up to her mid-calf. Her pupils are equal and reactive, with a normal fundoscopy. Blood tests were unremarkable, except for a C-reactive protein 20mg/L [normal range <10mg/L].

Which one of the following investigations would provide the most diagnostic information when performed acutely?

A

This is a classical description of the clinical presentation and course of Guillain-Barré Syndrome (GBS). GBS often emerges after a preceding gastrointestinal or respiratory infection, as a result of molecular mimicry stimulating an immune response against peripheral nerves. Common precipitants include Campylobacter jejuni (which is suggested by the prolonged bout of diarrhoea she experienced), cytomegalovirus, Epstein-Barr virus and human immunodeficiency virus. Alternative triggers include immunisation, especially against influenza, as well as trauma and bone-marrow transplantation.

GBS presents with a progressive, symmetric muscle paresis +/- hyporeflexia +/- sensory and/or autonomic abnormalities, which often begins distally in the lower limbs.

Lumbar puncture (LP) needs to be performed in suspected GBS to exclude other infective or inflammatory causes of an acute flaccid paralysis. CSF analysis in GBS often reveals an elevated CSF protein with normal CSF white blood cell count, known as albuminocytologic dissociation. This investigation is the most pressing initial test to order.

How well did you know this?
1
Not at all
2
3
4
5
Perfectly
58
Q

neck masses

A 46-year-old woman presents with a painless swelling in her neck (image). It has been present for three weeks. She is otherwise in good health. The lump is non-tender and smooth-surfaced.

Ultrasonography demonstrated that it is cystic.

likely

A

Learning Points

  • Thyroglossal cysts are the most common cause of midline neck masses in children
  • A common way to establish the cause of a neck lump is its location
  • Cystic hygromas are seen more commonly in paediatric populations
  • Branchial cysts are commonly located in the upper third of the anterior aspect of the sternocleidomastoid muscle

The neck lump described and displayed in the image can be best identified by its location in the neck. It is sitting about two-thirds of the way up the anterior aspect of the sternocleidomastoid muscle. This is the typical site of a branchial cyst. These congenital abnormalities typically appear in the second or third decades of life and occasionally, as in this patient, can appear later. They are often asymptomatic but can become enlarged and tender if they become infected and frequently change in size during an upper respiratory tract infection.

Thyroglossal cysts are one of the most common causes of midline neck lumps. They are located more anterior-medially on the neck below the hyoid bone and are developed from remnants of the thyroid gland. They most commonly present during childhood between the years of 2-10.

Cystic hygromas are congenital malformations of the lymphatic ducts. They are also common in paediatric populations and are very rare to present in adulthood. They are commonly found in the posterior triangle of the neck.

An epidermoid cyst is derived from the upper portion of a hair follicle and are filled with keratin. They are typically 1-3cm in size (much smaller than the lump in the image) and can often be diagnosed clinically based on their appearance - firm, round, yellowish, fixed nodule.

Caseating necrosis is frequently seen in tuberculosis infections and other infectious diseases. It describes a form of cell death where the tissue forms a cheese-like appearance. It can infect the hilar lymph nodes, but would be rare to have the appearance shown in the image or this distribution of lymph nodes in the neck and the woman is asymptomatic in terms of an infectious point of view, hence is an unlikely diagnosis.

How well did you know this?
1
Not at all
2
3
4
5
Perfectly
59
Q

Painless jaundice. → check l.f.t.s → distinguish between hepatic and post hepatic

A

If g.g.t and alp is very elevated → post hepatic obstruction. Head of pancrease (painless, constitutional) V’s cholangitis (charcotstriad of r.u.q pain, fever, jaundice) → painless jaundice → head of pancrease.

How well did you know this?
1
Not at all
2
3
4
5
Perfectly
60
Q

A 69-year-old woman presents with a three-week history of anorexia and weight loss. Over the last three days she has noticed yellowing of her skin and eyes, dark urine and pale stools. She had recently returned from a holiday in Bali and had been taking doxycycline for prevention of malaria. She has no other past medical history and takes no regular medications. On examination she is haemodynamically stable, saturating well on room air and afebrile. She has significant jaundice affected her skin and sclera. She has multiple scratch marks across her arms and chest. Her abdomen in non-tender and murphy’s sign is negative. Bowel sounds are present and the liver span is of normal size. The liver function test results are: Bilirubin 293µmol/L (<25) Alkaline phosphatase (ALP) 1914U/L (40-130) Gamma glutamyl transpeptidase (GGT) 859U/L (<50) Serum alanine transaminase (ALT) 361U/L (<51) Serum albumin 31g/L (35-50)

A

Painless jaundice must always raise a suspicion for pancreatic cancer until proven otherwise. Furthermore, from this history the unexplained weight loss and anorexia hint at a potential malignancy diagnosis. Pancreatic cancer causes obstructive jaundice when the mass becomes large enough to obstruct the bile drainage, resulting in a post-hepatic cholestasis. All of the conditions listed could cause jaundice the biochemical picture is that of obstruction (very high GGT and ALP suggest post-hepatic obstruction) and that limits the likely diagnoses to one of the two post-hepatic problems; pancreatic cancer or cholangitis. But what makes pancreatic cancer more likely is her age and the painless nature. Cholangitis is acute infection of the bile duct. This presents with Charcot’s triad of abdominal pain, jaundice and fevers/chills. She does not display any of these symptoms besides the jaundice. Furthermore, despite her recent trip to Bali and potential exposure to viral hepatitis, she does not display the typical flu-like symptoms, fever and abdominal pain. Her LFTs also do not reflect an acute hepatitis picture, where the AST and ALT would be more raised. Similarly, a liver abscess may create abdominal pain or discomfort and deranged ALT and AST more than GGT and ALP. Doxycycline can cause hepatocellular damage and hence jaundice, but this is rare and often occurs in combination with other medications. Malaria can also cause jaundice, but she does not display the typical features of malaria and if she was taking doxycycline effectively she should have avoided this risk. Learning Points Painless jaundice must always cause concern for pancreatic cancer Cholestasis causes raised ALP and GGT more than AST and ALT Acute cholangitis can present with Charcot’s triad of jaundice, fevers and abdominal pain

How well did you know this?
1
Not at all
2
3
4
5
Perfectly
61
Q

A previously well 45-year-old man attends the Emergency Department after vomiting a small amount of fresh blood (less than half a cupful). One week prior to admission he had started a course of diclofenac for a back injury which he sustained while lifting a heavy weight at the gym. His pulse rate is 90/min, blood pressure on lying is 130/80 mmHg, with no postural drop. His haemoglobin is 135 g/L and his serum biochemistry results are all within the normal ranges. Which one of the following would be the most appropriate management plan? Choose the single best answer. Outpatient-based surveillance Admit and observe Admit for blood transfusion Urgent endoscopy Admit, transfuse and endoscopy the following day

A

Upper gastrointestinal bleeding is a common problem encountered in the Emergency Department. The goal of the initial assessment of a patient suffering from an upper gastrointestinal bleed is to determine the severity - thus allowing decisions to be made regarding management. A thorough history should be taken, particularly assessing for possible aetiologies, while an examination should focus on determining haemodynamic stability. For high-risk patients who are haemodynamically unstable, emergent management including fluid resuscitation and urgent endoscopy to achieve haemostasis. For haemodynamically stable patients, risk stratification tools such as the Glasglow Blatchford score (GBS) may be used to determine severity. This tool takes into account history features such as cardiac or hepatic past medical history, syncope and melaena, features of haemodynamic stability, and laboratory values of haemoglobin and urea. For a GBS greater than 1, an inpatient endoscopy is recommended. When the GBS for a patient is less than 1, then they are at low-risk and should be managed on an outpatient basis, for the benefit of the patient, and this hospital. According to the GBS screening tool, and his clinical stability, this patient does not need hospitalisation or urgent medical intervention. He is normally in good health, has no evidence of haemodynamic instability, has normal haematological and biochemical measurements and could be quite easily managed on an outpatient basis - provided he has supports at home and has access to medical assistance, should it be needed. An endoscopy can be arranged semi-electively to confirm the almost certain diagnosis of NSAID-induced ulceration. He should be sent home on a proton pump inhibitor and instructed not to take any more diclofenac. It is estimated that using the GBS tool, 15% of patients with an acute upper gastrointestinal haemorrhage can be classified as low-risk and managed safely on an outpatient basis.

How well did you know this?
1
Not at all
2
3
4
5
Perfectly
62
Q

Upper gi bleed in patient. What criteria/considerations to use to decide on Mx in patient/ out patient o urgent endoscopy.

A

Bus criteria. ① history of liver or cardiac disease. ② any syncope or maleana O ③ heamodynamically stable? ④ abnormal hb or urea. GBs >l endoscopy. If less than I patient can be managed safely in outpatient w endoscopic review tor cause. Discharge W psi.

How well did you know this?
1
Not at all
2
3
4
5
Perfectly
63
Q

Order of sequence for occult bleeding investigations.

A

If active bleeding: (ctimr). angiography It no active bleeding: 1 colonoscopy and endoscopy. Repeat if inconclusive. Lie do C and G scope twice. ② capsule endoscopy ③ if inconclusive enteroclysis (ct or MRI).

How well did you know this?
1
Not at all
2
3
4
5
Perfectly
64
Q

The patient has no gastrointestinal symptoms. The photograph shows the view as the procedure is about to start. The lesions are manually reducible. Assuming that the colonoscopy is otherwise normal, which one of the following is the most appropriate initial advice to give to the patient? Choose the single best answer. High fibre diet Sclerotherapy Banding Incision and drainage Haemorrhoidectomy

A

The view shows two partially prolapsed internal haemorrhoids, and given they are manually reducible, this classes them as Grade 3. The grading of haemorrhoids is as follows: Grade 1: Do not prolapse; located above the dentate line; often bleed Grade 2: Prolapse with straining; spontaneously reduce Grade 3: Prolapse with straining; only manually reduce Grade 4: Irreducible prolapse; may be thrombosed or ulcerated If the haemorrhoids remain asymptomatic, they should be able to be managed conservatively with a high fibre diet. Other conservative measure may include education regarding straining/lingering while passing stools, regular physical exercise, and avoiding medications and food that may cause diarrhoea or constipation. If mild symptoms develop, topical agents may be offered. If more troublesome symptoms develop, rubber band ligation should be offered as an outpatient. Banding is preferred to sclerotherapy as it has been shown to be more effective and require less treatment sessions. However, sclerotherapy should be offered to patients who are on antiplatelet or anticoagulation therapy as the bleeding risk is significantly reduced. Haemorrhoidectomy is reserved for patients with severely symptomatic Grade 3 or 4 disease. Learning Points Asymptomatic haemorrhoids can be managed conservatively with a high fibre diet Rubber band ligation is the procedure of choice for symptomatic disease

How well did you know this?
1
Not at all
2
3
4
5
Perfectly
65
Q

A 56-year-old man is being investigated for what he describes as a “volcanic-like” burning sensation in the lower chest after eating. This has got progressively worse over the last few years. An oesophago-gastroscopy is performed. The following was seen during the procedure. Which one of the following is the most likely diagnosis?

A

This man’s symptoms of retrosternal burning after eating indicate likely gastro-oesophageal reflux disease (GORD). Chronic GORD is one of the key risk factors for development of Barrett’s oesophagus, which is characterised by metaplastic change just proximal to the gastroesophageal junction (GOJ) where the normal squamous epithelium is replaced by intestinal columnar epithelium. In this case the areas of salmon-pink columnar epithelium can be seen extending some 3 - 4 cms above the cardia with tongues going up into the squamous epithelium. Barrett’s oesophagus can become dysplastic and progress to adenocarcinoma if left untreated. Biopsy results are necessary to confirm this diagnosis or indicate if it has become cancerous. Squamous cell carcinoma of the oesophagus tends to occur in the upper and middle portions of the oesophagus. These patients more typically present with dysphagia and often have a history of smoking and/or alcohol intake. These tumours can be difficult to identify on endoscopy during the early stages and may require staining with Lugol’s iodine to be visualised. Whilst this man’s symptoms are consistent with gastro-oesophageal reflux, there is no evidence of inflammatory change. Eosinophilic oesophagitis tends to occur in younger patients and is associated with dysphagia. Although a hiatal hernia may cause similar symptoms, it is diagnosed when there is >2cm proximal dislocation of the GOJ, i.e. the upper margin of the stomach folds can be seen above the diaphragmatic indentation.

How well did you know this?
1
Not at all
2
3
4
5
Perfectly
66
Q

Q1 -More definitive treatment of achalasia- grade 1 and 2

vs

Q2 -Manometric studies confirm type III achalasia. Extensive ( possibly with spasm )

most appropriate plan of management?

Botulinum toxin
Pneumatic dilatation
Cardiomyotomy and fundoplication
Peroral endoscopic myotomy
Oesophagectomy

A

Laparoscopic cardiomyotomy with fundoplication. With the development of high-resolution manometry, a more subtle distinction can be made between the various types of achalasia.

The Chicago classification of motility disorders describes three variants for achalasia, where Type I details the classical changes of a raised lower oesophageal sphincter pressure and no peristalsis in the body of the oesophagus. Treatment in this group of patients is aimed at reducing or abolishing the lower oesophageal sphincter, to allow food to ‘drain’ out of the oesophagus.

Botulinus toxin can produce good short-term results, but its durability is measured in months. Balloon dilatation can give effective long-term results - but can be complicated in about 15% of patients with troublesome gastro-oesophageal reflux. Another problem with balloon dilatation is the frequent need for repeated procedures and the risk of the patient then requiring surgery and the technical challenges from the resultant scar tissue.

Oesophagectomy is reserved for those with end-stage disease - and in selected patients, can give excellent relief of symptoms.

The peroral endoscopic myotomy (POEM) is a relatively new endoscopic approach to the division of the lower oesophageal sphincter and probably is most applicable to Chicago Type III cases (those with spastic contractions of the body of the oesophagus). Current experience with POEM suggests a 50% risk of long term proton pump inhibitor use to control gastro-oesophageal reflux symptoms brought about by the procedure.

A laparoscopic cardiomyotomy will produce good symptomatic relief of the troublesome dysphagia and regurgitation associated with this type of achalasia. The addition of a partial fundoplication will minimise any gastro-oesophageal reflux.

Botulinum toxin injection may provide immediate relief of symptoms, but its duration of action is measured in months. This form of treatment might be suitable for high-risk patients.

Pneumatic dilatation can produce effective relief of symptoms and keep patients in remission for 5 - 10 years. This treatment approach is most effective for patients with type II achalasia.

Laparoscopic cardiomyotomy can produce effective and sustained relief of symptoms in some 90% of patients and is most effective in young patients with types I or II achalasia and a tortuous oesophagus.

Peroral endoscopic myotomy (POEM) is the most recent treatment option for achalasia and is showing similar results to laparoscopic cardiomyotomy - but with a higher incidence of gastro-oesophageal reflux disease. With the long submucosal tunnel formed in a POEM procedure, the myotomy length exceeds that achieved at laparoscopic cardiomyotomy and is the preferred treatment for type III achalasia.

Oesophagectomy is reserved for those with end-stage achalasia where other treatments have failed. Despite the magnitude of the procedure, it can provide a major improvement to the patient’s overall quality of life.

How well did you know this?
1
Not at all
2
3
4
5
Perfectly
67
Q

UTI

most likely organism in UTI vs catheter associated UTI ( and urolithiasis present)

A

Whilst E. coli is the organism most usually associated with urinary tract infections

  • *Learning Points**
  • *Proteus mirabilis is a urea-splitting organism that promotes the formation of urinary tract stone**

the presence of urolithiasis in this case suggests that the likely responsible organism is Proteus mirabilis. This organism is a common cause of catheter-associated urinary infections. It is a Gram-negative rod, facultatively anaerobic, and produces the enzyme urease. This urease activity converts urea in the urine into ammonia, leading to alkalinisation of the urine, often with pH upwards of 8.0. A

lkalinised urine also facilitates magnesium ammonium phosphate precipitates (i.e. struvite stones) which can form kidney stones - which in this case was confirmed on the abdominal X-ray.

Klebsiella pneumoniae is another cause of catheter associated urinary tract infections, and is also associated with mildly alkaline urinary pH as it produces small amounts of urease, the enzyme responsible for the conversion of urea into ammonia. However, it typically does not present with urinary pH levels above 8 and is only associated with nephrolithiasis in rare cases.

Pseudomonas aeruginosa is also commonly associated with catheter associated infections, especially due to its ability to form biofilms that are resistant to antibiotic therapy. However, it does not cause alkaline urine nor does it cause nephrolithiasis.

S. saprophyticus is a common cause of urinary tract infections in young, sexually active women, but is uncommon in older and/or catheterised patients. Also, it does not cause alkalinisation of urine nor does it predispose towards the development of kidney stones. Additionally, staphylococcal species are unable to reduce nitrate to nitrite hence the urinary nitrite levels will be negative if it were to be a staphylococcal infection.

How well did you know this?
1
Not at all
2
3
4
5
Perfectly
68
Q

On examination, there is some tenderness over the L5-S1 vertebral foramina. The pain is reproducible with leg flexion, accompanied by abduction and external rotation…

what test is described and what does this indicate.

A
  • buttock pain and worse with movements equivalent to the FABER test represents a classic presentation of sacroiliitis.
    • FABER test (Patrick test)
      • Procedure: Flexion, ABduction, External Rotation
        • The examinee lies supine.
        • The examiner sequentially performs passive hip flexion, abduction, and external rotation, so that the patient’s legs are arranged in a figure-of-4 position.
        • After performing the final rotation, the examiner gently pushes down on the knee of the folded leg.
      • Interpretation: The emergence of pain in the hip or sacroiliac joints indicates pathology in the corresponding structure.
How well did you know this?
1
Not at all
2
3
4
5
Perfectly
69
Q

Back pathology asscoiated with crohns disease

A 35-year-old man presents with a six-month history of pain in his upper buttocks. The pain is most troublesome at night, when it may awaken him from sleep, and in the morning as he feels stiff when he wakes up. He finds that as the day progresses the pain subsides and it also improves with exercise. The pain is not relieved by paracetamol and he was advised to avoid NSAIDs due to the risk of exacerbating his Crohn’s disease. He has no other symptoms. Crohn’s disease was diagnosed five years earlier and is well controlled with sulfasalazine.

On examination, there is some tenderness over the L5-S1 vertebral foramina. The pain is reproducible with leg flexion, accompanied by abduction and external rotation.

Which one of the following investigations is most appropriate?

  • *C-reactive protein**
  • *HLA-B27**
  • *Plain X-ray of the lumbar-sacral region**
  • *MRI sacroiliac joints**
  • *Bone scan**
A

This patient’s presentation of inflammatory back pain, buttock pain and worse with movements equivalent to the FABER test represents a classic presentation of sacroiliitis.

Inflammatory back pain

A type of nontraumatic back pain that occurs secondary to inflammatory processes such as ankylosing spondylitis, psoriatic arthritis, reactive arthritis, and juvenile idiopathic arthritis. The pain most commonly affects the lumbar spine and is characteristically insidious in onset, worse at night or at rest, and relieved by activity. Affected individuals are typically < 40–45 years of age with raised ESR and CRP levels on laboratory studies.

Sacroiliitis occurs in 15% of patients with inflammatory bowel disease.

A plain X-ray is the first-line investigation, due to cost, availability and high-degree of clinical suspicion. A plain X-ray may show sacroiliac joint sclerosis in this patient. Ideally, this should be treated with exercise and/or non-steroidal inflammatory drugs (NSAIDs). As the patient cannot take NSAIDs, he should be encouraged to exercise and seek physiotherapist input.

C-reactive protein may be useful in determining if this patient is currently experiencing an exacerbation of inflammatory bowel disease, however, it is unlikely to be of any use in differentiating the cause of this pain.

The HLA-B27 gene is associated with rheumatological conditions such as ankylosing spondylitis (90-95%) and inflammatory bowel disease with sacroiliitis (50-60%), it may be present in this patient but does not aid in the diagnosis of his current presentation.

An MRI is the best imaging modality, and is able to detect early disease, however it carries a high cost and is dependent upon availability, and thus may be considered if an X-ray was negative. A bone scan is inappropriate due to relatively poor sensitivity and relatively high radiation load and thus is best avoided in a young patient.

Learning Points
Sacroiliitis is a common association of inflammatory bowel disease

How well did you know this?
1
Not at all
2
3
4
5
Perfectly
70
Q

FEV1 and FEV1/FVC in obstructive vs restrictive

A

(2) Obstructive lung diseases result in a concave flow-volume loop due to increased resistance. Absolute FEV1, as well as FEV1 relative to vital capacity (FEV1/FVC), is diminished. BOTH ARE DECREASED
(3) Restrictive lung diseases result in a narrowed flow-volume loop due to reduced vital capacity and compliance. Absolute FEV1 is diminished, but not in relation to vital capacity (FEV1/FVC).

© AMBOSS

How well did you know this?
1
Not at all
2
3
4
5
Perfectly
71
Q

How does parenchymal damage affect lung function.

A

Normal FEV1 with decreased VC, decreased RV, and decreased lung compliance is typically seen in patients with restrictive lung disease due to an intraparenchymal pathology (e.g., pulmonary fibrosis). Ankylosing spondylitis results in dyspnea due to restricted mobility of the thoracic cage and spine; the lung parenchyma itself remains unaffected.

Individuals with ankylosing spondylitis frequently develop some of the classical features of restrictive lung disease (i.e., normal/decreased FEV1 and decreased FVC) because of impaired mobility of the spine and thoracic cage. In contrast to most forms of restrictive lung diseases, lung involvement in ankylosing spondylitis manifests with a normal FRC and a normal/increased RV because the lung parenchyma, and therefore lung compliance, are unaffected.

Most other restrictive lung diseases are due to intraparenchymal lung pathology and manifest with decreased RV.

How well did you know this?
1
Not at all
2
3
4
5
Perfectly
72
Q

Treatment of ankylosis Spondaloaropathy

A

Etanercept

Additional treatment with a TNF-α inhibitor, such as etanercept, is indicated for improving spine mobility and reducing pain in patients with AS who have persistent symptoms despite NSAID use, which is the first-line treatment for ankylosing spondylitis, for > 2 months. Some studies suggest that these agents may also slow down radiographic disease progression after several years of treatment.

In addition, every patient should receive regular physical therapy to maintain range of motion and posture.

TNF inhibitors
A group of medications used to treat chronic inflammatory diseases such as rheumatoid arthritis, ankylosing spondylitis, and Crohn disease. Examples include etanercept, infliximab, and adalimumab.

among the DMARDs used in the treatment of AS, sulfasalazine is preferred over MTX.

MTX is used as first-line treatment of rheumatoid arthritis and psoriatic arthritis. RA of the spine can also result in back pain that improves throughout the day and is commonly associated with a positive family history. As for psoriatic arthritis, patients may also present at this age with spinal joint pain (in up to 40% of cases) and ankylosis on x-ray that may cause restricted movement. However, this man lacks other important findings of both conditions, such as arthritis of the phalangeal joints.

  • *DMARDs such as leflunomid-**
  • *Leflunomide** is used as first-line treatment of nonsevere psoriatic arthritis. Psoriatic arthritis may also manifest at this age with spinal joint pain (in up to 40% of cases) and ankylosis on x-ray that may cause restricted movement. However, this man lacks other important findings of psoriatic arthritis, such as arthritis of the phalangeal joints or nail changes (pitting, onycholysis).
How well did you know this?
1
Not at all
2
3
4
5
Perfectly
73
Q

Common associations of AS:

A

Common associations of AS:

Ankylosing spondylitis is a seronegative spondyloarthropathy and is associated with extraarticular manifestations, the most common of which is acute unilateral anterior uveitis (∼ 25% of cases).

Further findings in this patient would, therefore, most likely include eye redness and pain. Other associated, though rarer, extraarticular findings of ankylosing spondylitis include gastrointestinal (inflammatory bowel disease), cardiac (aortic insufficiency, AV block), and renal (IgA nephropathy).

How well did you know this?
1
Not at all
2
3
4
5
Perfectly
74
Q

Inferior vena caval obstruction
Portal vein thrombosis
Hepatic vein occlusion
Cirrhosis
Superior vena cava obstruction

A

The patient is thin, but has a massively distended abdomen. There is a collection of subcutaneous veins running around the umbilicus (also known as ‘Caput Medusa’ when localised at the umbilicus). The abdominal distension is due to ascites and the dilated veins reflect portal hypertension and the opening up of collateral vessels (umbilical vein) to carry blood back to the systemic circulation. The most common cause of portal hypertension is cirrhosis, mostly secondary to alcohol abuse in this patient. Chronic liver disease and cirrhosis results in many symptoms/signs which are easily examinable and candidates should have a clear approach to examination of these patients.

Inferior vena caval (IVC) obstruction results in blood pooling in the IVC causing a greatly decreased venous return to the heart and subsequent haemodynamic compromise; which is not present in this patient. In addition, any subcutaneous venous collaterals would extend from the lower to upper abdomen, as a reflection of dilatation of the inferior and superior epigastric vessels.

Portal vein thrombosis could lead to these physical signs but is less common than cirrhosis and does not fit the history as well. - in acute PVT more congestive symptoms (hepatorenal syndrome, fever oesophageal)

Hepatic vein occlusion (Budd-Chiari syndrome) is quite uncommon but could be suggested if the patient had pro-thrombotic risk factors such as a myeloproliferative disorder.

SVC obstruction commonly occurs with lung cancer externally compressing (possible in this patient with a smoking history), however it presents with oedema of the upper extremities/neck, chest pain and cough.

How well did you know this?
1
Not at all
2
3
4
5
Perfectly
75
Q

Pagets Disease- Clinical Features?

A

This patient’s hearing difficulties, bone pain, and tibial bowing raise suspicion for Paget disease of bone.

Bisphosphonates, which inhibit osteoclast-mediated bone resorption, are the first-line treatment for Paget disease of the bone.

Clinical features

How well did you know this?
1
Not at all
2
3
4
5
Perfectly
76
Q

GIST vs Gastric Malt on endoscopy

microcytic anaemia and an endoscopy is arranged. This is the view of the anterior wall of the body of the stomach.

A

The photograph shows a well-defined mass lesion on the anterior wall of the body of the stomach that is protruding into the lumen. The overlying mucosa is intact apart from ulceration in the middle of the lesion. This is the typical appearance of a gastrointestinal stromal tumour (GIST).

GISTs are a malignant mesenchymal tumour that arise from interstitial cells of Cajal in the gastrointestinal tract. 60% of these are found in the stomach, 35% in the small intestine and 5% elsewhere (colon, rectum, oesophagus or omentum). These tumours often remain asymptomatic and are found incidentally. Large (>2 cm) tumours can present in the 6th decade of life with anaemia and upper gastrointestinal bleeding.

Gastric MALT lymphomas frequently present as infiltrative, ulcerated lesions arising from the deep mucosa or submucosa.

Linitis plastica is a type of gastric cancer that commonly occurs in the fundus. It is characterised by thickening of the stomach wall and deformation of the stomach, leading to a ‘leather bottle-like’ appearance. It is not associated with discrete lesions, as shown in the photograph.

Type II neuroendocrine tumours are associated with Zollinger-Ellison syndrome, which is an acid hypersecretion state triggered by increased gastrin production. Endoscopy in this case would exhibit single or multiple polypoid nodules in the fundus and antrum, with multiple peptic ulcers.

Gastric varices tend to bleed quite severely, often presenting with frank haematemesis, haemodynamic instability (sometimes hypovolaemic shock) and abdominal pain. These often look like serpiginous vascular structures, and are associated with hepatic cirrhosis in the Australian population.

Learning Points
GISTs are most commonly found in the stomach.
Large GISTs can present with symptoms of anaemia and upper GI bleeding, but they are often found incidentally.

How well did you know this?
1
Not at all
2
3
4
5
Perfectly
77
Q

A 28-year-old woman presents with a three-month history of abdominal pain, fatigue, and diarrhoea which frequently contains blood and mucus. Her symptoms have worsened recently and she now has to defecate 5-6 times per day. She describes faecal urgency and an occasional feeling of incomplete defaecation. She does not have any recent travel history and has not taken any antibiotics recently. Her temperature is 37.2 C, blood pressure 110/75 mmHg, pulse 108/min and respiratory rate 18/min. On physical examination several erythematous painful nodules are noticed on the anterior aspect of her shins. Her blood tests are shown below:

Haemoglobin = 100 g/L (115-165 g/L)
WBC = 13.0 x 109/L (3.6-11.0 x 109/L)
Platelets = 450 x 109/L (140-400 x 109/L)
ESR = 15 mm/hr (3-9 mm/hr)

A colonoscopy is arranged.

Which one of the following colonoscopic features would most likely be seen?

Choose the single best answer.

Erythema, mucosal friability, granularity and ulceration

Granulomatous inflammation on histology

Pale mucosa with petechial bleeding

Colonic biopsy showing inflammation extending to the muscularis

Hyperpigmentation of colonic epithelium

A

This patient’s bloody diarrhoea, faecal urgency, tenesmus and the erythema nodosum is most suggestive of ulcerative colitis.

Ulcerative colitis is an inflammatory bowel disease characterised by abdominal pain which may be diffuse or in the left lower quadrant, blood diarrhoea, faecal urgency, and tenesmus.

Ulcerative colitis is also associated with several extraintestinal symptoms (eg, erythema nodosum, uveitis, episcleritis, primary sclerosing cholangitis).

Patients may have anaemia due to blood loss, leucocytosis, thrombocytosis, and elevated inflammatory markers (eg, ESR). Platelets are acute-phase reactants and may be elevated in inflammatory conditions. Ulcerative colitis is difficult to differentiate from Crohn’s disease based on history and physical examination, and colonoscopy is indicated to confirm the diagnosis. The appearances of erythema, friability, granularity and ulceration - together with the clinical picture - would support a diagnosis of ulcerative colitis. Colonoscopic biopsy of the colonic epithelium in ulcerative colitis may reveal inflammation confined to mucosa and submucosa. A positive p-ANCA is characteristically associated with ulcerative colitis. It may also be associated with Crohn’s disease in 10-20% of the patients, however, it is more likely to be seen in patients with ulcerative colitis.

Inflammatory changes extending to the muscularis would be more in keeping with Crohn’s disease. Colonic biopsy of Crohn’s disease shows granulomatous (non-caseating) transmural inflammation involving all the mucosal layers of the intestinal wall. Crohn’s disease is more commonly associated with non-bloody diarrhoea. However, bloody diarrhoea may be seen in severe Crohn’s disease. p-ANCA is less commonly seen in patients with Crohn’s disease. Anti-Saccharomyces cerevisiae antibodies are more commonly associated with Crohn’s disease.

Pale mucosa with petechial bleeding is a characteristic finding in ischaemic colitis and is commonly seen in patients with hypotension or atherosclerosis. This patient does not have any recent history of vascular surgery and is unlikely to have atherosclerosis at this age. Moreover, elevated inflammatory markers support the diagnosis of inflammatory bowel disease.

Hyperpigmentation of colonic mucosa (Melanosis coli) is associated with laxative abuse and is more commonly seen in females in the healthcare profession (eg, nurse). Bloody diarrhoea, elevated inflammatory markers, and p-ANCA make this diagnosis unlikely.

How well did you know this?
1
Not at all
2
3
4
5
Perfectly
78
Q

Crohns vs Ulcerative Collitis

A

Crohn disease and ulcerative colitis Crohn disease

Pathophysiology

  • Cr: Mediated by dysfunctional IL-23-Th17 signaling
  • UC: Mediated by Th2 cells

Frequency/type of defecation

Cr

  • Increased
  • Typically nonbloody, watery diarrhea
  • May be bloody in more severe cases

UC-

  • Greatly increased
  • Bloody diarrhea with mucus
  • Tenesmus
  • Urgency

Nutritional status

  • Cr-Poor or malnourished
  • UC- Mostly normal, but weight loss and malnutrition may occur in severe disease [41]

Physical examination

Cr-

  • Mostly constant pain in RLQ
  • Palpable abdominal mass
  • Low-grade fever

UC

Extraintestinal manifestations

Crones

UC

Both-

Fistulas

Crones - note fistulas are not ulcers

UC- Rare for fistulas

Other complications-

Crones

  • Abscess
  • Strictures (obstructions)
  • Perianal fissures

UC

Cancer risk

  • Both:
  • Increased due to underlying pathology
  • Increased secondary to immunosuppression

Crones

U. Colitis

Antibodies

  • Crones- ASCA, Anti-Saccharomyces cerevisiae antibodies
  • UC: p-ANCA Myeloperoxidase antineutrophil cytoplasmic antibody

Endoscopy and imaging

Crones:

  • Typical location: terminal ileum and colon with rectal sparing
  • May affect the entire GI tract

UC:

Pattern of inflammation

Typical diagnostic findings

  • Cobblestone sign
  • Linear and/or serpiginous ulcerations
  • Small aphthous ulcerations
  • Creeping fat
  • String sign- A contrast-filled bowel segment that resembles a string on x-ray. Caused by marked narrowing of the bowel lumen, which may be caused by Crohn disease (usually stenotic terminal ileum), hypertrophic pyloric stenosis (elongated and narrowed pylorus), colon cancer, carcinoid tumor, or gastrointestinal tuberculosis

UC

  • Friable mucosa- bleeds easily on colonoscopy
  • Mucosal ulcerations can be deep or superficial
  • Crypt abscesses- An infiltration of neutrophils into the lumen of intestinal crypts due to a breakdown of the crypt epithelium. Associated with ulcerative colitis.
  • Loss of haustra (lead pipe sign)

Histology

  • Cr-
  • Transmural inflammation
  • Noncaseating granulomas
  • Giant cells
  • Lymphoid aggregates (NON-non hodgkins lymphoma)
  • UC-
  • Confined to mucosa and submucosa (layers mucosa: epithelium, lamina propria (CT), muscularis mucosae. then submucosa) eg crypt abcess breaches lamina propria-→ still in mucosa
  • No granulomas

Both have

TreatmentMedication

Crones- IL23 and Th17-secreates IL17 (fevers and wide spread)

UC- Th2 (promotes B cells)

Both

Surgery

  • Cr-Noncurative surgery may become necessary to alleviate symptoms; Stricturoplasty
    • UC- Curative surgery possible (proctocolectomy)
How well did you know this?
1
Not at all
2
3
4
5
Perfectly
79
Q

Food bolus obstruction and intermittent dysphagia in a younger patient

A

Food bolus obstruction and intermittent dysphagia in a younger patient- should raise the possibility of eosinophilic oesophagitis.

On endoscopy- concentric rings with linear bands.

First determine if ture EO or PPI responsive EO. 1 high dose PPI.

Repeact endo.

If unresponsive- true EO: Mx with concurrent elimination diet and high dose oral steroids (flucatisosne)

This is a typical history of a patient with eosinophilic oesophagitis and the endoscopy shows the characteristic concentric rings which occur secondary to eosinophil initiated oesophageal re-modelling and fibrosis.

Initially a 8-week course of high dose proton pump inhibitor (PPI), such as esomeprazole, is suggested followed by repeat endoscopy and biopsy to distinguish between patients with PPI-responsive oesophageal eosinophilia from true eosinophilic oesophagitis.

Once the diagnosis of true eosinophilic oesophagitis is confirmed then the patient can be commenced on concurrent elimination diet and oral ingested corticosteroids. Oral fluticasone has been shown to have good endoscopic and histological responses to treatment, with reduction in frequency of dysphagia.

The patient may need referral to an allergist. Rather than keeping patients on long-term PPIs, preference is now given to dietary modification.

How well did you know this?
1
Not at all
2
3
4
5
Perfectly
80
Q

When is SBP diagnosed? WHat is the treatment Empiric treatment of SBP.

A

Ascities more than 250 nuetrophills is diagnostic of SBP.

Patients with frank ascites, or those with cirrhotic liver disease at risk of ascites, should be managed with a high index of suspicion for spontaneous bacterial peritonitis (SBP) when presenting with abdominal pain or tenderness, fever, hypotension, or altered mental status.

Most likely organisms Enterobacteria or streptococcus- Empiric treatment needs Grm- and gram + cover ( Cephtriaxone cephotaxime)

  • Whilst the combination of sofosbuvir and ribavirin is among the current first-line treatments for Hepatitis C based on genotype, the priority in this case is the spontaneous bacterial peritonitis.
  • A therapeutic paracentesis might be helpful, but would not treat the underlying infection.

Note no anearobes in ascitiess- lots of O2

How well did you know this?
1
Not at all
2
3
4
5
Perfectly
81
Q

management of barrets oesophagus

On this occasion the presence of 3cm long segment of Barrett’s oesophagus is confirmed. There is a single 5mm area of irregularity and 4-quadrant biopsies are performed. The only abnormality is some high grade dysplasia in the irregular area. There is no evidence of oesophagitis or hiatus hernia.

Which one of the following is the most appropriate plan of management?
Choose the single best answer.

Endoscopic review in three months
Radiofrequency ablation
Endoscopic mucosal resection
Oesophagectomy
Neoadjuvant therapy followed by surgical resection

Q2:

three month history of dysphagia on a background of gastro-oesophageal reflux, the latter which has been present for many years. An endoscopy shows an area of ulceration with clear raised margins and surrounding inflammation extending up to the mid-oesophagus. The ulcerated area is biopsied and shows columnar-lined epithelium with areas of high-grade dysplasia.

Which one of the following is the most appropriate next step in management?
Choose the single best answer.

Repeat endoscopy and biopsies

pH and manometric studies
Esomeprazole and repeat endoscopy in three months
CT chest and abdomen
Argon ablation therapy

A

High grade dysplastic tissue can be difficult to distinguish from in-situ carcinoma and once identified must be actively treated. With the advances in endoscopic therapies, the current approach involves endoscopic mucosal resection (EMR) of the abnormal region and careful histological assessment of the resected tissues. This approach is all the more important if there are raised, irregular or nodular areas of Barrett’s oesophagus.

Radiofrequency ablation (RFA) may be used in combination with endoscopic resection and particularly for non-nodular areas of Barrett’s oesophagus. Further management will depend on accurate tumour (T) staging and possible use of endoscopic ultrasonography to assess local lymph node (N) status.

For high grade dysplasia alone, patients can be kept under endoscopic surveillance and where feasible, the entire segment of Barrett’s epithelium removed.

Oesophagectomy is indicated when cancerous cells have been identified and the tumour is considered resectable and beneficial to the patient’s prognosis (i.e. no known metastases).

-→ Q2

Learning Points
An area of ulceration in Barrett’s oesophagus must raise the concern of malignancy.

The presence of a well-defined ulcer and high grade dysplasia raises strong suspicions of oesophageal malignancy.

The length of the area of inflammation also increases the risk of malignant change.

Further endoscopy and biopsies of the ulcerated area must be done.

Patients who have biopsies showing high grade dysplasia should be referred to a tertiary specialist centre for repeat endoscopy, biopsy and potentially endoscopic therapy (endoscopic mucosal resection) or surgery.

Ablation therapy should only be done if frank malignancy is excluded. A single set of biopsies showing high grade dysplasia would not be sufficient.

This patient may require a proton pump inhibitor for control of their gastro-oesophageal reflux disease (GORD), however this can not be done in isolation. Esomeprazole alone is inappropriate without further investigation.

Manometric and ph studies are not helpful as GORD is already diagnosed.

Whilst a CT chest/abdomen will likely be done if the presence of malignancy is confirmed, the most important investigation is the repeat endoscopy and biopsy.

How well did you know this?
1
Not at all
2
3
4
5
Perfectly
82
Q

suspected TIA- how to decide on management: Asprin 300MG and Emergency center within 24 hours

A

Use risk stratifying for risk of stroke:
ABCD2

low (score: 1-4)- low

medium (score 4-5) - seen within 24 hours
high (score: 6-7)

This patient may have a transient ischaemic attack (TIA), which is supported by the transient focal neurological deficits e.g. right hemiplegia and transient vision loss.

According to the NICE guidelines and Australian Clinical Guidelines for Stroke Management 2010 by the NHMRC, individuals with TIA are given 300mg aspirin (loading dose) and referred for specialist assessment.

The ABCD2 score is a clinical predictor tool that risk stratifies an individual’s future stroke risk after a suspected TIA. IF suspeected TIA- ASPRIN AND neorovascular review and imaging in 24hours.

The ABCD2 score includes:

  • Age ≥60 years = 1
  • BP ≥ 140/90 = 1
  • Changes
    • Focal weakness = 2
    • Speech disturbance without weakness = 1
  • Duration:
    • Lasting ≥60min = 2
    • Lasting 1–59min = 1
  • Diabetes = 1

In this case, his ABCD2 score is 5 which is considered high risk. Individuals with ABCD2 score greater than 4 require an assessment within 24 hours.

How well did you know this?
1
Not at all
2
3
4
5
Perfectly
83
Q

suspected TIA imaging- high risk features

A

High-risk imaging findings:
Findings of high early stroke risk require rapid assessment and early secondary prevention measures.
High risk imaging findings include:
Distal vertebral and proximal basilar artery lesion -

Posterior circulation (vertebral, basilar, and posterior cerebral artery) ischemia is underdiagnosed because symptoms are often atypical (e.g., dizziness, ataxia, diplopia); however, the risk of early stroke is high and early revascularization may be indicated to prevent complications (e.g., locked-in syndrome).

Distal basilar artery and middle cerebral artery lesion - May portend an MCA stroke
Severe carotid artery stenosis - Lesions with ≥ 50% stenosis may benefit from early revascularization
Cerebral or cervical arterial dissection (see dissection of the carotid and the vertebral artery)- In young adults, dissection is an important cause of TIA to consider. Symptoms of arterial dissection may include head or neck pain, and the patient may benefit from endovascular repair

How well did you know this?
1
Not at all
2
3
4
5
Perfectly
84
Q

Shigella and Salmonella

A

Shigella and shigella toxin- bloody diarrhea. Only instance of dysentery where prescribe ABs to shorten duration- Floroquinalones - ciprofloxin or third gen cephalosporin.

Salmonella- thyphi and non thyphi. .

Salmonella Thyhi/para typhi– Typhoid/para typhoid fever: Constipated then diarrhea- possibly , Osteomelitis in SCD.
NO ABs- onli insever cases (floroquins)

Thyphoid/Paratyphoid fever
A systemic illness caused by by Salmonella enterica serotype Typhi/ Salmonella enterica serotypes Paratyphi A, B, and C.
Manifestations include fever, chills, abdominal pain, rose spots on the trunk and abdomen, and hepatosplenomegaly.

Salmonella non typhi (ie S typhrium)- Salmenellosis, diarrhea, possibly bloody. Reactive arthritis,
No ABS unless severe.

Salmonellosis-Salmonella gastroenteritis

A bacterial infection caused by Salmonella enteritidis and nontyphoidal salmonella. The second-most common cause of bacterial foodborne gastroenteritis. Associated with watery-bloody diarrhea, fever, chills, headache, myalgia, and severe vomiting.

How well did you know this?
1
Not at all
2
3
4
5
Perfectly
85
Q

Patient presents with abdominal pain, fever and positive S. non typhi stool culture. Then worsening abdominal pain.

what Complication of non-thyphoidal Salmonella looking for. What investigation.

A

Non-typhoidal Salmonella bacteraemia may occur as a result of intestinal salmonella infection.

A number of host factors predispose to salmonella bacteraemia including extremes of age, immunosuppression, chronic liver disease and haemoglobinopathies, as well alteration of the gastrointestinal tract by use of antibiotics, malnutrition or acid suppression.

Salmonella bacteraemia may result in a number of complications affecting a wide variety of sites including the urinary tract, lungs, pleura, heart as well as bones and joints.

An uncommon but important complication of salmonella bacteraemia is endovascular infection leading to a mycotic (or infected) aneurysm. Endovascular infections have been noted to occur in 10-20% of patients > 50 years old with Salmonella bacteraemia.

The classic presentation of an infected aneurysm is a painful, pulsatile, and enlarging mass with systemic features of infection. Salmonella typically targets the aorta, particularly if already diseased with atherosclerosis. The best diagnostic investigation for a suspected mycotic aneurysm is CT angiogram (MR angiography is an alternative if intravenous contrast is contraindicated).

A urogram may be a useful investigation if urinary tract colonisation was suspected.
A chest X-ray may be performed if there were respiratory symptom.
A CT brain and lumbar puncture may be useful if meningoencephalitis was suspected.

Learning Points
Aortitis is a rare but important complication of Salmonella bacteraemia

How well did you know this?
1
Not at all
2
3
4
5
Perfectly
86
Q

Hiatal hernias

six month history of heartburn and regurgitation. He is a non-smoker, walks 2km every day and goes to the gym regularly. He has not had any major illnesses in the past and does not use any medications. Physical examination is unremarkable. A contrast study is performed (see image).

appropriate plan of management?
Choose the single best answer.

  • Esomeprazole
  • Nifedipine
  • Oesophageal dilatation
  • Laparoscopic cardiomyotomy
  • Laparoscopic fundoplication
A

Often linked to GORD- dont need s

The contrast study shows a small hiatus hernia with volume reflux, consistent with gastro-oesophageal reflux disease (GORD). There is no evidence of stricture formation. The most appropriate initial management for this patient is acid suppression with a proton pump inhibitor. The large majority of patients with GORD get good control of their symptoms with a proton pump inhibitor.

Calcium channel blockers (e.g. nifedipine) and laproscopic cardiomyotomy are management options for achalasia (which would be evident on barium swallow as a tapering of the inferior oesophagus).

Oesophageal dilation is only needed if stricture is present. The patient does not have any dysphagia, so there is no clinical nor radiological evidence of stricture formation. With the advent of effective acid suppression (proton pump inhibitors), stricture formation is now a relatively uncommon complication of gastro-oesophageal reflux disease.

Fundoplication is last line therapy for uncontrolled GORD and considering no treatment has been trialled so far, this approach would be inappropriate.

How well did you know this?
1
Not at all
2
3
4
5
Perfectly
87
Q

Patient with hematemesis. With strong Alchol use and child Pugh B cirrhosis. He is noted to have jaundice, splenomegaly and multiple spider naevi. Apart from a proton pump inhibitor and ceftriaxone, which one of the following medications is most strongly indicated? DOC in acute esophageal varices rupture. Choose the single best answer. Octreotide Somatostatin Propranolol Terlipressin Tranexamic acid

A

The correct answer to this question is terlipressin. This patient has a history of cirrhosis (likely secondary to chronic alcohol misuse) and presents with symptoms and signs suggestive of a massive variceal haemorrhage. He has not seen a gastroenterologist in over 3 years, so it is unlikely that he has had any recent screening endoscopies. Oesophageal varices arise due to portal hypertension, which leads to the development of porto-systemic collaterals. Rupture of these varices can result in severe and sudden blood loss, often proving fatal. In this case, the initial priority is to stabilise the patient, especially given his current haemodynamic compromise. This is achieved through the careful administration of crystalloid fluids (and blood products if required), as well as terlipressin, which helps to decompress the portal circulation and reduce blood flow through the porto-systemic collaterals. Therefore terlipressin is the correct answer. An IV antibiotic such as ceftriaxone and an IV PPI bolus are also indicated in variceal bleeding events. Octreotide and somatostatin are medications that can also be used to decompress the portal circulation, however they are both second-line behind terlipressin. Therefore, octreotide and somatostatin are incorrect. Propranolol is a non-cardioselective beta blocker, which is used longer-term to help prevent the development of oesophageal varices. As such, it does not have a major role in the acute management of variceal haemorrhage, making this answer incorrect. Tranexamic acid was previously thought to have had a role as an adjuct in the management of acute variceal bleeding, however a recent multi-centre international randomised placebo-controlled trial (HALT-IT) found no reduction in mortality from the administration of tranexamic acid in upper GI bleeding. As such, tranexamic acid is an incorrect answer.

How well did you know this?
1
Not at all
2
3
4
5
Perfectly
88
Q

Child B means ..l what criteria look at fro child score

A
How well did you know this?
1
Not at all
2
3
4
5
Perfectly
89
Q

haematemesis, where oesophageal varices were found to be the source of bleeding. Variceal banding was carried out, his condition stabilised and the patient is now being prepared for discharge home. Beyond the variceal banding, other measures will be required as part of secondary prophylaxis. Which one of the following management options would be most appropriate? Choose the single best answer. Metoprolol ACE inhibitor Propanolol Band ligation Terlipressin

A

Non-selective beta-blockers are the current drug of choice for primary prevention of variceal bleeding. Propanalol While metoprolol is also a beta-blocker, it is cardio-selective (B1 specific) and so does not confer benefit for prevention of variceal haemorrhage. Band ligation can be considered in patients where beta-blockers are contraindicated, or in those with imminent risk of rupture at endoscopy. Terlipressin is a vasopressor analogue that has a role in the management of variceal haemorrhage, however this is usually limited to after the bleeding has occurred and not as primary prophylaxis.

How well did you know this?
1
Not at all
2
3
4
5
Perfectly
90
Q

An 80-year-old man is brought to his General Practitioner by his daughter, who is concerned about functional decline over the past several months. She describes periods of confusion and disorientation interspersed with days where he is lucid and well-oriented. He has mentioned seeing his young grandchildren playing in his home, but they live interstate. He is experiencing vivid nightmares. He has had multiple falls in the past year. He has been incontinent of urine due to inability to reach the toilet in time. Past medical history includes hypertension, hyperlipidaemia, and type 2 diabetes. On examination, he is alert, fluent in speech, and oriented to place and person but not time. His vital signs are normal. He has no tremor. Tone is mildly increased in all limbs throughout the range of motion. Reflexes are normal and his plantar response is downgoing. His gait is slow with short, shuffling strides and a narrow base. He scores 21/30 on a Mini Mental State Examination and is especially poor in short-term memory. His Frontal Assessment Battery is 14/18. Which one of the following is the most likely diagnosis? Choose the single best answer. Vascular dementia Normal pressure hydrocephalus Creutzfeldt-Jakob disease Lewy body dementia Corticobasal degeneration

A

This patient presents with dementia, recurrent visual hallucinations, parkinsonism (bradykinesia, limb rigidity, and typical gait disturbance), and evidence of REM sleep behavioural disorder; this constellation of features is most consistent with Lewy body dementia (LBD). LBD is characterised by fluctuating cognitive deficits, persistent detailed visual hallucinations, parkinsonian features, and REM sleep behavioural phenomena. A highly specific feature of the condition is extreme sensitivity to anti-psychotic medication, which might otherwise be considered to assist with managing problematic behaviour in dementia patients, but in LBD patients can precipitate severe extra-pyramidal symptoms or even neuroleptic malignant syndrome. LBD is a neurodegenerative disease whose pathological hallmark is the accumulation of alpha-synuclein - a feature it shares with Parkinson’s disease, hence the significant overlap in their clinical manifestations, especially as the disease becomes progressive. Diagnosis is predicated on eliciting the core features described above. Supportive radiological indicators have been identified on MRI and SPECT/CT scans, however these are not in broad clinical use and are not required for diagnosis. Normal pressure hydrocephalus, though notably associated with cognitive impairment, urinary incontinence, and gait disturbance, does not feature the same fluctuations, visual hallucinations, REM sleep behaviours, nor parkinsonian features, and the gait is typically wide-based rather than narrow. Creutzfeld-Jakob disease is a rapidly progressive prion disease with prominent dementia, myoclonus, and cerebellar manifestations; spasticity (i.e. velocity-dependent increase in tone that is unequal between antagonistic muscle groups) is much more common than rigidity; average survival is 6 months. Corticobasal degeneration typically onsets with prominent asymmetric motor features, often starting in a single limb. Vascular dementia is associated with a series of step-wise deteriorations in neurocognitive function rather than fluctuations that return to near-baseline, and often features associated focal neurological deficits.

91
Q

A 75 year old woman is brought into the emergency department via ambulance after suffering a fall at home. According to paramedics, she was found by her daughter on the bathroom floor, shivering and confused. Her past medical history includes hypertension, hypothyroidism, eczema, rheumatoid arthritis and a recent pulmonary embolism. Her medications include amlodipine, levothyroxine, methotrexate, hydroxychloroquine and apixaban, as well as topical betamethasone valerate. On examination, her heart rate is 52/min and irregular, her blood pressure is 134/88 and her temperature is 35.8 degrees. She is disorientated to person and place and is drowsy. There is a 3cm laceration on her forehead and small haematoma. On auscultation, there is an ejection systolic murmur radiating to the carotids. After rolling the patient, extensive bruising of the back and buttock is noted. The rest of the examination is normal. Which of the following is the most important initial investigation? Choose the single best answer. Electrocardiogram Postural blood pressure assessment Thyroid function tests Non-contrast CT head Urine dipstick

A

Falls in the elderly are a common presentation to emergency departments. Despite the high frequency of falls in the geriatric patient cohort, all should be properly assessed and investigated, as falling is NOT a normal part of ageing. The correct answer to this question is non-contrast CT head. This patient has sustained a fall and at the present time, the exact aetiology is somewhat unclear. It is very likely that the fall is multifactorial and thus, further investigation of potential causes will be required. Despite that, the patient has sustained a head injury, evidenced by the laceration and haematoma formation on her forehead. As she is currently taking an anticoagulant (apixaban), a non-contrast CT is indicated, as she may have suffered an intracranial bleed. In this case, she is drowsy and confused, so ruling out a haemorrhage should be done with an even greater sense of urgency. An electrocardiogram is an important investigation to perform, as cardiac arrhythmias and structural abnormalities (e.g. aortic stenosis) are a common cause of falls in the elderly. This patient is currently bradycardic, with an irregular heart rhythm and evidence of aortic stenosis, so an ECG should most certainly be done. Postural blood pressure assessment can be a simple and useful tool for detecting orthostatic hypotension and volume depleted states. These are both associated with falls. This patient has hypothyroidism and is currently bradycardic, hypothermic, confused and drowsy. To rule out a myxoedema coma, thyroid function tests should be performed. In the setting of normal thyroid function tests, a myxoedema coma is unlikely. Urinary tract infections are common in the elderly population and are more likely to present with non-specific symptoms, such as delirium and falls. A urine dipstick is an easy investigation to perform and can help guide the need for formal urine MCS, as well as empirical antibiotics. Whilst all of the investigations mentioned above are important, this question asks for the most important investigation. An intracranial haemorrhage can be fatal and in this patient, there are risk factors and physical examination signs that are suspicious for one. As such, a non-contrast CT is the most pressing investigation to perform.

92
Q

Hep B

Hepatitis antigens- resolved infection?

A
  • ANTIBODYs to each COMPONENT Surface, Capsule (igG) and Envelope
  • HB s AB → positive
  • HBe AB→ positive
  • HBc AB→ IgG positive

Resolved prior infection

  • HBsAg undetectable
  • Anti-HBs ↑
  • HBeAg undetectable
  • Anti-HBe ↑
  • Anti-HBc ↑ IgG
  • HBV DNA undetectable
  • Transaminases undetectable
93
Q

HEP B

Hep B vaccination

A

ONLY ONE:

Hep B surface antigen

94
Q

Acute hepB serology

Acute Hep B infection

A

antigens of each component E, S, C (IgM) possible viral DNA

HBsAg, HBeAg,HBcAg (IgM)

NB IgM indicates acute nature. IgG can can only indicate past acute phase.

Difference with resolved infection→ each antigen becomes component antibody

If surface antibody present - means 3 things acute or chronic (acitve or chronic)

95
Q

Chronic Hep B infection

A

Antibodies:
Active Chronic - HBsAB*: negative HBeAB: undetect HBcAB; IgG raised/IgM

Chronic carrier: HBsAB: neg HBeAB**: positive HBcAB IgG

Antigen:

Active Chronic hep: HBsAG*** positive, HBeAG: positive HBcAG: positive , HB DNA >2000 viral copies

Chronic carrier state: HBsAG***: positive, HBeAG: undetectable , HBcAG: negative; HB DNA less <2000

* Increased anti-HBs antibodies in an individual with hepatitis B infection indicates elimination of the virus and recovery from infection. The presence of anti-HBs antibodies in a healthy individual indicates immunity against the hepatitis B virus either as a result of a past infection or as a result of vaccination.

** HBeAB indicated that active viral replication is low/not occuring

*** HbsAg is the first marker to be elevated in a patient with hepatitis B infection. Higher levels of HbsAg also correlate with increased infectivity. The persistence of HBsAg in serum for more than 6 months indicates chronic hepatitis B infection

96
Q

Hep B

Window period before seroconversion of Hep B

A

During the window period, anti-HBc IgM and anti-HBe may be the only markers available to diagnose an acute HBV infection.

97
Q

hep B

Testing algorithm

A

Screening: measure HBsAg and anti‑HBc IgM
If HBsAg is positive: measure HBeAg and HBV DNA to determine transmissibility

98
Q

Sudden onset of nausea, jaundice, fever, and right upper quadrant pain with massively elevated serum transaminase levels (> 1000 U/L) in a patient with a history of injecting drugs suggests..

Serology:

HBcAg IgM: positive

HBsAB: negative

A

Acute hep B infection. HBsAB is positive when virus is cleared.

99
Q

case of community-acquired pneumonia in a young person. The unremarkable chest examination, normal white cell count and absence of consolidation on chest X-ray suggests ?

A

atypical pneumonia.

Atypical pneumonia refers to an “atypical presentation” - these pneumonias lack the classic productive cough and pleuritic chest pain and fever triad, and tend to have more prominent non-respiratory symptoms including ear pain, coryza symptoms, and nausea and vomiting.

It is important to note atypical does NOT mean “uncommon” - atypical pneumonia presentations are common and the organisms that cause them are common!

Common pathogens in atypical pneumonia include Mycoplasma pneumoniae, Chlamydia spp., Coxiella burnetti, Legionella, and viruses such as influenza A and B, and Respiratory Syncytial Virus (RSV).

Of these, Mycoplasma pneumoniae is the most common pathogen in atypical pneumonia (A is correct). L

egionella longbeachae is another atypical pathogen that may be suspected with a history of exposure to soil or potting mix.

Streptococcus pneumoniae and Haemophilus influenzae are both common pathogens in typical/classical pneumonia.

100
Q

when should back pain be investigated. ie X rays

A
  1. Red flags for back pain: anything that can be linked to a tumour/infection/an immune compromised individual/ cauda equina
  2. Indications for imaging include patients with red flags, abnormal neurology or pain lasting >4 weeks.

shifted away from ordering initial imaging to investigate acute lower back pain of < 4 weeks in duration without red flags as most cases of lower back pain are mechanical and improve with time
very important to triage for red flags in the assessment of lower back pain and determine need for imaging.Non redicular pain

Tumour

  • History of cancer
  • Weight loss
  • Night pain
  • Age >40 or <15 years

Fracture

  • History of trauma
  • Risk factors for fragility fracture: long term steroid use, existing osteoporosis.

Infection

  • Fever >38.0°C
  • Night sweats/chills
  • Immunosuppression- steroids
  • Intravenous drugs
  • Concomitant infection

Cauda equina syndrome

  • Urinary retention
  • Saddle anaesthesia
  • Worsening neurologyflaccid paralysis

Pain profile distinguishes between central and radicular pain.

Transverse LBP radiating to sacroiliac joints can originate from the intervertebral disc.

Pain radiating to the buttocks or posterior thigh can originate from facet joints.

Radicular leg pain can be described on the basis of dermatomal distribution.

It is important to consider non-spinal pathology, such as hip arthritis and intra-abdominal pathology.

Non-organic causes are suggested in patients with non-anatomic findings, superficial tenderness, pain with axial loading and non-sustained power.

101
Q

OA vs RA

A

RA does not affect the DIP!!

osteoarthritis. Osteoarthritis tends to affect elderly patients, has a gradual onset of joint pain and joint stiffness, and affects the distal interphalangeal joints (Heberden nodes) and proximal interphalangeal joints (Bouchard nodes).

Rheumatoid arthritis is unlikely to occur in a 70-year-old. It can be a symmetrical polyarthritis affecting the small joints of the hand, however it spares the distal interphalangeal joints.
features of active synovitis in this , rheumatoid arthritis does present with a gradual onset of joint pain and joint stiffness, and is more common in those with a personal or family history of autoimmunity.

102
Q

complication of diverticulitis.

A

Para-coloic abscess. Look for gas on CT

103
Q

Asymptomatic and incidental paraoesophageal hernia

A

Most hiatus hernias - even large para-oesophageal hernias - do not cause significant symptoms. If the patient had reflux symptoms, an endoscopy might be appropriate. Symptoms of early satiety or obstruction - which could be due to a large para-oesophageal hernia - might also lead to an endoscopy. If further investigations are indicated, large para-oesophageal hernias are better defined with a contrast study. Esomeprazole is not indicated in an asymptomatic patient and carries additional risk of infection and other side effects. In this case, the patient does not need any further investigation or management of the hernia, which is almost certainly an incidental finding and be ignored. Obviously the CT findings should be discussed with the patient and reassurance given that this incidental finding is of no clinical significance.

Learning Points
Most para-oesophageal hernias do not produce symptoms of any consequence.

104
Q
A
105
Q

When is unsynchronized cardioversion Performed

A

Nonsynchronized electrical cardioversion is indicated for patients with polymorphic ventricular tachycardia that present with features of hemodynamic instability, pulseless ventricular tachycardia, or ventricular fibrillation.

106
Q

Contraindications to adenosine

When is adenosine used? Narrow complex tachycardia (NCT) Regular

A

A set of contraindications that includes

  • pre-existing second-degree AV block,
  • complete heart block,
  • sick sinus syndrome in the absence of a pacemaker,
  • concurrent verapamil or digoxin therapy,
  • reactive airway disease, and
  • Wolff-Parkinson White syndrome with either atrial fibrillation or antidromic atrioventricular reentrant tachycardia.
107
Q

Atrial flutter. Narrow or broad complex tachy?

A

Atrial flutter with conduction (fixed): narrow complex regular tachy.

If A.Flut with block (regular wide complex)

Atrial flutter with 4:1 atrioventricular (AV) block and bifascicular block

12-lead ECG (paper speed: 25mm/s)

  • Regular rhythm
  • Atrial rate of ∼ 320/min and ventricular rate of ∼ 80/min
  • Sawtooth P wave morphology (flutter/F waves) best seen in II, III, aVF
  • Left axis deviation (LAD)
  • qR pattern in lead aVL, rS pattern in II/III/aVF (in combination with LAD, this indicates left anterior fascicular block (LAFB)
  • Broad QRS complexes (∼ 120ms) with RBBB morphology (in combination with LAFB, this indicates bifascicular block)
108
Q

complications post MI

Must access- time since MI. Certain complications are more likely.

Clinical features of complicationsfirst

A
  • Risk of re-infarction- constant but increases with time.
  • Arrhythmia and shock constant too

first 24 hours-

  • AV dysfunction (AV block), brady and arrhythmia (VT), cardiogenic shock, heart failure.

1-3 days- pericarditis

  • first week to 2 weeks- papillary muscle rupture (acute mitral valve regurge)
  • ventricular wall rupture ( holosystolic murmur, with features of heart failure)
  • left ventricular free wall rupture-(cardiac tamponade- look for becks triad muffled heart sounds/hypotension/raised JVP)
  • LV psuedo anuerysm

late 2 weeks to months-

  • Mural wall thrombus
  • Desslers Syndrome ( autoimmune mediated pericarditis)
109
Q

Management of choice of polymorphic Ventricular Tachy (torsades) in a hemodynamically stable person?

Mx of VT polymorphc in Heamodynamically unstable-

Especially possible post using a macrolide (Azithro/Ezythro) for an atypical pnuemonia treatement.

A

Heamodynamically stable:

  • Magnesium Sulphate DOC
  • (whetehr high or low magnesium levels)

Immediate electrical cardioversion

  • Emergent electrical cardioversion is the first-line treatment for TdP in hemodynamically unstable patients.

if conscious and hemodynamically stable, MgSO4 treatment more appropriate.

110
Q

K and Na levels

A

normal K levels 3.5 to 5

normal Na levels- 135-145

111
Q

A 75-year-old man becomes pulseless in the cardiac intensive care unit 48 hours after admission for ST-elevation myocardial infarction. His blood pressure is 50/20 mm Hg. The ECG monitor shows a tachycardic irregular rhythm with erratic undulations, but no discernible P waves or QRS complexes. During immediate advanced cardiac life support, two attempts at defibrillation are made, after which the patient is given 1 mg of IV ADRENALINE. After a third attempt at defibrillation, the patient remains pulseless and his blood pressure is 60/35 mm Hg. The ECG monitor shows no changes. Administration of which of the following is the most appropriate next step in management?

A

Despite two attempts at defibrillation and administration of epinephrine, the ECG shows an arrhythmic, fibrillatory baseline with no P waves or QRS complexes. This finding is consistent with refractory ventricular fibrillation (VF).

VT is managed with defibrillation (unsyncronysed cardioversion), performed twice.

Then one dose of adrenalin 1ml 1;1000

3rd defib

then amioderone ( av nodal blocking agent- alternative: lidocaine)

112
Q

Seventy-two hours after admission for an acute myocardial infarction, a 48-year-old man develops dyspnea and a productive cough with frothy sputum. Physical examination shows coarse crackles in both lungs and a blowing, holosystolic murmur heard best at the apex. ECG shows Q waves in the anteroseptal leads.

Pulmonary capillary wedge pressure is 23 mm Hg. Which of the following is the most likely cause of this patient’s current condition?

A

This patient’s new murmur (blowing, holosystolic murmur best heard at the apex) and evidence of heart failure (dyspnea, bilateral crackles, elevated capillary wedge pressure- the signs of isolated left heart failure (e.g., pulmonary edema, elevated PCWP) suggest acute mitral regurgitation.

Pulmonary capillary wedge pressure/Pulmonary artery occlusion pressure
An indirect measure of left atrial pressure and, therefore, an estimation for preload. Measured by advancing a balloon catheter into a small pulmonary artery and then inflating the catheter. The PCWP is then measured distal to the inflated balloon. Normal: 4–12 mm Hg

alt DDX in this patient

Rupture of the interventricular septum

Rupture of the interventricular septum may occur within the first 2 weeks after MI and typically presents with sudden hemodynamic instability (hypotension, tachycardia), dyspnea, and new-onset holosystolic murmur (lSB).
However, because blood is shunted from left to right, patients typically present with signs of right heart failure (e.g., increased JVP, peripheral edema) rather than the signs of isolated left heart failure (e.g., pulmonary edema, elevated PCWP) seen here. Moreover, the expected murmur would be heard best over the left sternal border, not the apex.

Rupture of the ventricular free wall

Ventricular wall rupture can occur within 2 weeks of myocardial infarction and typically results in hemopericardium and cardiac tamponade, which can manifest with profound right heart failure, shock, and often death. While free wall rupture and cardiac tamponade can also cause acute pulmonary edema, additional expected findings would include pulsus paradoxus and muffled heart sounds, none of which is observed in this patient.

  • Pulsus paradoxus*
  • A physical examination finding in which there is a pathologic decrease (> 10 mm Hg) in systolic blood pressure during inspiration. Classically associated with cardiac tamponade and constrictive pericarditis, but can also be seen in noncardiac conditions (e.g., massive pulmonary embolism, hemorrhagic shock, obstructive sleep apnea, obstructive lung disease)*
113
Q

distinguish between baby blues and perinatal depression

A

Perinatal depression is diagnosed according to the DSM-5 criteria

The DSM-5 criteria for perinatal depression:

A. 5 or more of the following symptoms that have been present for 2-weeks consistently (every or almost every day) and are a change from previous functioning. (One of these symptoms must be either Depressed mood or Loss of interest/pleasure).

Mood Interest W I P ED E g C

  • Depressed mood
  • Loss of interest/pleasure
  • Weight loss or weight gain or appetite changes
  • Insomnia or hypersomnia
  • Psychomotor agitation or retardation
  • Fatigue or loss of energy
  • Feelings or worthlessness or excessive guilt
  • Problems with concentrate or make decisions
  • Recurrent thoughts of death

B. These symptoms cause an impairment in functioning for the individual or clinically significant distress

C. These symptoms cannot be better explained by another medical condition or by other psychiatric diagnosis

Women experiencing baby blues may have some of these symptoms and it is extremely common, affecting up to 80% of new mums. Baby blues usually develops in the first few days after birth, classically day 3 and resolves <10 days.

114
Q

A 32-year-old man presents with intermittent abdominal pain and foul-smelling diarrhoea for the past 9 months. He recently developed foul-smelling anal discharge. He does not take any medications. He has been smoking 1 pack of cigarettes daily for the last 12 years. Temperature is 37.3 C, blood pressure is 130/85 mm Hg, and pulse is 89/min. Abdominal examination reveals right lower quadrant tenderness, but no guarding. Perianal examination reveals a posterior skin tag and a fistula draining foul-smelling purulent discharge. Oral examination reveals several shallow ulcers.

This patient is particularly at an increased risk of developing which type of renal stones?
Choose the single best answer.

Calcium oxalate stones
Calcium phosphate stones
Cystine stones
Magnesium ammonium phosphate stones
Uric acid stones

A

This patient with chronic non-bloody diarrhoea, anal fistula, and oral ulcers most likely has Crohn’s disease (CD). CD is an inflammatory disorder of the bowel that presents with non-bloody diarrhoea, malabsorption (eg, steatorrhoea), abdominal pain (most commonly in the RLQ), oral ulcers, and perianal fistulas. Smoking increases the risk of CD. A few patients may also have constitutional symptoms (eg, low-grade fever, weight loss). Patients with CD are particularly at an increased risk of developing calcium oxalate stones. This is because of fat malabsorption associated with CD. Normally, dietary calcium binds to the oxalate in the gut, thereby decreasing excess absorption of oxalate. In conditions that cause fat malabsorption (eg, CD), calcium binds to the fat, leaving oxalate unbound. Unbound oxalate is freely absorbed in the bloodstream. This leads to elevated oxalate levels in the blood which form calcium oxalate stones in the urine.

Calcium phosphate stones are associated with hyperparathyroidism and type 1 renal tubular acidosis. Cystine stones are usually seen in hereditary cystinuria. Magnesium ammonium phosphate stones are struvite stones usually seen in urinary tract infections with urease-producing bacteria (eg, Proteus). Uric acid stones are seen in gout and conditions with high cell turnover (eg, leukaemia, polycythaemia, chemotherapy).

Learning Points
Patients with Crohn’s disease have an increased risk of developing calcium oxalate renal tract stones.

115
Q

opthalmology- Mac degen

80 year old. Gradual loss of vision.

Which one of the following is the most likely diagnosis? See image

Choose the single best answer.

Central retinal artery occlusion
Optic atrophy
Optic neuritis
Macular degeneration
Diabetic retinopathy

A

Fundoscopy shows the presence of drusen which are small yellow deposits under the retina. This is consistent with Age related Macular degeneration. Further evaluation may involve the use of the Amsler grid.

Central retinal artery occlusion will present with sudden onset of painless vision and fundus examination will show retinal pallor and a ‘cherry red’ spot at the macula. Optic atrophy refers to death of the retinal ganglion cells and fundus exam will show a pale optic disc. Optic atrophy and chronic papilloedema may be secondary to any condition causing stress on retinal ganglion cells including glaucoma, optic neuritis, an papilloedema. Optic neuritis will present with painful blurring of vision associated on examination with a relative afferent pupillary defect, abnormal colour vision and a swollen optic disc. Diabetic retinopathy can also cause bilateral, subacute vision changes. Fundus examination will however show evidence of microaneurysms, hard exudates, cotton wool spots and potential neovascularisation.

Learning Points
A history of subacute vision changes and yellow Drusen under the retina supports a diagnosis of age-related macular degeneration

116
Q

Opthal

How to tell the difference between white/yello spots on fundoscopy ie- Cotton wool spots, hard and soft drusen?

A

Cotton wool spots
An ophthalmologic finding characterized by yellow-white deposits on the retina. Caused by swelling of retinal nerve fibers (e.g., due to ischemia as a result of diabetes and/or hypertension).

Spots around the retinal vessels -

Since these spots do not obscure the retinal vessels, the pathology must be below the nerve fiber layer (i.e., either a hard exudate or a drusen but not a cotton wool spot).

117
Q

opthal

what does this show

A

Proliferative diabetic retinopathy

Fundoscopic image of the right eye

Pathological findings:
– Neovascularization: networks of tortuous vessels originating from the normal retinal vasculature (example indicated by arrowhead)
– Microaneurysms and retinal hemorrhages: small red dots and larger red blotches (examples indicated by blue overlay)
– Hard exudates: yellow flecks (examples indicated by green overlay)
– Venous beading: periodic dilatations of retinal veins (example indicated by yellow overlay)

118
Q

increasing blurring of vision in both eyes.

blurring has made it difficult to read, although she has noticed that she can read a little better when holding the book below or above eye level.

She also requires a bright light to see objects more clearly.
began 8 years ago and have gradually gotten worse over time.
has hypertension and type 2 diabetes mellitus.
Current medications include glyburide and lisinopril. W
The results of a funduscopic examination are shown. Which of the following is the most likely diagnosis?

A

Image: Dry age-related macular degeneration- Fundoscopic image of the eye

  • There are drusen (yellow deposits in the large circled area) and areas with retinal pigment epithelium atrophy (shaded area).
  • P: papilla; M: macula

When looking at an Amsler grid, she says that the lines in the center appear wavy and bent.

The patient’s blurring of central vision and Amsler grid findings indicate central metamorphopsia. The fundoscopy showing multiple yellow areas at the macula confirms the diagnosis. Since these spots do not obscure the retinal vessels, the pathology must be below the nerve fiber layer (i.e., either a hard exudate or a druse but not a cotton wool spot). The clustering of spots around the macula, the absence of sharp margins, and the absence of other retinal pathology indicate that these spots are soft drusen rather than hard exudates.

The presence of soft drusen near the macula is consistent with age-related macular degeneration (AMD). AMD is the most common cause of impaired vision in the elderly in developed countries. Cardiovascular diseases such as hypertension increase the risk of AMD. Nonexudative AMD (dry AMD) is the most common type of AMD and leads to a slowly progressive central or pericentral loss of vision. Exudative AMD (wet AMD) makes up 10% of cases and is associated with retinal neovascularization, retinal hemorrhage, and/or loss of vision that progresses more rapidly.

119
Q

Cows milk (protein) allergy

A

HOW IS COWS MILK ALLERGY DIFFERENT TO LACTOSE INTOLERANCE? Lactose intolerance is when the body has difficulty digesting lactose, the natural SUGAR found in milk. Cows milk allergy is when the immune system reacts to the PROTEIN in milk. Lactose free products still contain cows milk protein and are not suitable for people with cows milk allergy.

120
Q

Mucocutaneous bleeds- nose, periods (menorrahgia), easy bruising Minor procedural heavy bleeding

A

Factor 5 Leiden deficiency - platelet aggregation and coupling of factor 8 Autosomal dominant, most common bleeding d/o Mx - antifibrinolytics (tranexamic acid) , Desmopressin (DPPAV), F5/F8 concentrate 1st choice mild disease- antifibrinolytics , used to decrease bleeding and adjunct to DPPV 2- impacting QOL- DPPAV 3- F5F8 prior to procedures Von Willebrands disease is the most common inherited bleeding disorder occurring in 1-2% of the general population and clinically significant in a small proportion of this. It is inherited in an autosomal dominant factor, but has variable clinical effects and penetrance. The main pathology is decrease or ineffective Von willebrand factor (VWF) which normally facilitates platelet adhesion as well as acts as a carrier for factor 8 and prevents its breakdown in the serum. Common presentations features includes mucocutaneous bleeding (epistaxis, menorrhagia, easy bruising) and increased post-procedural bleeding. For this patient this is the most likely diagnosis and there are various tests to diagnose this. There are three possible management arms: antifibrinolytic (tranexamic acid), desmopressin or factor VIII/VWF concentrates. Antifibrinolytics are useful for mild disease to decrease bleeding and can be used as an adjuvant to the two other therapies. Desmopressin triggers the release of stored endothelial VWF to increase serum concentrations. Importantly patients need a monitored desmopressin challenge as some patients do not respond and it is generally most effective for type 1 VWD. Finally, Factor VIII/WVF concentrate can be given but usually this is if desmopressin has failed and/or there is significant bleeding risk (pre-op for major surgery, previous significant bleeding such as GIT). Therefore, this patient will most likely benefit from desmopressin than the other agents. In menorrhagia alone and in VWD, the COCP has good evidence for symptom management but would not be first line or sole treatment. Learning Points VWD is the most common inherited bleeding disorder

121
Q

Hemophilia A and Hemophilia B .. what factors are deficient Heamophilla A has a management similar to which other blood d/o

A

A F8 , B F9 “A fate , B nine” Heamophilla A is a deficiency of F8 Heamophilla B is a deficiency of F9 F8 is carried on F5 Leiden, and F5 prevents its breakdown. So Heamophilla A is managed with same drugs as F5 Leiden deficiency.

122
Q

Australia from the United Kingdom 6 months ago. He has not connected with a haematology service since moving but reports that there were no concerns “back home” and that he was considered to have mild and well-controlled haemophilia A. Upon physical examination, he is assessed as being well with observations within normal limits. A focussed examination of the left knee demonstrates a mild bleed into the joint (haemarthrosis). He has been given analgesia, ice packs have been applied and the limb is immobilised. Which one of the following treatments should he receive to treat this bleeding? Choose the single best answer. DDAVP Factor VIII Prothrombinex Tranexamic Acid Factor IX

A

The correct answer is ‘Factor VIII’. In Haemophila A patient experiencing a mild bleed including mild haemarthrosis there are generally two appropriate treatments. One is administering Factor VIII, dosing for the patient’s weight and the desired increase in Factor VIII level. The other is to administer DDAVP (Desmopressin). DDAVP is not the most effective treatment option for this patient because the stem does not indicate whether he has received a testing dose of DDAVP previously. (Also it is unlikely that the Australian health system would have any documentation of a DDAVP test from the UK) Although DDAVP is appropriate for mild bleeds it is preferred that patients with haemophilia A have a documented response to a test dose of DDAVP under routine comprehensive care. Without this documentation, it is generally safer to give Factor VIII. Prothrombinex is not an appropriate treatment for patients with haemophilia A unless they are known inhibitors. These are patients who have antibodies that bind and render the administered Factor VIII ineffective. Tranexamic Acid (TXA) and other antifibrinolytic therapies are appropriate for mild bleeds in some patients with haemophilia A. However, this treatment is only recommended for mucosal bleeding, not haemarthroses. Factor IX is used for the treatment of bleeds in patients with haemophilia B not A.

123
Q

Inheritance of each Heamophilla. And factor deficiency

A

Hemophilia is caused by an X-linked recessive defect (inherited or spontaneous mutation) or antibody production against clotting factors. Hemophilia A (factor VIII deficiency): ∼ 80% of cases Hemophilia B (factor IX deficiency): ∼ 20% of cases Hemophilia C (factor XI deficiency): very rare (increased frequency in the Ashkenazi Jewish population); caused by an autosomal recessive defect Hemophilia usually affects males, as it is primarily an X-linked recessive disease.

124
Q

How diagnose Heamophilla .. how is severity determined. Does Heamophilla present with petechiae. Which Heamophilla is unlikely to present with deep tissue bleeding.

A

Diagnostics Patient and family history Genetic testing Screening Prothrombin time: normal Platelet count: normal Activated partial thromboplastin time (aPTT): usually prolonged If aPTT prolonged → mixing study If mixing study is positive (or if patient/family history are strongly positive) → quantitative assessment of factor activity levels Differential diagnosis: See ”Diagnostics” in “Hemostasis and bleeding disorders.”

125
Q

petechia vs bruising

A

Petechia-platelets Bruising- factors of coagulation + liver

126
Q

Reactive arthritis vs ankolysing spondylitis.. key clinical distinctions

A

ReA reactive arthritis (ReA), especially in light of the concurrence of features of arthritis, conjunctivitis, and urethritis (a triad formerly called Reiter’s syndrome) – hence the mnemonic “can’t see, can’t pee and can’t climb a tree”. ReA is the most common inflammatory polyarthritis in young men, and typically follows genitourinary or enteric infection, such as traveller’s diarrhoea. In incorrectly answered question- traveler’s diarrhea. Onset can be acute or insidious and usually occurs 1-6 weeks post-infection. People with the HLA-B27 gene are much more likely to develop ReA. AS- eye: uveitis ( often unilateral) - not conjunctivitis - no urethritis

127
Q

A 63-year-old woman presents with a two week history of increasing discomfort in her upper abdomen. There are no other symptoms and there is no history of trauma. She had similar pain six weeks ago which resolved spontaneously. Her medical history includes an episode of acute pancreatitis two years ago, type 2 diabetes and hypertension. The only medications she takes are metformin and perindopril. She does not smoke and drinks about four glasses of wine a week. On examination there is some fullness in her left upper quadrant and this area is dull to percussion. A CT scan is performed (Image 1, 2). Which one of the following is the most likely diagnosis? Choose the single best answer. Pancreatic pseudocyst Gastric outlet obstruction Splenic haematoma Gastrointestinal stromal tumour Ascites A large cystic structure is visible in the left upper quadrant of the abdomen. The stomach is compressed anteriorly. The contents are homogeneous and of the options provided, this is most likely to be a pa

A

A large cystic structure is visible in the left upper quadrant of the abdomen. The stomach is compressed anteriorly. The contents are homogeneous and of the options provided, this is most likely to be a pancreatic pseudocyst. The episode of pain six weeks early may well have been acute pancreatitis - from which pseudocyst formation is a well recognised complication. Most patients with ascites would have a prodromal illness - something suggestive malignancy or chronic liver disease. Sometimes gastrointestinal stromal tumours can get to this size - but they usually have more of a heterogenous appearance. There is nothing from the history that would support a diagnosis of splenomegaly or splenic trauma - the latter of which would also have a heterogenous appearance. Whilst judged from the axial view alone, this lesion could be a splenic cyst or haematoma, a normal spleen can be seen in the coronal view. Patients with gastric outlet obstruction would have symptoms of vomiting. Learning Points Pancreatic pseudocyst formation is an important, but uncommon cause of upper abdominal cystic swellings.

128
Q

Webber and Renee, which is test of bone conduction? What is an abnormal Rene

A

Abnormal Rene: bone conduction better than air conduction. An abnormal Rene is found in conductive hearing loss. Bone conduction- Webber. Lateralizes to bad ear in CHL. Lateralizes to good ear in SHL. In CHL- webber Lateralizes to bad ear. Rene- Bone conduction better than air conduction. CHL- bad ear - webber lateralization and abnormal Rene on same side. In SNHL- Webber : good ear and Renne no abnormality ( air better than bone). If ear that Lateralizes webber has a normal Rinne—- sensory

129
Q

Webber and Renee, which is test of bone conduction? What is an abnormal Rene

A

Abnormal Rene: bone conduction better than air conduction. An abnormal Rene is found in conductive hearing loss. Bone conduction- Webber. Lateralizes to bad ear in CHL. Lateralizes to good ear in SHL. In CHL- webber Lateralizes to bad ear. Rene- Bone conduction better than air conduction. CHL- bad ear - webber lateralization and abnormal Rene on same side. In SNHL- Webber : good ear and Renne no abnormality ( air better than bone). If ear that Lateralizes webber has a normal Rinne—- sensory

130
Q

A 7-year-old boy presents with a one-day history of right ear discharge. This was preceded by nasal congestion, rhinorrhoea, febrile illness and significant ear pain. The pain improved last night and the boy noted significant ear discharge. He is currently afebrile, there is a purulent discharge from the right ear canal. The left tympanic membrane appears erythematous and mobile. His Weber test lateralises to his right ear, Rinne is abnormal on the right and normal on the left. In addition to simple analgesia, which one of the following is the most appropriate treatment? What does hearing test indicate. treatment? Choose the single best answer. Oral amoxicillin and further review Amoxicillin and clavulanic acid and further review Ear toilet and further review Ciprofloxacin ear drops, ear toilet and further review Systemic amoxicillin, ear toilet and admission Otitis media involves infection of the middle ear by pathogens ranging from viruse

A

Hearing test- Webber Lateralizes to right and abnormal on the right (BC>AC). CHL- Lateralizes to bad ear with abnormal Rinne. Makes sense- no TM as ruptured on the right. Otitis media involves infection of the middle ear by pathogens ranging from viruses to various bacteria, most commonly Streptococcus pneumoniae, Haemophilus influenzae and Morazella catarrhalis. Otitis media most commonly presents in younger children with fever and ear pain manifested by grasping of the ear. It can follow an upper respiratory tract infection and the build up of purulent fluid can lead to conductive hearing loss of the affected ear. When there is a spontaneous resolution of otalgia followed by release of purulent fluid, a tympanic membrane perforation must be suspected. Despite being mostly a bacterial infection the use of antibiotics in its management is reserved for: Children with severe fever: > 39 degrees Moderately to severely systemically ill Failure of improvement after 48-72 hours Vulnerable category (<6 months old, immunocompromised, Aboriginal or Torres Strait Islander, only one hearing ear) Possible suppurative complications The child described in this case presents with a localised ear infection that has not persisted for more than 48 hours, but presents with a possible suppurative complication (significant ear discharge). This indicates there is likely a bacterial infection superimposed over the initial viral infection. Therefore ciprofloxacin ear drops, along with ear toilet and further review at 48-72 hours is the most appropriate treatment. Topical ear drops are more effective than oral antibiotics in the case of a perforated eardrum with active discharge, as they can reach the middle ear and deliver a much higher dose of antibiotic. Oral antibiotics are recommended for patients with an intact tympanic membrane and AOM not responding to 72 hours of conservative management Learning Points If the tympanic membrane is perforated, topical antibiotics have better penetration to the middle ear than systemic antibiotics.

131
Q

What are red flags for acute otitis media. When should antibiotics be subscribed. What if recent ABs or allergy. 3. Does a ruptured TM need different ABs. 4. What is supprative OM. 5. How is chronic OM managed

A
  1. Ruptured- Ciprofloxacin drops with aural toilet and 48 hour review. 4. Chronic otitis media (OM) refers to a group of chronic inflammatory diseases of the middle ear, which often affects children. Chronic suppurative otitis media (CSOM) is characterized by a persistent drainage from the middle ear through a perforated tympanic membrane (TM). The condition is often seen in patients with a history of acute otitis media with TM rupture and presents with painless otorrhea and conductive hearing loss. The diagnosis is confirmed by detection of a TM defect during otoscopy. Treatment consists of conservative measures (e.g., antibiotic drops) and tympanoplasty if conservative management fails. Chronic otitis media with effusion (OME) is most likely caused by eustachian tube dysfunction (ETD) and is characterized by a buildup of effusion behind the intact TM. Clinically, OME presents with a sensation of fullness or pressure in the ear, as well as conductive hearing loss. The effusion often resolves spontaneously and treatment (e.g., placement of tympanostomy tubes) is only indicated if hearing impairment occurs or if the effusion persists. Acute OME in adults should raise suspicion of a nasopharyngeal malignancy and prompt swift evaluation. 5. Treatment Goal: restore integrity of the tympanic membrane , prevent permanent hearing loss Conservative treatment: aural toilet; topical fluoroquinolone (e.g., ciprofloxacin) Surgical treatment: tympanoplasty with insertion of an autologous graft
132
Q

MMSE- what is total score. Bands for each Category f dementia.

A

Diagnostic criteria A maximum of 30 points is possible A patient who scores 24 points or less is generally considered to have dementia. 20–24 points: mild dementia 13–20 points: moderate dementia < 13 points: advanced dementia

133
Q

Essential tremor vs pill roll tremor

A

Essential tremor- worse with movement Pill roll- PD- resting

134
Q

Essential tremor- what inheritance.

A

Consider an essential tremor in a patient presenting with chronic bilateral hand tremors, without further neurological deficits and positive family history. Autosomal dominant.

135
Q

Tremors

A
136
Q

An otherwise well 56-year-old woman presents after an episode of upper abdominal pain. The pain came on suddenly, lasted about 30 minutes and then settled. Her medical history is unremarkable and in particular there is no cancer history. She does not take any medications. She has a 15 pack-year smoking history and drinks about two glasses of wine at weekends. The physical examination is normal. An ultrasound examination is performed. Which one of the following is the most likely diagnosis? Choose the single best answer. Liver cyst Haemangioma Focal nodular hyperplasia Metastatic deposit Hydatid disease

A

There is a solid lesion in segment 8 of the liver. Being solid excludes a simple cyst and hydatid disease. The lesion has a heterogenous appearance with a central scar. The rest of the liver appears normal. This lesion has the characteristics of focal nodular hyperplasia. These lesions are the second most common benign lesion found in the liver - after haemangioma. An haemangioma would have a more uniform appearance. Whilst this isolated solid lesion could be a metastatic deposit, the absence of any cancer history makes this diagnosis less likely. However, more information will be required and this ultrasound should be followed up with magnetic resonance imaging (MRI).

137
Q

Benign liver lesions

A

Most common- hemangioma. FNH. Then hepatic adenoma- linked OCP. Stop OCP immediately.

138
Q

A 76 year-old female presents to her GP with her daughter, who has recently become concerned around her mother’s ability to cope at home. She reveals that over the last two months, her mother’s memory has been “poor”, often forgetting to shower in the morning or pay her bills on time. On further history, you are unable to gather much information from the patient herself, as she presents as withdrawn and quiet. She is unable to recall what she had for breakfast and cannot state the day of the week or month. She is unable to recount any childhood memories, however she is able to talk about the early years of her marriage to her late husband, who passed away six months ago. Her past medical history is notable for hypertension, high cholesterol, osteoporosis and urinary stress incontinence. Her medications include captopril, simvastatin and denosumab. On examination, vital signs are normal, except for a raised blood pressure of 152/94mmHg. Her face is expressionless and her gait it slow. Reflexes are all normal, with an absence of primitive reflexes. Mini-mental state examination reveals a score of 18/30, with the patient frequently becoming frustrated and teary during the test. You notice a tremor of her right hand that becomes apparent when she writes. Which one of the following is the most likely diagnosis? Choose the single best answer. Frontotemporal dementia Vascular dementia Parkinson’s disease Depression Alzheimer’s disease The correct answer to this question is depression.

A

This patient presents with a two month history of memory impairment and increased difficulty in completing her activities of daily living. This has occurred in the context of her husband’s death six months ago. Her memory loss is patchy, with impairment of both short-term memory (e.g. unable to remember breakfast) and variable impairment of long-term memory (e.g. unable to recall childhood memories, yet able to remember early years of marriage). On examination, she demonstrates features of psychomotor retardation, including reduced facial reactivity and slowness of gait. Her MMSE is 18/30, indicating moderate cognitive impairment, and she seems frustrated and upset by her impaired performance. All of these features are highly suggestive of depression (presenting as pseudodementia) being the most likely cause of her presentation. Depression is a diagnosis, whereas pseudodementia is a label applied to one specific clinical presentation of that diagnosis. Frontotemporal dementia is not a likely diagnosis in this case. Patients with frontotemporal dementia will generally present with a number of features suggestive of frontal lobe impairment, including social disinhibition, incontinence and hyperorality. Primitive reflexes may be present. This patient has none of these symptoms (although she does have a pre-existing history of stress incontinence). Classically, patients with frontotemporal dementia will be younger than those with Alzheimer’s dementia, dementia with Lewy bodies, and vascular dementia. They also tend to have relatively higher MMSE scores. A MMSE score of 18/30 is not consistent with this diagnosis. For these reasons, frontotemporal dementia is not the correct answer. This patient exhibits risk factors for vascular dementia, with advanced age, hypertension and high cholesterol all contributing to an increased risk. Having said that, vascular dementia tends to progress over a period of years in a “step-wise” fashion, with patients experiencing sudden and noticeable cognitive insults with each cerebrovascular event. Signs of upper motor neuron (UMN) injury are also often present on physical examination. The cognitive impairment in this patient appears to have developed more quickly than one would expect in vascular dementia and there is no evidence of an UMN lesion on physical exam. Therefore vascular dementia is not a correct answer option. Parkinson’s disease is very unlikely in this case, as the dementia associated with this progressive neurological illness develops years after it is first diagnosed. There is no history of Parkinson’s disease in this patient and whilst she does have a tremor on physical examination, this is not the resting “pill-rolling” tremor usually seen in Parkinson’s. It sounds more like an essential tremor. Alzheimer’s disease is the most common single cause of dementia (combined Alzheimer’s and vascular dementia is more common overall). It typically develops over a period of years and results in insidious cognitive decline. Patients will generally experience impairment in short-term episodic memory (e.g. forgetting appointments, forgetting tasks), with sparing of earlier memories. This patient’s rapid cognitive decline and pattern of memory impairment is inconsistent with a diagnosis of Alzheimer’s disease, as is the frustration and distress noticed during MMSE testing. Patients with Alzheimer’s will often have limited insight into their poor MMSE scores compared with patients who have depression. For these reasons, Alzheimer’s disease is an incorrect answer option.

139
Q

LMN features mnemonic - Bs FFART

A

Babinsky ( upcountry g plantar), fasciculations and flaccid paralysis, atrophy reflexes decrease, tone decreased LMN lesions present with muscle atrophy, fasciculations (muscle twitching), decreased reflexes, decreased tone, negative Babinsky sign, and flaccid paralysis

140
Q

Classic LMN pure

A

Poliomyelitis, spinal muscle atrophy (floppy baby) , bells poliomyelitis and spinal muscular atrophy are two classic examples of isolated LMN disease. CNS, the virus destroys the anterior (ventral) horn of the spinal cord, resulting in LMN paralysis. Because LMNs originate in the anterior horn of the spinal cord, this results in LMN signs such as asymmetric weakness, flaccid paralysis, fasciculations, hyporeflexia, and muscle atrophy. The infection could also result in respiratory involvement leading to respiratory paralysis. Other systemic signs of infection include fever, headache, nausea, and malaise. congenital degeneration of the anterior horn of the spinal cord. Unlike polio, this results in symmetric weakness, flaccid paralysis, fasciculations, hyporeflexia, and muscle atrophy. Because it is congenital, it has classically had associations with a “floppy baby” with marked hypotonia and tongue fasciculations. This disease carries an autosomal recessive inheritance and is due to a mutation in the SMN1 gene. Spinal muscular atrophy type I is also known as Werdnig-Hoffmann disease, which is the most severe form of the disease and usually results in childhood death due to respiratory failure.[8] Spinal muscular atrophy type II and III are less severe and often result in a lifelong inability to ambulate.

141
Q

Bells

A

However, if the lesion occurs anywhere from the facial nucleus along CN VII, it will result in LMN deficits, affecting the ipsilateral side of the face and involve both the upper and lower muscles of facial expressions. This condition presents as incomplete eye closure (orbicularis oculi), dry eyes, corneal ulceration, hyperacusis, and taste sensation loss to the anterior tongue. Because the forehead is involved, the affected individual will be unable to wrinkle their forehead (lift their eyebrows).

142
Q

Spinal cord lesions

A
143
Q

What is bulbar palsy

A

“Bulbar” Palsy (umn), also called Progressive Bulbar Palsy, prominently affects the muscles involved in speech, swallowing, and tongue movements

144
Q

What is bulbar palsy and ALS

A

“Bulbar” Palsy ), also called Progressive Bulbar Palsy, prominently affects the muscles involved in speech, swallowing, and tongue movements ALS is a disease of “motor neurons”, the cells that initiate and control the movement of muscles. Motor neurons are characterized as “upper” if they originate in the brain, and “lower” if they originate in the spinal cord. ALS affects both upper and lower motor neurons, although each person with ALS has varying amounts of upper and lower motor neuron disease. Upper motor neuron disease causes stiffness, which is called “spasticity”. Lower motor neuron disease causes weakness, loss of muscle (“atrophy”) and muscle twitching (“fasciculations”). ALS may begin with abnormalities of upper or lower motor neurons. “Bulbar” ALS, also called Progressive Bulbar Palsy, prominently affects the muscles involved in speech, swallowing, and tongue movements. Primary Lateral Sclerosis (PLS) is a variant of ALS that affects only upper motor neurons. People with PLS have problems with severe stiffness (“spasticity”) that may affect their ability to speak, swallow, and walk. Progressive Muscular Atrophy (PMA) affects only lower motor neurons, causing weakness and loss of muscle bulk (“atrophy”). PLS and PMA are considered variants of ALS, and people with these disorders may progress to develop true ALS.

145
Q

Rule of 4s

A

THE FOUR RULES In the rule of 4 there are 4 rules There are 4 structures in the ‘midline‘ beginning with M There are 4 structures to the ‘side‘ (lateral) beginning with S There are 4 cranial nerves in the medulla, 4 in the pons and 4 above the pons(2 in the midbrain) The 4 motor nuclei that are in the midline are those that divide equally into 12 except for 1 and 2, that is 3, 4, 6 and 12 (5, 7, 9 and 11 are in the lateral brainstem)

146
Q

Rule of 4s - 1st rule - midline

A

MEDIAL STRUCTURES The 4 medial structures and the associated deficits are: Motor pathway (or corticospinal tract): contralateral weakness of the arm and leg Medial lemniscus: contralateral loss of vibration and proprioception in the arm and leg Medial longitudinal fasciculus: ipsilateral inter-nuclear ophthalmoplegia (failure of adduction of the ipsilateral eye towards the nose and nystagmus in the opposite eye as it looks laterally) Motor nucleus and nerve: ipsilateral loss of the cranial nerve that is affected (3, 4, 6 or 12)

147
Q

The rule of 4- sides

A

Sides The 4 ‘side’ (lateral) structures and the associated deficits are: Spinocerebellar pathway: ipsilateral ataxia of the arm and leg Spinothalamic pathway: contralateral alteration of pain and temperature affecting the arm, leg and rarely the trunk Sensory nucleus of the 5th cranial nerve: ipsilateral alteration of pain and temperature on the face in the distribution of the 5th cranial nerve (this nucleus is a long vertical structure that extends in the lateral aspect of the pons down into the medulla) Sympathetic pathway: ipsilateral Homer’s syndrome, that is partial ptosis and a small pupil (miosis)

148
Q

Cranial nerves- position in brain stem and medial/ lateral

A

CRANIAL NERVES The 4 cranial nerves in the medulla are CN9-12: Glossopharyngeal (CN9): ipsilateral loss of pharyngeal sensation Vagus (CN10): ipsilateral palatal weakness Spinal accessory (CN11): ipsilateral weakness of the trapezius and stemocleidomastoid muscles Hypoglossal (CN12): ipsilateral weakness of the tongue The 12th cranial nerve is the motor nerve in the midline of the medulla. Although the 9th, 10th and 11th cranial nerves have motor components, they do not divide evenly into 12 (using our rule) and are thus not the medial motor nerves. The 4 cranial nerves in the pons are CN5-8: Trigeminal (CN5): ipsilateral alteration of pain, temperature and light touch on the face back as far as the anterior two-thirds of the scalp and sparing the angle of the jaw. Abducens (CN6): ipsilateral weakness of abduction (lateral movement) of the eye (lateral rectus). Facial (CN7): ipsilateral facial weakness. Auditory (CN8): ipsilateral deafness.The 6th cranial nerve is the motor nerve in the medial pons. The 7th is a motor nerve but it also carries pathways of taste, and using the rule of 4 it does not divide equally in to 12 and thus it is not a motor nerve that is in the midline. The vestibular portion of the 8th nerve is not included in order to keep the concept simple and to avoid confusion. Nausea and vomiting and vertigo are often more common with involvement of the vestibular connections in the lateral medulla. The 4 cranial nerves above the pons are CN1-4: Olfactory (CN1): not in midbrain. Optic (CN2): not in midbrain. Oculomotor (CN3): impaired adduction, supraduction and infraduction of the ipsilateral eye with or without a dilated pupil. The eye is turned out and slightly down. Trochlear (CN4): eye unable to look down when the eye is looking in towards the nose (superior oblique). The 3rd and 4th cranial nerves are the motor nerves in the midbrain.

149
Q

Interpretation of rules of 4

A

INTERPRETATION Localisation and brainstem syndromes a medial brainstem syndrome will consist of the 4 M’s and the relevant motor cranial nerves a lateral brainstem syndrome will consist of the 4 S’s and either the 9-11th cranial nerve if the lesion is in the medulla, or the 5th, 7th and 8th cranial nerve if the lesion is in the pons. If there are signs of both a lateral and a medial (paramedian) brainstem syndrome, then one needs to consider a basilar artery problem, possibly an occlusion. These syndromes correlate with the blood supply of the brainstem: 1. paramedian branches — lesions cause medial (or paramedian) brainstem syndromes 2. long circumferential branches (SAP) — lesions cause lateral brainstem 3. syndromes (can also occur with unilateral vertebral artery occlusion) - superior cerebellar artery (SCA) -anterior inferior cerebellar artery (AICA) - posterior inferior cerebellar artery (PICA)

150
Q

An 81-year-old woman is admitted to hospital following recurrent falls at home. Her daughter is concerned about her level of cognitive impairment over the last 5 years, which has acutely worsened every 2-3 months in the past 12 months. There are concerns about her slowness to respond to questions, impaired concentration, and inability to complete simple tasks such as using the home telephone. She has a history of hypertension, hyperlipidaemia, and osteoarthritis. On review, the patient is tearful and distressed as she ‘wants to go home’. She believes she is ‘at a stranger’s house’ and that it is ‘1940’. Her peripheral neurological examination is unremarkable. Which one of the following is the most likely pathological process? Choose the single best answer. Vascular dementia Frontotemporal dementia Lewy body dementia Alzheimer’s disease Mixed dementia The stepwise pattern of deterioration on a background of vascular disease is suggestive of vascular dementia

A

The stepwise pattern of deterioration on a background of vascular disease is suggestive of vascular dementia. These episodes of deterioration occur due to vascular events, such as cerebrovascular accidents. The other types of dementia listed are characterised by progressive cognitive impairment. Patients with Lewy body dementia often have fluctuating cognition and attention, visual hallucinations or REM sleep behaviour disorder, making this less likely. There is no personality or behavioural change that would indicate the most common type of frontotemporal dementia. Furthermore, the patient is less likely to have Alzheimer disease which often presents with insidious onset and progressive memory impairment.

151
Q

3 prevention with exisitring RF: Risedronate Denosumab Raloxifene

2-year-old woman sustains a fracture to her left neck of femur following a fall from standing height and a total arthroplasty (total hip replacement) is performed. She is usually independent at home and mobilises with a four wheel walker. Her medical history includes hypercholesterolaemia, epilepsy, urinary incontinence, oesophageal dysmotility, hypertension, previous provoked above-knee deep vein thrombosis (ten years ago), and excision of a squamous cell carcinoma from her face. Her current medications include rosuvastatin, levetiracetam, perindopril, calcium/colecalciferol and oxybutynin. A bone densitometry (DEXA) scan one year ago showed T-score of -2.0 at the femoral neck. Her current thyroid function tests and vitamin D and calcium levels are within normal range.

Which one of the following is the most appropriate choice for fracture prevention?
Choose the single best answer.

Risedronate
Denosumab
Calcium/colecalciferol only
Hip protectors
Raloxifene

A

This patient has sustained a minimal trauma (fragility) fracture with known osteopaenia (T-score between -2.5 and -1.0) diagnosed one year ago. Therefore, she would benefit from initiation of anti-resorptive therapy to reduce her fracture risk in addition to her current calcium/colecalciferol supplementation. Bisphosphonates (e.g. risedronate) would be the most appropriate first line therapy in a patient without upper gastrointestinal disorders (this includes strictures or dysphagia). This is because bisphosphonates can cause oesophagitis as a side effect. Her oesophageal dysmotility makes her an unsuitable candidate for bisphosphonate therapy. She would be better treated by denosumab, a human monoclonal antibody used in the treatment of osteoporosis. It reduces the incidence of hip fracture by 40% and of clinical vertebral fracture by 68%. It is given as a subcutaneous injection every six months.

Use of hormonal therapy (e.g. raloxifene) should be carefully weighed against her cardiovascular risk, thrombotic risk,and cancer risk (specifically breast cancer). Given she has two other cardiovascular risk factors and has suffered a previous above-knee deep vein thrombosis, hormonal therapy would be a less appropriate choice. There is good evidence (level 1a) that hip protectors are not effective in reducing the incidence of hip fracture following a fall, in elderly people living in the community, but may be effective for elderly people with a previous history of hip fracture living in aged-care facilities.

152
Q

A 70-year-old man presents to the Emergency Department with confusion, jaundice and a visibly distended abdomen. He reports having poor sleep and concentration over the past few days. His wife attributes this to his poor diet and a long-standing habit of binge drinking alcohol, however, this current episode is “unlike his usual drunk self”. He admits to drinking 8 – 10 standard drinks a day for many years.

His vital signs show the following:
Respiratory rate of 12/min
Heart rate of 72/min
Temperature 38.6℃
Oxygen saturation 98% on RA
Blood pressure 130/80
On examination, he is visible jaundiced and a grossly distended abdomen with shifting dullness. He also has conjunctival jaundice, palmar erythema and several spider naevi across his chest and arms. His GCS is 10. The cardiorespiratory examination is unremarkable.

Which one of the following is the most important next step in investigation?
Choose the single best answer.

Urgent CT abdomen
Measure serum Alpha fetoprotein (AFP) assay
Perform a Mini-mental State Examination (MMSE)
Urgent paracentesis, with microscopy and culture
Measure serum ammonia levels

A

This patient presents with acute episode jaundice, ascites, confusion and examination findings of liver cirrhosis – this suggests a sudden decompensation of chronic liver disease.

Infection is a common, life-threatening complication of decompensated liver disease. In this case, the combination of a raised temperature, ascites, new-onset encephalopathy and jaundice would suggest a possible case of spontaneous bacterial peritonitis (SBP). Urgent ascitic fluid and microscopy and culture should be undertaken to exclude this. Urgent IV antibiotics would be necessary if SBP is confirmed.

Performing an MMSE and measuring serum ammonia levels aid in the management of encephalopathy but excluding SBP – a life-threatening complication – should be prioritised first in the acute setting.

Serum AFP is raised in hepatocellular carcinoma. Whilst he may have hepatocellular carcinoma, this test should not be done in the acute setting.

A CT scan may show a cirrhotic liver but this is already strongly suggested on history/examination, and it would not aid in the clinical diagnosis and management of SBP.

153
Q

Flag for Review LaterPrevious QuestionNext Question
A 79-year-old man who lives alone is brought to the General Practitioner by his son due to concerns about his father’s memory. Over the past 12 months he has missed several doctors appointments and his son’s recent birthday dinner, despite reminders. His son reports that often his father has word-finding difficulty in conversation and finds himself prompting his father. On further questioning the father states his is still able to perform tasks around the home such as cooking and cleaning, but they sometimes take a little longer if he misplaces an item and needs to find it again. His son expresses concern that he has often found his father confused. The patient is aware his memory is not what it once was, but brushes off his son’s concerns and feels he is coping just fine on his own at home. His medical history is significant for hypothyroidism, osteoarthritis and hypertension. His medications are thyroxine, perindopril and ibuprofen and paracetamol as needed.

Which one of the following is the most likely diagnosis?
Choose the single best answer.

Frontotemporal dementia
Mild cognitive impairment
Vascular dementia
Alzheimer’s dementia
Lewy body dementia

A

Frontotemporal dementia
FTD often presents for the first time at a younger age (<65) and the decline can be quicker than in other dementia types (1). This patient does not fit the picture of FTD as his cognitive decline appears stable over the past 12 months and there is no evidence of personality change, inappropriate social behaviours or blunted emotions that is usually seen in FTD (1).

Mild cognitive impairment
MCI is characterised by a memory worse than expected for a patient’s age, however they do not meet the criteria for diagnosis of dementia and are still able to perform activities of daily living (ADLs) independently and maintain social functioning (1). This diagnosis best fits this patient’s presentation as his is still managing on his own at home.

Whilst MCI is a worsened memory condition, it does not mean a patient will go onto develop dementia, however they are at a greater risk and therefore should be followed up to ensure patient safety (1).

Vascular dementia
This form of dementia typically follows a stepwise decline in cognitive function and usually co-exists with a cardiovascular risk profile and therefore is associated with detected or un-detected strokes (2). To aid differentiating vascular from Alzheimers dementia, as these are the 2 most common forms of dementia, MRI can be utilised by demonstrating vascular disease changes (2). Although this patient has hypertension, his cardiovascular risk is minimal and there is no history of cardiac and cerebrovascular events that would increase the likelihood of a vascular origin of his cognitive decline.

Alzheimers dementia
Although he has decline in the memory domain and is of the correct age bracket for AD, this patient does not yet meet the criteria for AD as his cognitive deficits do not interfere with everyday life and his still maintains his independence at this stage (3).

Lewy body dementia
Lewy body dementia usually presents concurrently with Parkinsonism. Features usually require for diagnosis of LBD such as fluctuating alertness and visual hallucinations are absent in this patient making another diagnosis more likely (1).

154
Q

Proximal DVT Mx

A 50-year-old woman presents with a two day history of right-sided leg pain. The pain is present over her entire right leg. She underwent resection of a rectal carcinoma with formation of a stoma one week ago. She is being planned for further adjuvant chemotherapy. She had a deep vein thrombosis when she was 30 years old during pregnancy. On examination, her right leg is swollen and tender. There is no overlying erythema or warmth. A lower limb ultrasound reveals an occlusive thrombus in the common femoral vein. Her renal function is normal.

Which one of the following is the most appropriate treatment approach?
Choose the single best answer.

6 weeks Enoxaparin
3 months Apixaban
3 months Fondaparinux
6 months Warfarin
6 months Dabigatran

A

The patient has an occlusive thrombus in the common femoral vein (i.e., proximal DVT) which carries a higher risk of embolization and death.

Given that this patient is arguably ‘cancer free’ (from her operation one week ago which is ideally curative) the DVT arises in the setting of surgical provocation. For patients with an index DVT in the setting of surgical provocation, the risk of recurrence is 15% over the next 5 years. As such, a course of anticoagulation of at least 3 months with Apixaban is recommended.

The aim of immediate anticoagulation is to halt the prothrombotic state and to reduce thrombus propagation and/or embolization. Ongoing anticoagulation (6 weeks to 6 months) aims to avert new thrombus formation while the initial clot is removed or stabilized by homeostatic mechanisms.

DOACs have been shown to be as effective as warfarin, LMWH and UFH, LMWH, and warfarin for the treatment of DVT. DOACs require no monitoring and bleeding complications are similar to those in patients on warfarin. The DOACs have a rapid onset of action and are generally, recommended in preference to warfarin, UFH, and LMWH except for special populations (i.e., severe hepatic, renal impairment, and active cancer, etc.) A course of treatment for a provoked (minor or major transient risk factors) DVT is recommended for 3 months.

Other options:

6-weeks Enoxaparin: While this patient may have been appropriate in the initial post-op period the patient should have been transitioned to a DOAC for continued prophylaxis after recognition of the likely surgical-related DVT.

6-months Warfarin: Typical indications for warfarin include valvular atrial fibrillation and where DOACs are unable to be implemented (i.e., severe hepatic or renal failure). In thromboprophylaxis, DOACs exhibit similar efficacy and safety to Warfarin. However, warfarin use is confounded by the ongoing requirement for dose adjustment and monitoring (i.e., INR) and numerous diet or drug-drug interactions.

3-months Fondaparinux: The use of Fondaparinux is generally reserved as an alternative to LMWH and UFH for patients with heparin-induced thrombocytopenia.

6-months Dabigatran: The only use for Dabigatran, in Australia, is in the management of atrial fibrillation.

  • *Learning Points**
  • *For a provoked DVT the risk of developing recurrent VTE within the next 5 years is ~15%. These patients should be treated with a 3-month course of Apixaban, unless contraindicated**
155
Q

difference between benzylpenicillin and benzathine penicillin G

A

What is the difference between benzathine penicillin G and benzylpeniccilin G?

Penicillin G benzathine intramuscular injections are primarily used to treat Treponema infections and to eradicate colonization of group A β-hemolytic streptococci in chronic carriers. Benzathine is the least soluble salt form of penicillin G, thus providing the longest duration of action (up to 3 weeks).

It is in the penicillin and beta lactam class of medications and works via benzylpenicillin.
The **benzathine component slowly releases the penicillin** making the combination long acting. Hence choice for rheumatic fever prophalaxis

Benzylpenicillin 1.2 g with Azithromycin for moderate CAP pnuamonia- 5-7 days.

Prophylactic benzathine penicillin G until age 28 (if infected at 18)

This is the clinical picture of an episode of rheumatic fever, complicated by carditis and mitral valve regurgitation. The rash is that of erythema marginatum, which is seen in about 10% of individuals with rheumatic fever. It is the cardiac complication which is the only complication associated with rheumatic fever that will have any potential serious long-term sequelae. To minimise this risk, such patients should be treated with benzathine penicillin G for some ten years or until the age of 25 - whichever is the longer. An intramuscular dose should be given every three weeks. With this form of secondary prophylaxis, recurrences are rare after the age of 21 or more than 10 years after the initial episode of acute rheumatic fever.

The recommendation on the use of prophylactic antibiotics in for individuals who have had an episode of acute rheumatic fever is to reduce the risk of streptococcal infection and rheumatic fever recurrence. A Cochrane systematic review (cohort studies) showed that the risk of rheumatic fever recurrences could be reduced with intramuscular benzathine penicillin. This antibiotic also improves the long term cardiac outcomes in patients with rheumatic heart disease. If the patients are allergic to penicillin, erythromycin is the recommended alternative.

Learning Points
Antibiotic prophylaxis is recommended in patient who have had an episode of acute rheumatic fever.

156
Q

2 day hx of back pain and mild fever in IV drug user. Choice of imaging.

A

This patient has fever, erythema and tenderness of the spine, and normal neurological examination of the lower limbs. In the context of intravenous drug use and localised tenderness in the spine, the most suspicious underlying source would be bacteraemia resulting in osteomyelitis or an epidural abscess. An epidural abscess is more likely to cause upper motor neuron lesion signs in the lower limbs due to vertebral compression. The best investigation for the exclusion of an epidural abscess is an MRI.

An X-ray is another investigation of choice for vertebral osteomyelitis and may show destructive changes in the bone. However, an X-ray is normally unremarkable in the first 2 weeks of osteomyelitis and only shows destructive changes in advanced osteomyelitis. This patient with a two-day history is unlikely to have advanced disease.

MRI is a more sensitive test during the initial weeks immediately following the onset of the disease. The confirmatory test for vertebral osteomyelitis is a CT-guided needle aspiration biopsy. At least three sets of blood cultures would be vital in diagnosis, immediately prior to initiating empirical intravenous antibiotic treatment.

157
Q

ECG reveals hyperacute T waves, prolonged PR intervals and broadened QRS.

A

hyperacute T waves, prolonged PR intervals and broadened QRS.

Abnormalities of the T-wave

ECG schematic (paper speed: 25 mm/s)

– Left: normal T wave
– Top right: Benign early repolarization (BER) is a usually benign variant that is associated with high vagal tone, physical fitness, and younger age. These T waves are narrow, asymmetrically peaked, and have upwardly concave ascending limbs; the ST segment may be elevated.
– Middle right: The peaked T waves of hyperkalemia are narrow and symmetrical.
– Bottom right: Hyperacute T waves may occur in the early stages of an ST-segment elevation myocardial infarction (STEMI). They are broad and asymmetrically peaked, with no upward concavity of the ascending limb; the ST segment may be elevated.

158
Q

one day history of nausea, vomiting and diarrhoea. He has had no subjective fevers. Current medical history includes acute myeloid leukaemia, for which he has recently completed his first course of chemotherapy.

On examination, he is afebrile and all other vital signs are within normal limits. ECG reveals hyperacute T waves, prolonged PR intervals and broadened QRS.

Which one of the following electrolyte abnormalities is most likely to be present?

  • Hypokalaemia
  • Hypercalcamia
  • Hyponatraemia
  • Hypouricaemia
  • Hyperphosphataemia
A

Standard serum concentrations electrolyte

Standard range*

  • Potassium: 3.5–5.0 mmol/l
  • Magnesium: 0.7–1.1 mmol/l
  • Sodium: 135–146 mmol/l
  • Calcium: 2.20–2.67 mmol/l

Sodium is the primary ion affecting the QRS complex.
Potassium is the primary ion affecting T waves
If there is a QT abnormality, you should check a serum calcium!

Calcium is a positive inotrope, and increased contractility shortens the QT

– Calcium transport affects potassium transport. More calcium causes bigger shifts in potassium, and may cause earlier depolarisation.

159
Q

one day history of nausea, vomiting and diarrhoea. He has had no subjective fevers. Current medical history includes acute myeloid leukaemia, for which he has recently completed his first course of chemotherapy.

On examination, he is afebrile and all other vital signs are within normal limits. ECG reveals hyperacute T waves, prolonged PR intervals and broadened QRS.

Which one of the following electrolyte abnormalities is most likely to be present?

  • Hypokalaemia
  • Hypercalcamia
  • Hyponatraemia
  • Hypouricaemia
  • Hyperphosphataemia
A

Tumour lysis syndrome (TLS) is a collection of metabolic abnormalities caused by the rapid destruction of malignant cells. When these cells are destroyed, typically in the setting of cytotoxic therapy, intracellular ions become extracellular resulting in hyperkalaemia, hyperphosphataemia and hyperuricaemia.

Additionally, high free serum phosphate allows for the formation of calcium phosphate, resulting in a secondary hypocalcaemia. It is not associated with sodium abnormalities.

High serum levels of these electrolytes can result in nonspecific symptoms including gastrointestinal disturbance, lethargy, muscle cramping, arrhythmia and seizures.

Patients with malignancies with high proliferation rates (e.g. lymphoma, leukaemia and small cell lung cancer) and a significant tumour burden are at increased risk. In this demographic, preventative measures may be taken including pre-chemotherapy isotonic saline, phosphate binding agents and/or allopurinol.

Learning Points
In tumour lysis syndrome, intracellular ions become extracellular, resulting in hyperkalaemia, hyperphosphataemia, hyperuricaemia, and secondary hypocalcaemia.

hyperacute T waves, prolonged PR intervals and broadened QRS.

hyper acute T waves is from Hyper K, and QRS broadening.

PR interval prolonged- low calcium: late depol.

Calcium is a positive inotrope, and increased contractility shortens the QT

Calcium transport affects potassium transport. More calcium causes bigger shifts in potassium, and may cause earlier depolarisation.

160
Q

What does potassium do in cardiac cycle.

A

Returns to resting membrane potential (after depol) - ie t wave

K

Alternatively, if you’ve got too much potassium on board it’s easy to predict that the influx of potassium will be more impressive than it otherwise would.

Therefore the repolarisation curve will be steeper than normal, and overshoot. This will increase the size and volume of the electrical deflection measured on the ECG, which is of course the T wave. In extreme cases this will cause very early repolarisation, and at the same time an increase in basal K+ exchange across the membrane will slow the rate of sodium induced depolarisation. A sinusoidal waveform develops.

The opposite effect occurs with hypokalaemia. Depolarisation is less impressive, and T waves “flatten out”.

161
Q

What electrolyte causes Prolonged PR and QT intervals

A

PR interval - Or three to five small (1 mm) boxes

QT prolonged >460

Duration
Adult men: QTc = 390–450 ms
Adult women: QTc = 390–460 ms
Generally, the QT interval should not be more than half of the RR interval.

Other electrolytes that cause QT prolongation: Electrolyte disturbances (e.g., hypocalcemia, hypokalemia, hypomagnesemia)

A serum magnesium concentration < 0.62 mmol/L Magnesium: 0.7–1.1 mmol/l

Associated with tetany, torsades de pointes, hypokalemia, and hypocalcemia.

Hypomagnesemia
Prolonged PR and QT intervals

Clinical
Anorexia, nausea, vomiting
Muscle weakness, muscle cramps/spasms
Tremor
Ataxia, nystagmus
Seizures

162
Q

Calcium effects on ECG

A

Positive ionotrope- shoten QTC
Calcium is a positive inotrope, and increased contractility shortens the QT

increase in calcium– Calcium transport affects potassium transport. More calcium causes bigger shifts in potassium, and may cause earlier depolarisation. (shorter PR)

163
Q

appropriate diagnostic investigation for which disorder
Choose the single best answer.

  • ADAMTS 13
  • Anti-factor Xa levels
  • Fibrinogen level
  • Coagulation studies
  • HIT antibody testing
A
  • ADAMTS 13 - TTP
  • Fibrinogen and Coagulation - useful in ruling out other differentials for coagulopathy and thrombocytopaenia such as DIC
  • Anti-factor Xa levels- are used to monitor when using LMWH and UFH. They are not used is dx of HIT
  • HIT antibody testing- to confirm HIT : Immunoassays are the most widely used and detect the presence of the platelet factor 4-heparin antibody.
  • When suspicious of Autoimmune HIT:
    • Thrombocytopeania and no other cause of this Thrombocytopeania
      Timing (5-10 days)
      Thrombosis features
  • Laboratory testing includes two major types of antibody tests, immunoassays and functional assays.
  • Immunoassays are the most widely used and detect the presence of the platelet factor 4-heparin antibody.
  • Functional assays detect the ability of the antibody to bind and activate platelets; they are more resource-intensive however carry a higher specificity than immunoassays. The serotonin release assay is the gold standard for the diagnosis of HIT, it measures platelet activation by detecting the release of serotonin from platelets.
  • HIT antibody testing is widely available but the serotonin release assay is only performed at a few laboratories in Australia.
  • Therefore antibody testing is performed routinely to investigate HIT and the serotonin release assay is reserved for confirmatory testing in cases without conclusive antibody test results.
164
Q

What is HIT

A

HIT is a life-threatening, autoimmune complication of exposure to heparin. HIT results from the production of autoantibodies directed against platelet factor 4 in complex with heparin. This leads to the activation of platelets and thrombocytopaenia, as well as catastrophic arterial and venous thrombosis.

The clinical presentation of HIT is variable and the condition may occur with any heparin dose, schedule or route of administration, however, there is a greater incidence of HIT in patients receiving unfractionated heparin, as opposed to low molecular weight heparin. HIT typically occurs 5-10 days after the initiation of heparin, while clinical manifestations may include haemorrhage, thrombosis, as well as sequelae of thrombosis such as skin necrosis, limb gangrene or organ ischaemia.

Diagnosis of HIT is based on clinical and laboratory findings and should be suspected in a heparinised patient with new-onset thrombocytopaenia or the clinical symptoms of HIT. The 4Ts score may be used to calculate the pretest probability of HIT. It includes:

Thrombocytopaenia
Timing after heparin exposure [5-10 days]
Thrombosis or other clinical features
No oTher cause of thrombocytopaenia
Laboratory testing includes two major types of antibody tests, immunoassays and functional assays. Immunoassays are the most widely used and detect the presence of the platelet factor 4-heparin antibody. Functional assays detect the ability of the antibody to bind and activate platelets; they are more resource-intensive however carry a higher specificity than immunoassays. The serotonin release assay is the gold standard for the diagnosis of HIT, it measures platelet activation by detecting the release of serotonin from platelets. HIT antibody testing is widely available but the serotonin release assay is only performed at a few laboratories in Australia. Therefore antibody testing is performed routinely to investigate HIT and the serotonin release assay is reserved for confirmatory testing in cases without conclusive antibody test results.

Treatment of HIT should involve the cessation of all formulations of heparin and careful monitoring of clinical features and platelet count. Alternative anticoagulation should be initiated immediately, options include Fondaparinux or argatroban in the acute setting, followed by transition to a DOAC (Direct-Acting Oral Anticoagulant) or Warfarin. Warfarin should not be used until the platelet count has normalised as it may exacerbate the imbalance between anticoagulant and procoagulant proteins seen in HIT.

165
Q

Choice of drug for

1st mild c.diff , in well patient
1st recurrance ?

2nd ie Chronic c.diff

A

Australian guidelines currently endorse the use of metronidazole for mild C. difficile-related diarrhoea in an otherwise well patient,

vancomycin or fidaxomicin for first-recurrent, or refractory disease, and

faecal microbiota transplantation for second or subsequent recurrences or ongoing refractory disease.

166
Q

five days after a laparotomy and small bowel resection for an enterocutaneous fistula. The antibiotics he has been given since the operation are ceased and over the next three days the diarrhoea settles and he is making a good post-operative recovery. A stool specimen sent for analysis reveals the presence of Clostridioides difficile toxin.

Which one of the following is the most appropriate management plan?
Choose the single best answer.

Fidaxomicin1%
Your Answer
Vancomycin24%
Metronidazole47%
No change required to current management26%
Faecal microbiota transplantation1%

A

Diarrhoea in the hospitalised patient is extremely common, occurring in up to a third of admitted patients. The majority of cases are non-infective in aetiology, occurring as a consequence of antibiotic use, enteral feeding, the patient’s underlying illness, or other alterations to the gut microbiome.

The most common infective cause is Clostridioides (formerly Clostridium) difficle, accounting for up to 20% of cases. In the absence of key risk factors such as known outbreaks or immunocompromise, hospitalised patients are very unlikely to test positive for typical community-acquired bacterial, viral, or parasitic organisms.

Clinical symptoms should be correlated with test results when making a decision about whether to treat.
C difficile may be present on testing in asymptomatic individuals who are carriers of C. difficile, as well as during the recovery of an episode of C. difficile diarrhoea - these patients do not require treatment. Since this patient has no symptoms now, he does not require further antibiotic therapy. Repeat testing during the same episode and test for cure is not required.

Australian guidelines currently endorse the use of metronidazole (although vancomycin has similar efficacy, it is not the preferred first line for antibiotic stewardship reasons) for mild C. difficile-related diarrhoea in an otherwise well patient.

Vancomycin or fidaxomicin are recommended for first-recurrent or refractory disease.

Faecal microbiota transplantation (FMT) is the preferred treatment for second or subsequent recurrences or ongoing refractory disease. If FMT is not available, vancomycin or fidaxomicin may be used.

Severe disease is determined by the presence of any of:

  • leucocytosis,
  • severe abdominal pain,
  • elevated serum creatinine,
  • elevated blood lactate,
  • low serum albumin,
  • high fever,
  • or organ dysfunction.

Expert advice should be sought for severe disease. Oral or enteral vancomycin is the preferred first line (over IV vancomycin, due to inadequate penetration of the IV drug into the lumen of the colon). Data for the use of fidaxomicin in severe disease is lacking. In complicated cases, such as in the presence of hypotension, shock, ileus, and megacolon, IV metronidazole may be added to po/enteral vancomycin. The addition of intracolonic vancomycin may also be considered particularly in the presence of ileus.

Elsewhere in the world, vancomycin and fidaxomicin are increasingly used as first-line agents due to evidence of superiority over metronidazole.

Learning Points
Australian guidelines currently endorse the use of metronidazole for mild C. difficile-related diarrhoea in an otherwise well patient, vancomycin or fidaxomicin for first-recurrent, or refractory disease, and faecal microbiota transplantation for second or subsequent recurrences or ongoing refractory disease.

REFERENCES
Next Question

167
Q

what is considered as a positive tuberculin skin test

A

5mm in an immune compromissed- HIV/ chest x ray suggestive of TB

10 mm if from endemic area

15 mm in a healthy individual,

Classification of the Tuberculin Skin Test Reaction

An induration of 5 or more millimeters is considered positive in
-People living with HIV

-A recent contact of a person with infectious TB disease

– People with chest x-ray findings suggestive of previous TB disease

  • People with organ transplants
  • Other immunosuppressed people (e.g., patients on prolonged therapy with corticosteroids equivalent to/greater than 15 mg per day of prednisone or those taking TNF-a antagonists)

An induration of 10 or more millimeters is considered positive in
-People born in countries where TB disease is common, including Mexico, the Philippines, Vietnam, India, China, Haiti, and Guatemala, or other countries with high rates of TB

  • People who abuse drugs
  • Mycobacteriology laboratory workers
  • People who live or work in high-risk congregate settings (e.g., nursing homes, homeless shelters, or correctional facilities)
  • People with certain medical conditions that place them at high risk for TB (e.g., silicosis, diabetes mellitus, severe kidney disease, certain types of cancer, and certain intestinal conditions)
  • People with a low body weight (<90% of ideal body weight)
  • Children younger than 5 years of age
  • Infants, children, and adolescents exposed to adults in high-risk categories

An induration of 15 or more millimeters is considered positive in

-People with no known risk factors for TB

168
Q

migrant from India and migrated to Australia 12 years ago. He does not present with any fevers or cough in this visit. He has a medical history of pulmonary tuberculosis as a child and has not had the BCG vaccination since birth. He has no known allergies and he is not on any current medications.

On examination, he appears well and comfortable. His BMI is 22 kg/m². His vital signs are normal. His chest is clear on lung auscultation. A tuberculin skin test of 15mm at 48 hours is positive and an erect chest X-ray shows a Ghon focus, but no additional findings.

Which one of the following is the most appropriate management?
Choose the single best answer.

  • Induced sputum and low-dose CT chest
    Reassurance and follow-up in 3 months
    Prescribe Isoniazid and Rifapentine for 4 to 6 months
    Prescribe Isoniazid, rifampicin, ethambutol and pyrazinamide for 2 to 6 months
    Prescribe BCG vaccine immediately
A

The patient has no immunisation history but a positive tuberculin skin test and this is very suggestive of tuberculosis (TB), given he is from a country when the disease is endemic (India). The patient does not have any current respiratory symptoms and the chest X-ray does not show any active disease. This makes latent TB a possibility.

Most reactivations of latent TB occur within the first two years, but can be treated to prevent progression to active disease, particularly in younger and immunocompromised patients. Induced sputum and low-dose chest CT is necessary for microscopy and culture.

Treatment for latent TB is a two-drug regimen with Prescribe Isoniazid and Rifapentine for 4 to 6 months. If an active TB is established, a combination of Isoniazid, rifampicin, ethambutol and pyrazinamide for 2 to 6 months is necessary.

The Bacille Calmette-Guérin (BCG) is a live vaccine against tuberculosis (TB) that is indicated for some patients at an increased risk of TB. In this case, this patient already has TB and vaccination would not be effective against a pre-existing TB.

Learning Points
The Bacille Calmette-Guérin (BCG) is a live vaccine against tuberculosis (TB) that is indicated for some patients at an increased risk of TB.

169
Q

patient presents with shortness of breath on a background of end-stage renal disease (ESRD). She has missed her past three sessions of dialysis.

An ECG shows peaked T waves with prolonged PR intervals.

what first- dialiyse or calcium gluconate

A

Stabilisation of the cardiac membrane is the priority in hyperkalaemia with ECG changes

he stem describes a patient presenting with acute pulmonary oedema (APO) secondary to missed dialysis. Hyperkalaemia, suggested by these ECG changes, is a common electrolyte abnormality in patients who have missed dialysis. Management of severe hyperkalaemia with ECG changes typically begins with calcium gluconate, which stabilises the cardiac membrane to prevent arrhythmia.

Salbutamol may be used to drive potassium intracellularly, thereby reducing serum potassium. Other treatments for hyperkalaemia include insulin with dextrose and resonium.

Furosemide is an important medication in the treatment of APO and would likely be used in this scenario alongside other therapies until dialysis can be arranged. However, the immediate priority is to reduce the imminent risk of a cardiac complication of hyperkalaemia.

Ultimately, this patient will require haemodialysis to remove fluid, potassium and urea. However, this question asks for the most appropriate initial management, which would take place in the emergency department.

There is no chest pain and these ECG changes are not suggestive of an acute myocardial infarction (MI). As such, dual antiplatelet therapy (DAPT) with aspirin and clopidogrel is not indicated.

170
Q

VTE post surgery. What VTE and for how long?

A

duration of treatment is individualised based upon presence or absence of provoking events, risk of recurrence and bleeding

  • if provoked- 3 months
    • Provoking factors include:
      • Surgery
  • *Trauma**
  • *Imobilisation**
  • *Prolonged hospitalisation**
  • *Hormone replacement therapy**
  • *Oral contraception**
  • If provoked- 6-12 months

a first episode of a venous thromboembolism should have anticoagulation for a minimum of 3 months, this 3 month period to be the highest risk of recurrent thromboembolism.
Extending coagulation beyond this period is not considered if the venous thromboembolism is provoked with transient risk factors.

If the venous thromboembolism is unprovoked, then the evidence supports a longer duration of therapy of 6-12 months, while those with known malignancy may be anticoagulated indefinitely.

Changing anticoagulants during this 3 month period should be avoided due to the increased risk of bleeding and thromboembolism recurrence associated with interruptions in therapy.

This patient has had a provoked pulmonary embolism that has since resolved with 3 months of anticoagulation, so the warfarin should be ceased.

171
Q

Watery diarrhea. No other symptoms

A

This case describes travellers’s diarrhoea in someone who has gastroenteritis symptoms after returning from a south-east asian country. Most causes of acute travellers’ diarrhoea can be of bacterial, viral or parasitic origin and often a stool culture is required to identify the pathogen.

However, sometimes typical features from the history can suggest a causative organism too.

Watery stools in the absence of associated symptoms such as fever, abdominal pain or bloody stools is typical of an ETEC infection

whereas Campylobacter, Shigella and Salmonella infections often present with those associated symptoms.

Clostridioides difficile is not a typical causative organism of traveller’s diarrhoea and is often associated with prior antibiotic use.

Learning Points
Enterotoxigenic Escherichia coli (ETEC) is a common organism that causes traveller’s diarrhoea which typically presents with watery diarrhoea.

172
Q

nephrotic ayndrome- mx of hyperlipidemia

65-year-old male presents

ankle swelling and deranged biochemistry.

he underwent a renal biopsy, which was normal under light microscopy but there was effacement of the foot processes on electron microscopy.

Today, his blood results are significant for low albumin (20g/L) and high total cholesterol (9mmol/L). His urine dipstick is positive 4+ for protein.

Which one of the following is the most appropriate way to manage his high cholesterol initially?

A

Nephrotic syndrome caused by minimal change disease:

  • renal biopsy, which was normal under light microscopy
  • but there was effacement of the foot processes on electron microscopy. (MCD)
  • low albumin (20g/L) and high total cholesterol (9mmol/L). His urine dipstick is positive 4+ for protein. (nephrotic syndrome)

the most appropriate management of his high cholesterol is to treat the underlying condition. General approaches would be through fluid and salt restriction, the use of an ACE-I and in some cases steroids. Hence a trial of steroids may be used to treat his minimal change disease.

Learning Points

  • *Hyperlipidaemia in nephrotic syndrome** does not warrant lipid lowering therapy
  • *Steroids are often trialled** in the management of primary nephrotic syndrome, such as in minimal change disease

nephrotic syndrome, as demonstrated by

  • heavy proteinuria,
  • hypoalbuminaemia,
  • clinical oedema and
  • hyperlipidaemia.

In this case the cause of the nephrotic syndrome, as suggested by the biopsy results is minimal change disease. Suggested by the name, the pathological hallmark for this condition is the absence of changes under light microscope, but effacement of the foot processes under electron microscope.

It is important to recognise that the hyperlipidaemia in the context of nephrotic syndrome does not require lipid lowering medications and will not respond effectively to lifestyle changes.

The treatment requires treating the underlying nephrotic syndrome and heavy proteinuria, which is driving the hyperlipidaemia. There may be some role for these medications and lifestyle changes in chronic unremitting nephrotic syndrome, but in the acute phase of nephrotic syndrome the management of hyperlipidaemia centres around managing the cause of the nephrotic syndrome.

Therefore, starting him on atorvastatin or starting a fibrate would not be effective and are not recommended in this case. Exercise has been shown to increase HDL levels, and likely have a positive effect on cholesterol levels in the general population. But the underlying pathophysiology of the hyperlipidaemia is not driven by lifestyle factors in nephrotic syndrome.

Diet plays an important role in hyperlipidaemia, but the advice should be for patients to avoid saturated fats rather than unsaturated fats, which actually have positive impacts on health and lipids. Similarly, diet would be unlikely to influence hyperlipidaemia in nephrotic syndrome.

173
Q

ITP Mx and presentation

A

This patient’s unrelenting nosebleed, a history of previous episodes of similar nosebleeds, heavy menstrual bleeding, scattered ecchymosis, and isolated low platelet count are consistent with the diagnosis of immune thrombocytopaenia (ITP). ITP is an autoimmune condition characterised by the formation of IgG autoantibodies against platelets. This leads to an isolated decrease in platelet count and episodes of mucosal bleeding (eg, nosebleeds, heavy menstrual bleeding). These patients are susceptible to bruises and have characteristic scattered ecchymosis on their bodies. No hepatomegaly or splenomegaly is seen. Observation is indicated in asymptomatic ITP patients with only cutaneous symptoms (eg, purpura) and platelet count >30,000/mm3. This patient has bleeding, and her platelet count is <30,000/mm3, hence requiring active intervention. The most appropriate initial management for the haemodynamically stable patients diagnosed with symptomatic ITP is administration corticosteroids if there is symptomatic bleeding present or platelet count is <30,000/mm3. Intravenous immunoglobulin plus corticosteroids plus platelet transfusion is indicated in haemodynamically unstable patients with life-threatening bleeding. This patient is haemodynamically stable at this moment and does not require this therapy. Second-line treatment is further immunosuppression. For this, rituximab is favoured, but alternatives include azathioprine, mycophenolate and dapsone. Platelet transfusion is not recommended as any extrinsic platelet infused into the patient will likely be attacked by the existing anti-platelet antibodies and will render this treatment ineffective. If splenectomy is to be considered, it is usually deferred for at least 12 months after the diagnosis of ITP since many patients undergo spontaneous remission. It should be undertaken only in those patients who have not undergone spontaneous remission and are not showing any signs of improvement with the first-line treatment. Splenectomy is third-line treatment in Australia in patients who have failed the first-line treatment (eg, corticosteroids, IVIG) and second-line treatment.

174
Q

ITP Mx and presentation

A

This patient’s unrelenting nosebleed, a history of previous episodes of similar nosebleeds, heavy menstrual bleeding, scattered ecchymosis, and isolated low platelet count are consistent with the diagnosis of immune thrombocytopaenia (ITP). ITP is an autoimmune condition characterised by the formation of IgG autoantibodies against platelets. This leads to an isolated decrease in platelet count and episodes of mucosal bleeding (eg, nosebleeds, heavy menstrual bleeding). These patients are susceptible to bruises and have characteristic scattered ecchymosis on their bodies. No hepatomegaly or splenomegaly is seen. Observation is indicated in asymptomatic ITP patients with only cutaneous symptoms (eg, purpura) and platelet count >30,000/mm3. This patient has bleeding, and her platelet count is <30,000/mm3, hence requiring active intervention. The most appropriate initial management for the haemodynamically stable patients diagnosed with symptomatic ITP is administration corticosteroids if there is symptomatic bleeding present or platelet count is <30,000/mm3. Intravenous immunoglobulin plus corticosteroids plus platelet transfusion is indicated in haemodynamically unstable patients with life-threatening bleeding. This patient is haemodynamically stable at this moment and does not require this therapy. Second-line treatment is further immunosuppression. For this, rituximab is favoured, but alternatives include azathioprine, mycophenolate and dapsone. Platelet transfusion is not recommended as any extrinsic platelet infused into the patient will likely be attacked by the existing anti-platelet antibodies and will render this treatment ineffective. If splenectomy is to be considered, it is usually deferred for at least 12 months after the diagnosis of ITP since many patients undergo spontaneous remission. It should be undertaken only in those patients who have not undergone spontaneous remission and are not showing any signs of improvement with the first-line treatment. Splenectomy is third-line treatment in Australia in patients who have failed the first-line treatment (eg, corticosteroids, IVIG) and second-line treatment.

175
Q

ITP Mx and presentation

A

This patient’s unrelenting nosebleed, a history of previous episodes of similar nosebleeds, heavy menstrual bleeding, scattered ecchymosis, and isolated low platelet count are consistent with the diagnosis of immune thrombocytopaenia (ITP). ITP is an autoimmune condition characterised by the formation of IgG autoantibodies against platelets. This leads to an isolated decrease in platelet count and episodes of mucosal bleeding (eg, nosebleeds, heavy menstrual bleeding). These patients are susceptible to bruises and have characteristic scattered ecchymosis on their bodies. No hepatomegaly or splenomegaly is seen. Observation is indicated in asymptomatic ITP patients with only cutaneous symptoms (eg, purpura) and platelet count >30,000/mm3. This patient has bleeding, and her platelet count is <30,000/mm3, hence requiring active intervention. The most appropriate initial management for the haemodynamically stable patients diagnosed with symptomatic ITP is administration corticosteroids if there is symptomatic bleeding present or platelet count is <30,000/mm3. Intravenous immunoglobulin plus corticosteroids plus platelet transfusion is indicated in haemodynamically unstable patients with life-threatening bleeding. This patient is haemodynamically stable at this moment and does not require this therapy. Second-line treatment is further immunosuppression. For this, rituximab is favoured, but alternatives include azathioprine, mycophenolate and dapsone. Platelet transfusion is not recommended as any extrinsic platelet infused into the patient will likely be attacked by the existing anti-platelet antibodies and will render this treatment ineffective. If splenectomy is to be considered, it is usually deferred for at least 12 months after the diagnosis of ITP since many patients undergo spontaneous remission. It should be undertaken only in those patients who have not undergone spontaneous remission and are not showing any signs of improvement with the first-line treatment. Splenectomy is third-line treatment in Australia in patients who have failed the first-line treatment (eg, corticosteroids, IVIG) and second-line treatment.

176
Q

Meniers, Vestibular Nueritis, presbecuysis, VPA tumour. How distinguish

SNHL webber and Rinne?

58-year-old man presents with a 9 month history of recurrent, sudden episodes where he feels that the room is spinning around him. Each episode is accompanied by nausea and ringing in his left ear. He has also noticed that it has been increasingly difficult to hear lower-pitched sounds in the same ear. On examination, Weber’s test reveals lateralisation to the right ear and the Rinne test shows air conduction is greater than bone conduction in both ears.

Which one of the following is the most likely diagnosis?

A

SNHL- Webber lateralizes to the good ear (opposite ear has the SHL) . AC>BC in both ears (normal Rinne)

Meniers- Middle age. Tinnitis, vertigo, SNHL of low frequencies.

  • *Learning Points**
  • *Ménière disease is characterised by recurrent attacks of vertigo, associated with progressive sensorineural hearing loss, nausea, vomiting, tinnitus and aural fullness**

BPPV- no hearning loss/ tinnitus

Presbucusis- high frequency. No vertigo.

VPA tumour- ddx in Meniers disease- V+T+SNHL but also mass effect of tumour- CN 7 effects, ataxia etc. Mx with galidium enhaced MRI.

Vestibule Neuritis- younger. Complain of ear pain, vertigo and tinnitus. last 6-12 weeks max. No HL

Sensorineural hearing loss is established on the tuning fork tests, where lateralisation to the right ear indicates impaired sensorineural hearing function of the left ear. A normal Rinne test indicates normal conductive hearing bilaterally. Lower frequencies tend to be affected initially.

Ménière disease commonly occurs in middle-aged individuals. It is characterised by recurrent attacks of vertigo, associated with progressive sensorineural hearing loss, nausea, vomiting, tinnitus and aural fullness. It may sometimes present with horizontal nystagmus.

Vestibular neuritis is more common in younger patients. Although it may also present with vertigo, it is not associated with progressive hearing loss. Symptoms usually only last 6-12 weeks and patients often complain of ear pain.
Benign paroxysmal positional vertigo is not associated with hearing loss.
Cerebellopontine angle tumours (such as acoustic neuroma) can present similarly to Ménière disease - unilateral sensorineural hearing loss, tinnitus, vertigo. However, these patients may present with unsteady gait and symptoms of other cranial nerve disturbances, such as facial numbness or weakness. Nevertheless, in this case it would be essential to rule out a diagnosis of malignancy by performing a gadolinium-enhanced MRI of the brain and internal acoustic meatus and this should be undertaken in patients with new onset sensorineural hearing loss.
Although presbycusis (age-related hearing loss) is sensorineural in nature, it is not associated with recurrent vertigo attacks. It also differs to Ménière disease in that it initially affects the higher frequencies, rather than lower frequencies.

177
Q

ddx of a wet umbilical stump

6-month year old boy is brought by his parents, concerned about a lump on his abdomen. This has been present since birth and his parents think that it might be getting larger. They have also noticed that there has been intermittent wetness around the lump.

His mother’s pregnancy was uncomplicated (all antenatal investigations during pregnancy were noted to be normal, including morphological scans).

Q2
7-year-old boy is brought to the Emergency Department with a one day history of central, colicky abdominal pain, relieved by vomiting. His parents report he has not passed stool or wind for 24 hours. He is normally well with no medical or surgical history. On examination his abdomen is distended, but soft. There are hyperactive bowel sounds. An X-ray confirms small bowel obstruction. His condition does not improve with conservative management and at laparoscopy, a fibrous band is found running from the umbilicus to ileal mesentery, through which a small bowel loop has herniated.

Which one of the following is the most likely underlying problem?
Choose the single best answer.

Patent processus vaginalis
Omphalomesenteric duct remnant
Gastroschisis
Omphalocele
Idiopathic intussusception

A

This swelling is a partially patent urachus sinus. The urachus normally closes shortly before birth and urachal abnormalities are not uncommon. The abnormalities include cyst, sinus and fistula. If the urachus remains patent or partially patent, there will be a persistent or intermittent discharge of urine – as occurred with this child.

A patent vitello-intestinal duct (also known as the omphalomesenteric duct) is extremely rare and it is very unlikely an infant with such a defect would survive into childhood without surgical correction. It can present with umbilical drainage more consistent with bowel contents.A fibrous band running from the umbilicus to the ileal mesentery is likely the remnant of the omphalomesenteric duct (vitelline), which connects the primitive gut to yolk sac and is obliterated by the eighth week of gestation. This fibrous band would be connected to a Meckel’s diverticulum in the ileum, situated some 45 - 90 cm from the ileocaecal valve. This band may cause entrapment, volvulus or intussusception of small bowel.

An umbilical hernia should be covered by skin, and increase in size with increased intra-abdominal pressure (such as crying, coughing, straining), and can potentially be reduced. An umbilical hernia also does not discharge any fluid.

A strawberry naevus and umbilical granuloma would not discharge fluid, and typically do not have the same appearance as the defect in the image. An umbilical granuloma typically results from a low-grade infection of the umbilical stump, and presents soon after cord separation as red, friable granulation tissue.

2 This presentation of central, colicky abdominal pain, relieved by vomiting, with distension, suggests small bowel obstruction. A fibrous band running from the umbilicus to the ileal mesentery is likely the remnant of the omphalomesenteric duct (vitelline), which connects the primitive gut to yolk sac and is obliterated by the eighth week of gestation. This fibrous band would be connected to a Meckel’s diverticulum in the ileum, situated some 45 - 90 cm from the ileocaecal valve. This band may cause entrapment, volvulus or intussusception of small bowel.

Small gut entrapment through a patent processus vaginalis would be an incarcerated indirect inguinal hernia, the commonest hernia of childhood.

Gastroschisis (RIGHT OF UMBILICUS, NOT COVERED BY PERITONEUM. ONLY DUO ATREASIA) and omphaloceles (THROUGH UMBILICUS, COVERED BY PERITONEUM, ASS WITH TRISOMY 18,21, WIED-BECKWORTH -ALL THINGS BIG BABY, CARDIAC ISSUES) are neonatal abdominal wall defects through which small and large bowel protrude.

178
Q

CRC AND SCREENING

CRC

Colorectal screening is individualised based on individual risk of colorectal cancer.

Risk is classified as

  • average or slightly above average risk (95-95% of the population),
  • moderately increased risk (2-5% of the population)
  • or high risk (<1% of the population).

EG 1

This boy’s family history (3 first or second degree relatives diagnosed with colorectal cancer with at least one diagnosed <55 years old)

EG 2

53-year-old woman presents seeking advice on screening for bowel cancer. She has no symptoms, but is concerned since her father was diagnosed with carcinoma of the rectum at the age of 51. She has never had a faecal occult blood test (FOBT).

Which one of the following is the most appropriate advice?
Choose the single best answer.

  • Annual FOBT starting at the age of 55, followed by 5-year colonoscopy
  • Annual FOBT starting now and colonoscopy when she is 55
  • Colonoscopy now
  • Colonoscopy at the age of 55
  • FOBT now, followed by a colonoscopy, if positive
A

EG 1:

For most people with average risk of colorectal cancer the screening guidelines are faecal occult blood test ever 2 years from 50-74.
This boy’s family history (3 first or second degree relatives diagnosed with colorectal cancer with at least one diagnosed <55 years old) places him in the high risk category for colorectal cancer. He should be referred for genetic testing to test for high risk syndromes for colorectal cancer e.g. Lynch Syndrome or familial polyposis syndrome and his surveillance schedule should be determined by specialist advice after results of his genetic testing.

EG 2

In terms of risk stratification, this patient is a Category II or moderate risk. With a first-degree relative having been diagnosed with bowel cancer before the age of 55, she should have started two-yearly faecal occult blood testing from the age of 40 and a colonoscopy every five years from the age of 50. Thus she should be counselled to have a colonoscopy now.

Learning Points

  • iFOB from 40-49 years. For 2 years.
  • Colonoscopy should be considered for moderate risk patients over the age of 50.
  • Low dose Asprin (100mg-300mg) from 50 for at least 2.5 years
179
Q

What is classified as average risk of CRC, Mx of average risk CRC

A
  1. Asymptomatic- with no RF for CRC (IBD/colonic adenomas/no personal or fam hx of CRC)
  2. Asymptomatic with a 1st or 1st + 2nd deg relative dx with colon cancer after 55 years of age

Mx 50-74: NO colonoscopy. Low dose asprin (100-300mg for) for at least 2.5 years from 50-70 Asprin

180
Q

CRC medium risk mx

CRC medium risk and Mx

A

who:

  • 1 relative (1st degree) dx with CRC before 55
  • or any 1st degree dx with CRC at any age
  • 1st and 2nd dx with CRC at any age

How Mx:

  • 4O-49 iFOBT every 2 years
  • 50 onwards colonoscopy every 5 years
181
Q

High risk CRC with Mx

A

Who-

3 first degree or second degree relatives with dx with CRC before 55
3 first degree relative dx with CRC at any age

High risk features CRC before age 50. Multiple CRC in one person.

FAP (1 relative),
Lynch sydrome or
Lynch related syndromes ( 2 or more 1st/2nd degree)

Lynch related cancers- EGOUSS
endometrial, gastric, ovarian, urothelial, skin, small bowel.

Mx

Familial cancer assesment with genetic cancer risk assesment

35-44 iFOBT 2 yearly
45-74 Colonoscopy

182
Q

54-year-old man with documented hypertension is reviewed as part of ongoing assessment. For the last three months he has been on hydrochlorothiazide 25 mg daily, in addition to stopping smoking, regular exercise and losing weight. His home blood pressure monitoring averages 162/100 mmHg.

His only other medication is atorvastatin for hyperlipidaemia. No secondary causes of hypertension have been identified. No abnormalities are found on physical examination, aside from a blood pressure of 160/98 mmHg and a BMI of 26. His current biochemical profile is within normal limits.

In addition to ongoing dietary advice, which one of the following would be the most appropriate next step in management?
Choose the single best answer.

  • Increase dose of hydrochlorothiazide
    Start atenolol
    Start bisoprolol
    Start perindopril
    Start clonidine
A

Lifestyle advice including not smoking, eating a nutritious diet and regular exercise is recommended for all patients. For patients at a low absolute cardiovascular risk (5-year risk <10%) and a persistent blood pressure >160/100 mmHg, anti-hypertensive therapy should be started. For patients with a moderate absolute cardiovascular risk (5-year risk 10-15%) and a persistent blood pressure >140/90 mmHg, anti-hypertensive therapy should be started.

Blood pressure for patients with uncomplicated hypertension should be targeted at <140/90 mmHg with current evidence suggesting lower targets, if tolerated, may confer improved cardiovascular outcomes, especially in those with known cardiovascular disease.

In patients with uncomplicated hypertension, angiotensin-converting enzyme (ACE) inhbitors or angiotensin-receptor blockers (ARBs), calcium channel blockers and thiazide diuretics are all suitable first line anti-hypertensive medications. They may be used as monotherapy, or if the patient is non-responsive to the initial agent, in combination unless contraindicated. Of the options provided, perindopril is the most suitable choice of medication for this patient.

The balance between efficacy and safety for beta-blockers is less favourable than other agents and are not recommended as primary agents in uncomplicated hypertension, but can be used in cases which are complicated with myocardial infarction or angina.

If, after three months of antihypertensive therapy, the target for blood pressure control is not reached, a second drug from a different pharmacological dose should be added to the regimen. This can be given at a relatively low dose, providing good antihypertensive therapy and reduces the risk of adverse effects which could be brought on by increasing the dose of a single agent.

  • *Learning Points**
  • *If, after three months of antihypertensive therapy for uncomplicated hypertension, the target for blood pressure control is not reached, a second drug from a different pharmacological dose should be added to the regimen.**
183
Q

Osteoporosis diagnosis

what reading- t or z score

what is opteopeania range what is osteoporosis range

70 year old. Her DEXA scan reveals a Z-score of -1.5 and T-score of -2.7. Her blood tests are as follows: calcium 2.30 mmol/L (Normal: 2.10 - 2.60 mmol/L), creatinine 90 µmol/L (Normal: 45- 90 µmol/L), phosphate 1.0 mmol/L (Normal: 0.75- 1.50 mmol/L), parathyroid hormone 50 ng/L (Normal: 14-65 ng/L), 25-Hydroxy vitamin D 80 nmol/L (Normal: ≥ 50 nmol/L), and estimated glomerular filtration rate 70 mL/min/1.73 m^2.

A
  • Diagnosis of oteoporosis/peania is based on t score
  • Osteopeania- T score of -1 to -2.5–low bone mass
  • Osteoporosis T score less than -2.5- low BMD
  • T score of more than -1 is considered as normal
  • Osteoporosis can be diagnosed when bone mineral density T-score is <-2.5 on DEXA bone scan.

Do not confuse T-score and Z-score

  • DEXA T-score: Comparison of a person’s bone density with a healthy 30-year old person of the same SEX and ETHNICITY.
  • DEXA Z-score: Comparison of a person’s bone density with an average person of the same AGE, SEX and ETHNICITY. (used in individuals that are less than 50)
184
Q

bone conditions

pagets and osteomalacia- how to distinguish

A

Paget’s disease is a disease that causes chronic enlargement and weakening of bones secondary to increased and uncontrolled bone turnover with excessive osteoclastic resorption followed by increased osteoblastic activity. In this condition, bone mineral density generally increases, (resulting in an increased T-score), thus, it is unlikely that this patient has Paget’s disease.

X-ray

  • Deformed bones with both sclerotic and osteolytic lesions
    • Sclerotic lesion: focus/foci of ↑ density (usually the result of increased mineralization and/or thickening).
    • Osteolytic lesion: focus/foci of ↓ density (usually the result of decreased demineralization).
  • Thickened cortical bone
    • Coarsened trabeculae; expansion or enlargement of a region of the bone.
    • Skull x-ray: thickening of the diploe; osteoporosis circumscripta (cotton wool appearance).
    • Vertebral x-ray: thickening of the upper and lower plates of the vertebral body gives rise to a “picture frame” appearance; diffuse enlargement of the vertebrae (ivory vertebra)

Osteomalacia is defined as a decreased bone mineral content secondary to low vitamin D levels. Our patient does has a normal vitamin D level which can be ruled out in this patient.

185
Q

assessing his risk of suicide, which features in presentation is most concerning?ie Greatest risk factor

A
  • Acute stressor, on a background of longstanding depression are often present risk factors.
  • single biggest risk factor for a future suicide attempt is a previous suicide attempt.
  • expressing a wish to die,
  • disengagement from mental health services,
  • medication non-compliance,
  • increased alcohol usage
  • age/ gender

In the 10-44 age group, suicide is in the top three causes of death worldwide, with the highest rates occurring in males aged 30-59.

Despite all of these, the single biggest risk factor for a future suicide attempt is a previous suicide attempt. Any patient with a previous history of suicide or self-harm should be taken seriously and thoroughly assessed before being declared “safe”.

185
Q

assessing his risk of suicide, which features in presentation is most concerning?ie Greatest risk factor

A
  • Acute stressor, on a background of longstanding depression are often present risk factors.
  • single biggest risk factor for a future suicide attempt is a previous suicide attempt.
  • expressing a wish to die,
  • disengagement from mental health services,
  • medication non-compliance,
  • increased alcohol usage
  • age/ gender

In the 10-44 age group, suicide is in the top three causes of death worldwide, with the highest rates occurring in males aged 30-59.

Despite all of these, the single biggest risk factor for a future suicide attempt is a previous suicide attempt. Any patient with a previous history of suicide or self-harm should be taken seriously and thoroughly assessed before being declared “safe”.

186
Q

Medical board reporting

supervising surgeon arriving to start his morning operating list smelling of alcohol. This is now happening on a regular basis. The nurses that work with the surgeon have also raised concerns. The surgeon appears to be able to operate and get through the list without any problems, but the registrar comments that he often makes inappropriate comments on various topics that are out of character for the surgeon.

Upon direct questioning about this issue from her supervisor, the trainee admits to their actions in creating the forgeries, but says that they did not want to bother the surgeon to get their signature.

A

EG1
This comes under the category of mandatory reporting. Perhaps if it had only been noticed on one occasion, the issue might have been ignored; however, if this is occurring on a regular basis and is being noticed by multiple medical professionals. The registrar and the other health professionals who have noticed this behaviour have a duty to report to the Medical Board, particularly as the surgeon is showing signs of being intoxicated during surgical procedures. The report to the Medical Board must be based on reasonable evidence (not hearsay) that there is a potential risk to the public. It would also be wise for the registrar to approach their medical indemnity insurer for advice prior to reporting to the medical board.

EG 2

The trainee has admitted to forgery. Forgery in such circumstances demands a report to the Medical Board.The medical board states that “if a practitioner’s practice shows a significant departure from accepted professional standards that places the public at risk of harm, it can trigger a mandatory notification”.

187
Q

Misscarriage

at 9 weeks gestation, presents to the hospital 3 hours after the onset of vaginal bleeding and abdominal pain. She reports bleeding several large clots associated with cramping lower abdominal pain. Her pain was 7/10 in intensity but now as reduced to 3/10, and she has no current active bleeding.

Pelvic examination revealed traces of blood and clots surrounding her vagina, on speculum the cervix is closed and there is no active vaginal bleeding. Pelvic ultrasound shows a small amount of fluid in the endometrium and an absent gestational sac with no adnexal masses or free fluid in the abdomen. There are no suggestive findings of an ectopic pregnancy.

This patient presents with crampy abdominal pain and vaginal bleeding/discharge which supports the diagnosis of miscarriage. The improvement of abdominal pain after vaginal bleeding/discharge could indicate that the spontaneous passage of tissue has relieved the pain.

A

Closed Os

complete,
missed,
threatened

open os-

incomplete
inevitable

A pelvic ultrasound is necessary to ascertain the status and diagnosis of miscarriage as well as rule out an ectopic pregnancy which must be a differential in a woman with pain, bleeding and ultrasound findings of an empty uterus.

In this patient, the cervical os is closed and her uterus is empty – this would suggest a complete miscarriage and passage of tissue.

  • *Missed miscarriage is defined as pregnancy loss with minimal to no symptoms.** The cervical os will also be closed and there will be an empty gestational sac or no fetal cardiac activity in a missed miscarriage.
  • *Th**is patient noted the passage of tissue with painful bleeding which is not in keeping with a missed miscarriage.

An incomplete miscarriage would present similarly to a complete miscarriage. However, the cervical os often remains open and there would be remnants of tissue within the uterus. Ultrasound may also reveal an empty uterus depending on how much tissue remains in the uterus.

Threatened miscarriage presents with vaginal bleeding and a closed cervical os. The ultrasound would reveal a live intrauterine pregnancy.

An inevitable miscarriage is similar to a threatened miscarriage, however, the cervical os is open indicating that a miscarriage is imminent. A fetal heart rate (FHR) may or may not be present on ultrasound.

REFERENCES

188
Q

2-year-old child of South East Asian descent presents to the emergency department with a fever and mild respiratory symptoms, and looks unwell. Skin changes are noted over the lower back, buttocks and lower limbs. There are multiple lesions which are a dark red/purple colour, palpable, and non-blanching.

On further history, these lesions were not present last week, and since they were first noticed by the parents they have progressed.

A

Meningococcal disease needs to be seriously considered in this presentation of an infant presenting with a fever and new onset of petechiae / purpura (non blanching rash). This rash presents in 80% of those with invasive meningococcal disease. The progression of the rash with a fever leads to this being the most likely diagnosis, and requires urgent intervention with IV access, blood cultures, lumbar puncture and urgent intravenous antibiotics.

Mongolian blue spots (dermal melanocytosis) occur commonly over the sacral region, buttocks, and lower limbs. They are most common in African-America, Asian and Hispanic infants. They usually fade within the first few years of life. The characteristic appearance and congenital onset distinguish these spots from the bruises of child abuse.

Non accidental injury needs to be considered in all children presenting with lesions that could be bruising, and may need further investigation once the child is stabilised and further investigations return.

Henoch-Schönlein purpura (HSP) is the most common small vessel vasculitis in childhood and is characterised by a purpuric rash (which is often palpable), abdominal pain and arthritis, and 50% present with renal manifestations.

Immune thrombocytopenic purpura occurs in 1 in 20,000 children after exposure to a common viral infection. This would need to be considered, and platelet results would assist with the diagnosis; however in this situation meningococcal disease would be more likely with the clinical situation.

Learning Points
There are many causes of petechiae in infants, and serious bacterial infections always need to be considered
Non accidental injury needs to be considered in all children presenting with any concerning lesions such as bruising that cannot be explained.

189
Q

ILD vs Obstruction

he focussed respiratory examination demonstrated a suggestion of some fine crackles. The GP referred the patient to pulmonary function tests within the next week.

The pulmonary function tests are conducted and compared as a percentage of the expected reference range with the results below:

  • *FEV1:** 2.55L (3.59 predicted) 70% (Ref: ≥ 80%)
  • *FVC:** 70% (Ref: ≥ 80%)
  • *FEV1/FVC: ≥92%** (Ref: 70-80%)
  • *DLCO: ≥70%** (Ref: ≥ 80%)

Which one of the following is the most likely diagnosis?

Asthma, ILD, COPD

A

When interpreting the PFTS, the key findings to note for ILD include a decreased FEV1, a decreased FVC, normal or increased FEV1/FVC ratio and in particular a decreased DLCO.

Although asthma is a possible diagnosis for the clinical presentation alone, patients with asthma do not present with a decreased DLCO and in some cases show an increased DLCO.

COPD may present with decreased DLCO due to decreased surface area in the alveoli. However, in patients with disease severe enough to affect the DLCO, it is likely the FEV1 and FEV1/FVC ratio would be significantly lower than the findings in the stem.

190
Q

obstructive vs restrictive Lung Fnctn Tests

A

Obstructive

FEV1/FVC is reduced ( where Fev1 is significantly reduced compared to FVC-→ overall decrease) <70 (Ref: 70-80%)

FEV1 less than 80

FVC less than 80

DLCO-→ increased ( if decreased– sever decline in all Lung Fnctn Tests)

Restrictive picture

FEV1/FVC is increased or Normal ( FEV1 and FVC are reduced proportionally) (Ref: 70-80%)

FEV1 (n/dec)

FVC ( decreased)

DLCO-→ ( decreased) eg DLCO: ≥70% (Ref: ≥ 80%)

191
Q

MS imaging

MS investigations

A

CSF sampling via lumbar puncture with analysis for oligoclonal bands is among the ancillary tests available to investigate multiple sclerosis - multiple features of this presentation would not be typical, and MRI is first-line to assess suspected multiple sclerosis.

192
Q

Macrocytic anaemia

She is a strict vegetarian, and consumes 1-2 glasses of wine most nights. Physical examination reveals loss of vibration sensation and proprioception in both lower limbs. A complete blood examination shows a macrocytic anaemia and a hypersegmented neutrophil is noted on the peripheral blood smear. Renal and liver function tests are normal.

Which one of the following is the next most appropriate initial investigation?

A

This is a clear description of a case of vitamin B12 deficiency, both in terms of the clinical picture and the laboratory findings. The anaemia would explain her tiredness and the neurological observations would suggest subacute combined degeneration of the spinal cord. There are a number of different causes for vitamin B12 deficiency and in this case it is most likely to be a dietary deficiency. However, the current commonest cause of vitamin B12 deficiency is ‘food-bound cobalamine malabsorption.’ This is the failure of release of vitamin B12 from its transport proteins and is seen typically in hypochlorhydric individuals - that is, those on proton pump inhibitors or after gastric resection/bypass.

Among the diagnostic options, the serum vitamin B12 level is the best investigation, providing direct evidence of deficiency and confirming the diagnosis with approximately 95% sensitivity in symptomatic individuals. Once B12 deficiency is confirmed, further testing for underlying causes (e.g. anti-intrinsic factor or anti-parietal cell antibodies for pernicious anaemia) could be considered.

In pernicious anaemia, endoscopy would likely reveal atrophic gastritis, but would not be the first line investigation in the case presented.
MRI spine might demonstrate the characteristic ‘inverted V’ sign of high T2 signal in the posterior column associated with subacute combined degeneration, however is unnecessary for preliminary diagnosis.

193
Q

Anticoagulant

On examination, she is found to have an irregularly irregular pulse, a loud S1 with an opening snap, and a rumbling mid-diastolic murmur heard best in expiration with the patient in the left lateral decubitus position. An ECG is performed and confirms newly-identified atrial fibrillation. Consideration is given as to whether the patient requires any medications to reduce her long-term risk of thromboembolism.

A

Warfarin is the anticoagulant of choice in ***valvular*** AF

This patient has been newly diagnosed with atrial fibrillation (AF) in the context of a history of rheumatic heart disease and findings consistent with mitral stenosis on auscultation; this patient therefore has ‘valvular AF’.

Valvular AF is commonly defined as AF arising either in the setting of haemodynamically significant mitral stenosis or the presence of mechanical prosthetic heart valve(s), because each of these factors confers a significantly greater risk of thromboembolism over ‘non-valvular AF’. As a result, anticoagulation is essentially always indicated in valvular AF.

Warfarin is the anticoagulant of choice in valvular AF, including for our patient in this case, due to the lack of available data supporting the use of novel oral anticoagulants (NOACs; e.g. dabigatran, apixaban, rivaroxaban) in this condition

194
Q

dyspnoea, dry cough and reduced exercise tolerance. These symptoms have been present for two years and have been slowly getting worse. He is known to have asthma, which is managed with a daily low dosed inhaled corticosteroid, which he only takes occasionally.
On examination, there is clubbing of his fingers, reduced chest expansion, and crepitations at the lung bases. There is no wheeze. A chest X-ray shows a non-specific reticulonodular pattern.

Which one of the following is the most likely aetiology for this presentation?
Choose the single best answer.

Idiopathic
Drug-related
Hypersensitivity
Sarcoidosis
Pulmonary neoplasm

A

Idiopathic pulmonary fibrosis (IPF) is a chronic and progressive form of interstitial lung disease. IPF primarily occurs in those older than 50 years, occurring most commonly in patients aged 60 to 70 years old. The pathophysiology of IPF is unknown, but various causes have been suggested in literature including genetic susceptibility, environmental and occupational exposure, smoking, GORD and viral infections, leading to repetitive micro-injury to the lung tissue and vasculature, resulting in inflammation and fibrosis.

Symptoms of IPF are non-specific, but include slow, progressive worsening of lung function, such as dyspnoea, cough and reduced exercise tolerance. Physical examination findings are also non-specific but may include bibasilar mid- to end- inspiratory crackles, end-expiratory squeaks due to bronchiectasis and clubbing of the fingernails in advanced disease.

Diagnosis of IPF is based on clinical and radiological findings and pulmonary function testing. All patients with suspicion of IPF should undergo high resolution CT (HRCT) which characteristically reveals reticular opacities, traction bronchiectasis, honeycombing and ground-glass opacities. Pulmonary function testing will show a restrictive picture with impaired diffusion (reduced DLCO).

Despite current advances, IPF remains a fatal disease with a median survival of 2-3 years following diagnosis. Non-pharmacological management of IPF includes supplemental oxygen, pulmonary rehabilitation, mechanical ventilation and involvement of palliative care services. The pharmacological management of IPF is an evolving area, with induction using corticosteroids, and maintenance therapy using anti-fibrotic and anti-inflammatory agents such as pirfenidone and nintedanib, respectively. Lung transplant is also an option in patients with moderate to severe IPF; the ideal time to transplant is a conversation with much controversy, but the general consensus is to transplant earlier in the disease course.

Learning Points
Idiopathic pulmonary fibrosis is a common cause of interstitial lung disease in adults over 50

195
Q

Mature minor - COCP

A

There is no need to contact the patient’s parents or caregivers if the concept of a mature minor can be or has been established (also known as Gillick Competency).

This involves:

Understand choices and consequences of these choices
Are willing and able to make a choice
Understand nature and purpose of a treatment
Understand risks and benefits of a treatment
Understand alternative options
In a setting that is free from pressure

The Royal Australian and New Zealand College of Obstetricians and Gynaecologists (RANZCOG) supports the legal concept of the mature minor.

196
Q

Contraception- missed

2 pills missed in the first week of cycle . No intercourse in week.

what should be done

A

first week missed- need full 7 days to take effect. So 7 days of barrier protection. Dont need emergency contraception- no sex had. Take last missed pill.

The COCP takes 7 days of continuous use before becoming effective. The stem explains how Mary has missed two days of the COCP (48 hours). Although missing a single pill is unlikely to alter the contraceptive effect, missing the COCP for 48 hours may. As a result, ‘missing two pills may mean that ovulation is not suppressed, so she will require 7 days of barrier contraception before the COCP becomes effective again’ is the correct answer.

Furthermore, ‘the contraceptive effect of the pill will not be altered if she has only missed two pills, no further is action required’ is the incorrect answer. Mary’s ovulation may not have been suppressed, and thus further contraceptive action is required to ensure that she does not become pregnant before the COCP becomes effective in 7 days.

The option ‘if Mary takes the two pills that she missed immediately, she will not require any further contraception’ is incorrect. If a pill is missed, it is recommended that the most recent pill be taken, but not that all missed pills should be taken at once. If 2 or more pills are missed, Mary will still require barrier contraception for 7 days.

The option that ‘Mary should continue taking the COCP as directed, but skip the placebo tablets and replace with active tablets’ is incorrect. Whilst ‘continuous use regimens’ are becoming more popular, this particular strategy will not prevent Mary from falling pregnant in the following 7 days before the COCP becomes effective again.

The option that ‘Mary should continue with the COCP as directed, and also use an emergency contraceptive method as she may be pregnant’ is an incorrect answer. The stem says that Mary has not had sexual intercourse in the last 7 days. Thus, it is highly unlikely that she would have fallen pregnant before she had missed the pills. The issue with this scenario is not that Mary could be pregnant, as this is unlikely, but more that she could potentially fall pregnant if a backup contraception option is not used for the next 7 days (e.g. condoms).

Learning Points

  • The COCP takes 7 days of daily use before it becomes effective
  • Missing a single pill is unlikely to alter the contraceptive effective of the COCP
  • Missing two or more pills in a row may cause ovulation to occur, and thus a backup form of contraception must be used for the following 7 days until the COCP becomes effective again
197
Q

missed COCP

16-year-old girl presents concerned as she has “missed the pill”. Her last menstrual period was three weeks ago. She had intercourse 48 hours ago and missed her regular oral contraceptive pill the following two mornings. She was compliant prior to this.

Which one of the following is the most appropriate advice

A

As this patient is in the final week of her pill cycle before taking inactive pills, 2 missed pills triggers the ‘7 day rule’: she requires 7 days of hormonal contraception with additional barrier precautions during this time.
As her contraception was still effective at the time of intercourse, she does not require emergency contraception in this instance.
If barrier protection fails during this time, further emergency contraception is indicated.
Up to 8 pills can be missed outside of the first or last week of the pill cycle with effective contraception maintained, but as this patient is in the last week of her active pills she must continual to take the active pills and effectively skip her ‘period’ this cycle.

Up to 8 pills can be missed outside of the first or last week of the pill cycle with effective contraception maintained. As her contraception was still effective at the time of intercourse, she does not require emergency contraception in this instance.

198
Q

anaphylaxis

1:1000 adrenaline (1 mg per 1ml) vs 1: 10 000 (1mg per 10ml)

A

anaphylaxis in kids
10mcg/kg adrenaline IM

in kids 0,01 mg per Kg
max dose: 0.5 mg per dose (max 500mcg)

  • 1:1000 : 1mg adrenaline per 1ml
  • 10 mcg / 0.01 mg per 0.01 ml
  • 1:10 000 means 1mg per 10ml
  • 0,1 mg per 1 ml-→ 0.01mg per 0.1ml / 10mcg per 0.1 ml

eg if kid weighs 15:

  • *15 x 10mcg = 150 mcg ( 0.15 ml of 1:1000)**
  • *15 x 10mcg = 150mcg (1.5 ml of 1;10 000)**

Therefore, if this child weighed 15kg, the correct adrenaline dose to administer would be 150mcg, meaning 0.15mL of 1:1000 adrenaline or 1.5mL of 1:10000 adrenaline.

199
Q

anaphylaxis

signs and Mx of anaphylaxis

A

This question requires identification of the fact that this child is suffering an acute episode of anaphylaxis.

Features in the stem that support this diagnosis include

(1) rapid-onset shortness of breath (food is a very common trigger);
(2) inspiratory stridor;
(3) reduced oxygen saturations;
(4) angioedema; and
(5) an urticarial rash.

Adrenaline is the most critical aspect of management and early administration of this drug has been consistently shown to improve outcomes. This makes 10mcg/kg adrenaline IM the correct answer. The standard initial dose is 10mcg/kg. Adrenaline is generally administered via the intramuscular route. In some cases, repeated doses of IM adrenaline may be required at 5 minutely intervals. If 2-3 doses are required due to ongoing anaphylaxis, IV adrenaline infusion should be considered as the next step in management. Whilst there may be a role for adjunctive therapies as discussed below (for example, nebulised adrenaline, normal saline boluses etc.), adrenaline needs to be at the centre of the management plan

A common point of confusion among students is the difference between 1:1000 adrenaline and 1:10000 adrenaline. The 1:1000 preparation refers to 1mg of adrenaline per 1mL of solution, whereas the 1:10000 preparation corresponds to 1mg of adrenaline per 10mL of solution. Therefore, if this child weighed 15kg, the correct adrenaline dose to administer would be 150mcg, meaning 0.15mL of 1:1000 adrenaline or 1.5mL of 1:10000 adrenaline.

200
Q

Parkinson’s disease

PD drugs

A

Dopamine precursors

  1. L-DOPA (levodopa)

Decarboxylase inhibitors

  1. Carbidopa and Benserazide

Predominantly controls bradykinetic symptoms

First-line treatment for patients > 65 years of age or patients with comorbidities
Second-line treatment for patients < 65 years of age (treatment with L-DOPA is avoided as long as possible to prevent early onset of side effects)

Dopamine agonists: Act directly at striatal dopamine receptors
Predominantly control bradykinetic symptoms

  1. Non-ergot
    Ropinirole
    Pramipexole
    Apomorphine
    Rotigotine

First-line treatment for patients < 65 years of age
Adjunctive treatment for patients of any age
First-line treatment for restless leg syndrome

Less effective than L-DOPA, but fewer motor side effects
Effective in advanced disease stages- Unlike levodopa, dopamine agonists do not require presynaptic decarboxylation, which the damaged neurons in advanced disease can no longer achieve.

  1. Ergot
    Bromocriptine

Second-line treatment for patients < 70 years of age

COMPT inhibitors

Entacapone:
If L-DOPA loses effect or motor symptoms fluctuate during L-DOPA therapy
Tolcapone:
for refractory Parkinson disease

COMT inhibitor monotherapy is ineffective; therefore, it should always be combined with L-DOPA and carbidopa.
Inhibition of COMT reduces catabolism of levodopa, which increases drug availability, prolongs its action, and reduces motor fluctuations.

NMDA antagonists
Amantadine

Short-term treatment of mild symptoms (QT issues)
Drug of choice during akinetic crisis
Reduction of L-DOPA-induced dyskinesia

- - - -

MAO-B inhibitors
Rasagiline
Safinamide
Selegiline

Can also be given in combination with L-DOPA → ↓ motor fluctuations
Alternative to L-DOPA or dopamine agonists for short-term therapy (only has a mild motor effect) in patients with mild symptoms in early disease stages

- - - -

Anticholinergic drugs (muscarinic antagonists)
Benztropine
Trihexyphenidyl
Biperiden

  • *Useful as monotherapy in patients < 65 years of age with tremor as the main symptom**
  • *Beneficial regarding tremor and rigidity but does not improve bradykinesia**
  • *Generally avoided in elderly- anticholenergic effects**
201
Q

A 58-year-old woman comes to the physician because of a 6-month history of difficulty walking, clumsiness of her arms and legs, and slurred speech. Physical examination shows masked facies and a slow, shuffling gait. When her ankles are passively flexed, there is involuntary, jerky resistance.

Treatment is initiated with a combination of levodopa and carbidopa. The addition of carbidopa is most likely to decrease the risk of which of the following potential adverse drug effects?

  • *Resting tremor**
  • *Orthostatic hypotension**
  • *Urinary retention**
  • *Visual hallucinations**
  • *Dyskinesia**
A

the question is asking about the reduction of side effects of increased peripheral levels of L-Dopa. Central side effects are

Carbidopa is a peripheral decarboxylase inhibitor, which decreases the peripheral conversion of L-DOPA to dopamine and, thus, increases the amount of L-DOPA that enters the central nervous system.

Following administration, L-DOPA is absorbed into the systemic circulation where it is partially converted to dopamine by the enzyme DOPA decarboxylase. The remaining L-DOPA is then transported across the blood-brain barrier and enters the central nervous system to produce the desired effects. Carbidopa reduces the peripheral (systemic) conversion of L-DOPA to dopamine, thereby mitigating the peripheral sides effects, including orthostatic hypotension, nausea, and vomiting. Although the combination of carbidopa-levodopa significantly improves drug compliance, many patients still experience some degree of these side effects.

Visual hallucinations are a central side effect of L-DOPA therapy and they correlate to the concentration of dopamine in the central nervous system. By decreasing the peripheral conversion of L-DOPA to dopamine, carbidopa increases the fraction of the administered drug that crosses the blood-brain barrier. Therefore, visual hallucinations would be expected to increase rather than decrease.

Dyskinesia is a central nervous system side effect of L-DOPA that typically occurs with long-term therapy. By decreasing the peripheral conversion of L-DOPA to dopamine, carbidopa increases the fraction of the administered drug that crosses the blood-brain barrier. Therefore, symptoms of dyskinesia would be expected to increase rather than decrease.

Resting tremor is reduced by L-DOPA therapy. The addition of carbidopa is likely to further reduce the incidence of resting tremor because a combination of these drugs would increase the amount of centrally available dopamine. However, resting tremor is a feature of Parkinson disease and not an adverse effect of L-DOPA therapy.

202
Q

PD drugs

Possible SE of dopamine agonists

A

Dopamine agonists (DAs) such as ropinirole may lead to impulse control disorders (ICDs), irrespective of the underlying disease. Patients taking DAs should be aware of this adverse effect and regularly monitored, because ICDs may have debilitating effects on their work and social functioning. If the patient shows signs of ICDs, the dose should be slowly tapered or discontinued until symptoms resolve. Abrupt discontinuation of DAs can lead to dopamine agonist withdrawal syndrome. Although most patients with ICDs due to DA therapy respond well to discontinuation of the medication, ICDs may persist in some patients.

Other adverse effects of DAs include restlessness, nausea, orthostatic hypotension, drowsiness, hallucinations, and psychosis.

203
Q

PD: Wearing off effect and post dose dyskinesia: how to manage

diagnosed six years ago on the basis the clinical findings of a classical resting tremor, rigidity, and bradykinesia. He has been taking a levodopa/carbidopa combination twice a day. Over the last two months he has noticed that his tremor has worsened and only improves once he takes his next dose of the medication. At the same time he has noticed that for about 30 minutes after taking this dose he has increased and uncontrolled movements of his arms.

Change to rotigotine patch

Increase dose of levodopa/carbidopa

Smaller and more frequent dosing of levodopa/carbidopa

Add entacapone

Cease carbidopa; give levodopa only

A

This patient is now experiencing two symptoms.

  1. Firstly, wearing-off effect prior to the next dose, demonstrating inadequate dopamine and recommencement of parkinsonian symptoms.
  2. Secondly, post-dose dyskinesia, reflecting excess dopamine post-dose.

The most effective way of dealing with these two symptoms is to provide smaller (to prevent dyskinesia) more frequent (to prevent wearing-off effect) dosing of levodopa.

Increasing the dose of levodopa/cabidopa may reduce his wearing-off effect, as there will be more dopamine available, but is also likely to result in increased post-dose dyskinesia.

Dopamine agonists:
Rotigotine could be used as adjunctive therapy in the initial management of Parkinson disease, but this will not have any impact on the wearing off effect.

Catechol-O-methyltransferase (COMT) inhibitor
Entacapone, may help to prevent breakdown of levodopa in the periphery. COMT inhibitors work to prevent breakdown of levodopa in the periphery, thus increasing the amount available to the brain. This may help to improve wearing off by prolonging the duration of action of levodopa, but may the post dose dyskinesia

Ceasing carbidopa would only worsen the wearing-off effect, result in an increased levodopa requirement, and therefore more side-effects. Carbidopa is a decarboxylase inhibitor which works by preventing levodopa to dopamine production before it reaches the brain, thus enabling a smaller dose of levodopa to be given and thus fewer side effects.

204
Q

T2DM Rx with limitations in Rx options

GLP-1 agonists ( “Aglutide”)
SGLT-2 DOC in (“flozin”)
DOC with which co-existing Cardiovascular concern ()?

A
  • GLP-1 agonists and SGLT2 inhibitors have the best evidence base for cardiovascular benefits in patients with atherosclerotic cardiovascular disease
    • (GLP-1, esp vascular problems)
    • (SGLT-2, esp heart failure).
  • Considering renal function is important when selecting oral diabetic medications
    • SGLT2 inhibitors require decent renal function to adequately cause a glucose lowering effect, and so in this patient with ESRF, this would not be an appropriate choice.
  • DPP-4 inhibitors and GLP-1 agonists are contraindicated if the patient has a history of pancreatitis
  • SGLT-2 inhibitors are cardio and renal protective- desired in diabetics with risk factors for CVD and renal impairment by having hypertension, diabetes and high cholesterol

SGLT-2 “iglaflozin”

  • Dapagliflozin
  • Empagliflozin
  • Canagliflozin

S/E of keto acidosis- must be stopped prior to surgery

From stages 3–4 of chronic kidney disease, the side effects of the drugs outweigh the benefits.

GLP1 antaganists: incretins have “aglutide” anomoly exenetide

  • semaglutide
  • Exenatide
  • Liraglutide-
    • A rapid-release formula that is administered daily, liraglutide is also approved for the treatment of severe obesity with secondary complications
  • Albiglutide-
    • An extended-release formula that is administered once a week
  • Dulaglutid

GLP-1 antaganist MOA

205
Q

T2DM unresponsive to Rx with limitations in Rx options

DMT2 review after a recent admission to hospital with an episode of acute pancreatitis. This was the first time he had experienced this problem and has made some significant lifestyle changes since then, including reducing his alcohol intake and quitting smoking. He remains asymptomatic since his discharge.

His most recent laboratory investigations since his discharge are all within normal range, apart from his HbA1c, which is 9%. He was diagnosed with type 2 diabetes five years ago and has been managing with lifestyle changes and metformin extended release 1g twice daily since then.

He takes perindopril for high blood pressure and atorvastatin for high cholesterol.

On examination he has a blood pressure of 130/93 mmHg, pulse 77/min and temperature 36.5C. Cardiovascular and abdominal examination are unremarkable.

Which one of the following is the most appropriate next step in management?
Choose the single best answer.

Repeat the HbA1c again in 3 months
Start a long acting insulin at night
Start empagloflozin (SGLT2 inhibitor)
Start sitagliptin (DPP-4 inhibitor)
Start exenatide injections (GLP-1 agonist)

A

Repeat the HbA1c again in 3 months ie metformin as is

The maximum dose of metformin -2000mg daily ( 1g TDS) a first line treatment for type 2 diabetes.

If target HbA1c is not achieved another agent needs to be added and then reviewed in 3 months. If the target is still not reached in 3 months it is time to move further down the algorithm. This is a recent shift towards aggressive treatment of diabetes to reach a target HbA1c rather than delaying starting other agents and using a ‘wait and see approach’.

Second line options available include starting a SGLT2 inhibitor, DPP-4 inhibitor, GLP-1 agonist, sulfonylurea, insulin. The decision on which agent to use should be based on personal patient factors as well as co-morbid conditions and contraindications.

In this case the major factor is his recent acute pancreatitis. This is a contraindication to starting a DPP-4 inhibitor AND GLP-1 agonist, hence both of these option cannot be used in this patient.

  • Start sitagliptin (DPP-4 inhibitor)
  • Start exenatide injections (GLP-1 agonist)

long acting insulin at night: Although insulin can be used second line it is a risky medication and it would be appropriate in this case to try alternative oral medications before starting insulin injections. This often suits patients better, as they may not like the idea of life-long injections and also reduces the risk of hypoglycaemia and complications of insulin such as local reactions, infection and risk of incorrect dosing in the community by patients.

An SGLT-2 inhibitor such as empagloflozin would also offer benefits to this patient in terms of cardiovascular and renal protection. He already has risk factors for CVD and renal impairment by having hypertension, diabetes and high cholesterol, hence if he can tolerate this medication it remains the best option in this case.

206
Q

pharmacotherapy for acute delirium.

0-year-old man with a background of Alzheimer’s disease was admitted for an accidental overdose of diazepam four days ago. A day prior to his planned discharge, he presents with aggressive behaviour and loud noises at 12 am in the medical ward. He has been pacing along the corridor and refuses to return to bed. When instructed by the nurse, he started throwing chairs and pushing security staff members. A low-stimulus environment is set up. Verbal de-escalation and re-orientation is attempted but proves unsuccessful to control his agitation. His clinical workup was unremarkable apart from a positive urine drug screen for benzodiazepine on admission.

Which one of the following is the most appropriate next step in controlling his agitation?

Risperidone
Lorazepam
Flumazenil
Complete Inpatient Treatment Order form
Urgent psychiatry review

A
  • Risperidone is the preferred pharmacotherapy for acute delirium. (not haliperidole)
  • Benzodiazapam preferred for alcoholic delirium, not acute delirium.

Delirium on the ward is a common clinical presentation faced by junior doctors. In managing delirium, it is important to investigate the underlying cause with physical examination and appropriate laboratory/radiological tests and manage the patient using non-pharmacological measures such as verbal de-escalation and creating a low-stimulus environment as well as treating any physical illness found.

However, if non-pharmacological interventions are unsuccessful and the patient continues to be a threat to harming others, short-term medications can be administered. This is to calm the patient down and prevent harm to himself and others. This gives clinicians the opportunity to investigate the underlying cause more effectively and safely.

Risperidone is the mainstay pharmacological intervention for acute delirium. It is used as a last resort and may alleviate agitation. It, however, does not treat the underlying cause of delirium. Haloperidol may be preferred by some, but its side effect profile and efficacy may be marginally not as good as second generation antipsychotics such as risperidone (though no antipsychotics change the duration or severity of delirium as they do not treat the cause and all are associated with increased mortality if used long term).

Benzodiazepine is less favourable as it often worsens cognition. However, it is the preferred drug in delirium due to alcohol withdrawal.

A psychiatric review may be required in this man’s management plan, but is not the next step. Often, psychiatric reviews may not available at night.

An in-patient treatment order may be required if the patient chooses to self-discharge at this stage, but this is not the most immediate management plan.

Learning Points
Risperidone is the preferred pharmacotherapy for acute delirium.

207
Q

Will naloxone work in an alcohol or benzodiazepine (Valium®, Klonopin®, Xanax®) overdose?

When is flumanazil used

A

No. Naloxone rapidly reverses sedation and respiratory depression due to previously administered narcotics ie reverses the effects of opioids. Examples of opioids include hydrocodone, oxycodone, fentanyl, morphine, and heroin.

Flumazenil (Romazicon®) is the antidote for benzodiazepines, such as diazepam (Valium®), lorazepam (Ativan®), and midazolam (Versed®), and must be administered intravenously.

flumazenil overturns the effects of midazolam on the central nervous system, normally post operatively

When are benzos used in siezures

diazepam 10 mg IV or rectal (may be repeated as above)- good if IV access issues

lorazepam 4 mg IV push over 2 minutes (may be repeated in 5-10 min if seizure activity continues)

midazolam 10 mg IV (may also be given IM if no IV access)

208
Q

Potent BZDs with shorter half-lives, such as alprazolam and lorazepam, are linked to higher risk of dependence. Abrupt cessation of chronic BZD use is also associated with a withdrawal syndrome, characterized by CNS hyperexcitability, and symptoms such as anxiety, restlessness, agitation, hallucinations, delirium, and seizures.

A

hould We Use Flumazenil in BZD Overdose?
Flumazenil is a competitive antagonist of the BZD receptor and reverses BZD-induced sedation. However, use of flumazenil is controversial, because it can precipitate withdrawal seizures in patients who take chronic BZDs, and then since the receptor is blocked, BZDs would be ineffective in terminating the seizures. A secondary issue is that flumazenil does not consistently reverse respiratory depression caused by BZDs [8]. The consensus seems to be that flumazenil should only be used when the risk of BZD overdose outweighs the risk of flumazenil use, such as to avoid endotracheal intubation in a BZD-naive patient who was inadvertently oversedated during procedural sedation.

Use in Alcohol Withdrawal
BZDs are commonly used to treat the psychomotor agitation experienced by patients withdrawing from alcohol, and to prevent development of serious complications, such as delirium, hallucinations, and seizures. While all BZDs could theoretically work, long-acting BZDs with active metabolites such as diazepam and chlordiazepoxide are preferred due to more consistent drug levels and decreased chance of symptom recurrence [5]. On the other hand, for patients with advanced liver disease, which is true of many alcoholics, a BZD with a moderate duration of action may be preferred, like lorazepam, because it may help avoid oversedation in a patient with impaired hepatic clearance [5].

BZDs are classified as low, medium, or high potency. Most of the BZDs we use in the ED are either medium (diazepam) or high potency (clonazepam, lorazepam).

Diazepam may be useful when a longer duration of action is desirable [active metabolites]

Lorazepam requires a longer time to reach peak effect than either midazolam or diazepam and may lead to stacking doses

BZDs are first-line agents for termination of seizures in status epilepticus.

If IV access is available, lorazepam 4 mg IV (0.1 mg/kg IV, in children, max 4 mg) should be administered, and repeated in 5 minutes as needed. Onset of action is within 2 minutes, and duration is 4-6 hours.
Diazepam (0.15 mg/kg IV in adults, 0.2 mg/kg IV in children) is an alternative [3]. Efficacy of lorazepam and diazepam at terminating seizures is similar, though studies show a trend toward improved efficacy in lorazepam

209
Q

siezures in kids

A

Key considerations in acute assessment

  • Duration of seizure including pre-hospital period
  • Past history: previous seizures and anti-seizure medication (management plan if in place), neurological comorbidity (eg VP shunt, structural brain abnormality) renal failure (hypertensive encephalopathy), endocrinopathy (electrolyte disturbance)
  • Focal features
  • Evidence of underlying cause that may require additional specific emergency management. Underlying causes include:
    • hypoglycaemia
    • electrolyte disturbances
    • meningitis
    • drug/toxin overdose
    • trauma
    • stroke and intracranial haemorrhage
  • Age: treatable cause is more likely in children <6 months

History

Detailed chronological history of events and behaviours before, during and after the seizure

History should be taken from the child if possible and obtain bystander account

Ask about:

  • aura, focal features
  • level of awareness
  • recent trauma, consider non-accidental injury
  • focality of limb or eye movement
  • post-ictal phase/hemiparesis

Relevant past history

  • Family history of seizures or cardiac disorders/sudden death
  • History suggestive of absence seizures or myoclonic jerks, nocturnal events
  • Developmental history

Examination

  • Full neurological examination looking for any abnormal neurological findings, signs of meningitis or raised intracranial pressure
  • Cardiovascular examination including BP and look for any signs that suggest an underlying cause eg neurocutaneous stigmata, microcephaly

Red Flags

  • Head injury with delayed seizure
  • Developmental delay or regression
  • Headache prior to the seizure
  • Bleeding disorder, anticoagulation therapy
  • Drug/alcohol use
  • Focal signs

Differential diagnosis of seizure

  • Arrhythmia
  • Breath holding spell (episode occurs when the child is crying)
  • Vasovagal syncope with anoxic seizure (postural change, preceded by dizziness and nausea)
  • Non-epileptic paroxysmal disorder

Management

Initial support

In most situations, observation for 5 minutes is appropriate whilst waiting for seizure to stop spontaneously

Treat the child the way the parents will at home – keep safe and observe

At this stage there is no need to check oximetry or apply oxygen

Active seizure flowchart

  • Account for benzodiazepine doses given pre-hospital (eg by parents or paramedics)
  • If available, refer to patient specific seizure management plan in children with a known seizure disorder
  • Do not give a medication if the child is allergic, has previously been unresponsive, or if he/she already taking it

Active seizure flowchart

MIDAZOLAM
In adults my first choice would be MIDAZOLAM. If there is no intravenous access, it can be given intramuscularly at a dose of 0.15mg/kg, or a standard adult dose of 10mg IM.

Dosage:
In adults and children: IV/IM/IO 0.15mg/kg
In children:
Buccal: 0.3mg/kg to a maximum of 10mg
Intranasal: 0.2-0.5mg to a maximum of 10mg

210
Q

perinatal depression (PND)

perinatal depression (PND), DOC

A

This stem demonstrates a woman with perinatal depression (PND), which has worsened in the postpartum period. She fits the DSM-5 criteria for perinatal depression as she has a depressed mood, feelings of guilt, appetite changes, sleep changes and loss of interest in hobbies. These have been present prior to birth and she is 4 weeks postpartum so she fits into the time criteria as well (>2 weeks). These symptoms are also clearly impacting on her ability to function, as is outlined by Stacey who has reasonably good insight into her condition. It sounds like she previously had much milder symptoms and was able to be manage without medication. However, currently she would fit more into a moderate – severe category of perinatal depression and therefore requires medication. It is positive to see that her risk assessment revealed her as being low risk as she has no current thoughts of self-harm or suicide.

Cognitive behavioural therapy and interpersonal therapy is first-line for mild PND and should also be used in conjunction with medications for more severe cases. From the stem it sounds like Stacey is engaged in this with a psychologist.

The safest anti-depressant class and the one that is recommended in guidelines to use in pregnant women and breastfeeding is an SSRI (selective serotonin re-uptake inhibitor). This should be started in conjunction with a perinatal psychiatrist, or with approval from an obstetrician. Sertraline is an example of an SSRI and hence would be the most appropriate medication to trial for Stacey.

Fluoxetine is also an SSRI, however it has been found to have the highest concentrations in breast milk when mother’s take it, so it should be avoided in mother’s who are breast feeding.

Duloxetine is an SNRI (selective noradrenaline re-uptake inhibitor). It is not recommended in breast feeding as it has not been adequately studied. Similarly, to mirtazapine, which is a noradrenergic and specific serotonergic antidepressant (NaSSA).

Tricyclic anti-depressants (TCA) are considered safe for breast feeding but are often not used first line unless the person has had a good response previously. Doxepin is a TCA should be avoided as there have been cases of sedation and respiratory depression in infants.

Learning Points
SSRI’s except fluoxetine are considered safe to use in breastfeeding mothers

211
Q

EPSE, Serotonin Syndrome, NMS, MH

Tardive dyskinesia, acute dystonia, NMS, Serotonin syndrome, MH. DOC and how to manage each

A
  • Acute dystonia- Benztropine *** anticholinergic
  • Tardative dyskinesia- Tetrabenzine ***, clozapine/quetiapine as alternatives in schizophrenia
  • NMS- Dantrolene/Bromocriptine
  • Serotonin syndrome- cryoheptadine (5-HTa receptor antagonist- also in carcinoid syndrome)
  • MH- Volatile anaeathetics and DMR: Rapid Cooling and dantolene

Tardative Dyskinesia - A condition affecting the nervous system, often caused by long-term use of neuroleptic drugs. Tardive dyskinesia causes repetitive, involuntary movements, such as grimacing and eye blinking.

Benztropine is used in acute dystonic reactions, it blocks striatal cholinergic receptors, thus helping to balance the cholinergic and dopaminergic activity. Drugs responsible for dystonic reactions other than antipsychotics include: Antiemetics — e.g. metaclopramide, proclorperazine.

Tetrabenazine inhibits presynaptic dopamine release and blocks postsynaptic dopamine receptors. It has been used for decades to treat a variety of hyperkinetic movevment disorders, including tardive dyskinesia

MH: Volatile anesthetic gases, such as halothane, sevoflurane, desflurane, isoflurane, enflurane or the depolarizing muscle relaxants suxamethonium and decamethonium
Some examples of drugs that don’t cause MH include local anesthetics (lidocaine, bupivacaine, mepivacaine), opiates (morphine, fentanyl), ketamine, barbiturates, nitrous oxide, propofol, etomidate, and benzodiazepines. The nondepolarizing muscle relaxants pancuronium, cisatracurium, atracurium, mivacurium, vecuronium and rocuronium also do not cause MH

212
Q

TCAs not to be fooled by

A

Amoxapine , Doxepin

213
Q

Infectious mononucleosis

18-year-old man presents to his General Practitioner with a one-week history of fatigue, malaise, and sore throat. He denies recent travel, intravenous drug use, or exposure to sick contacts. On examination, his pulse rate is 110/min, blood pressure 120/70mmHg, respiratory rate 19/min, and temperature is 38.5C. His chest is clear on auscultation. Exudates are present on both tonsils. Cervical lymphadenopathy is present bilaterally. There is fullness and mild tenderness in the left upper quadrant of his abdomen.

Which one of the following is the most likely diagnosis?
Choose the single best answer.

Group A streptococcus
Cytomegalovirus
Epstein-Barr virus
Adenovirus
Influenza

A

Infectious mononucleosis can be diagnosed with a combination of clinical features, blood examination, Heterophile test (Monospot test), or EBV serology.

This patient presents with a one-week history of fever, pharyngitis, and lymphadenopathy which is the classic triad of infectious mononucleosis (IM). While there are numerous causes (i.e., Group A Strep and CMV) of IM, the clinical syndrome is most commonly (~75%) associated with an Epstein-Barr virus (HHV-4) infection. Furthermore, splenomegaly favors the diagnosis of EBV-associated IM, as GAS is not a well-documented cause of splenomegaly.

Patients with suspected IM based upon the clinical picture (history and exam) should have a white blood cell count with differential and a heterophile test (Monospot test) or EBV serology (IgM & IgG) serology ordered. IgM & IgG antibodies for EBV become detectable around the time of symptom onset due to the prolonged incubation period (i.e., 4-8 weeks). IgM-EBV antibodies are a marker of acute infection and disappear after approximately three months, the IgG antibodies remain for life and are a marker of prior EBV infection. IgM and IgG antibodies for the EBV capsid antigen have a sensitivity and specificity of 97% and 94%, respectively.

Much debate exists around the use of the heterophile antibody test (Monospot test) and EBV serology as both can be used to diagnose acute EBV infections. Both tests have high sensitivity and specificity and are still used, however, the Monospot test is available by point-of-care testing in some centers. Some providers prefer to use the EBV serology as it provides information about acute and past EBV infections, and if the only IgG is elevated it virtually excludes the diagnosis of acute EBV infection.

Special attention should be given to Group A streptococcus (i.e., strep pyogenes) in the diagnosis of EBV, particularly in at-risk populations like Aboriginal people and Torres Strait Islander people to help prevent rheumatic fever. EBV and GAS pharyngitis present as similar syndromes and can be difficult to distinguish clinically.
One tool is the CENTOR criteria (McIsaac criteria) which calculates the probability of GAS infection

  1. cough absent,
  2. tonsillar exudate,
  3. anterior cervical lymphadenopathy,
  4. fever,
  5. and age 3-14 years.

Diagnosis of GAS pharyngitis is confirmed via throat culture for group A streptococcus is positive and may benefit from a course of antibiotic treatment. With this in mind, antibiotic therapy for GAS pharyngitis may be indicated presumptively in situations where there is a high epidemiological risk (e.g. among Aboriginal and Torres Strait Islander populations) or where the diagnosis is confirmed with a throat culture.

Other answers:

Cytomegalovirus: Typical clinical picture of EBV with lymphadenopathy and splenomegaly. The Monospot test is negative for heterophile antibodies and CMV serology confirms the diagnosis.
Adenovirus: Typical presentation includes cough, coryza, conjunctivitis, and diagnosis is confirmed via nasopharyngeal swab.
Influenza: Typically acute onset of symptoms with myalgia, arthralgia, high fever, and cough with a history of sick contacts. Monospot, EBV/CMV serology is negative.

214
Q

18-year-old man presents to his General Practitioner with a one-week history of fatigue, malaise, and sore throat

On examination, his pulse rate is 110/min, blood pressure 120/70mmHg, respiratory rate 19/min, and temperature is 38.5C. His chest is clear on auscultation. Exudates are present on both tonsils. Cervical lymphadenopathy is present bilaterally.

If left UQ tenderness present- immediate infectious mono with EBV most likely / possibly r/o CMV with negative monospot test and EBV-IgM positive

A
215
Q

EBVclinical features, diagnostics and treatment

A
216
Q

photograph shows a white plaque-like covering of most of the tongue

A

typical appearance of oral candidiasis. Whilst all the other conditions listed can be associated with changes to the appearance to the tongue, none have the ‘coated-on’ appearance of the discharge associated with thrush (candida infection).

Iron deficiency anaemia - smooth, glossy appearance

Diabetes mellitus - geographic tongue, fissured tongue

Gastro-oesophageal reflux disease - black, hairy tongue

Scleroderma - atrophic, smooth, constricted, xerostomia

necessity of mouth rinsing immediately after using the inhaler. This helps to remove medication that is remaining in the mouth. The use of spacers not only improve the delivery of medication to the lungs but also reduces the risk of candidiasis.

To treat oropharyngeal candidiasis, miconazole 2% gel 2.5 mL can be used topically (then swallowed), 4 times daily, after food, for 7 to 14 days and continued for at least 7 days after symptoms resolve. Other treatment methods include amphotericin B lozenge and nystatin liquid both for 7-14 days and continued for up to 3 days after symptoms resolution.

Learning Points
Oral candidiasis (thrush) is associated with long term oral/inhaled steroids and antibiotics use.
217
Q

Acid Base

47-year-old female presented with severe bilateral flank pain and developed tachycardia, hypotension, tachypnoea and a temperature up to 39.5 degrees Celsius. On CT scan of the kidneys, ureters, and bladder, bilateral ureteric stones with upstream hydronephrosis are apparent. She received 4L normal saline, and was intubated for surgical management of her stones. She is sent to ICU for stabilisation. The following arterial blood gas is obtained 4 hours later:

pH 7.26 (7.35-7.45)

pO2 82 mmHg (>100)

pCO2 39 mmHg (35-45)

HCO3- 16 (22-26)

Na+ 147 (135-145)

K+ 3.9 (3.5-5)

Cl- 119 (100-109)

A

For primary acid base disturbances:

In a Metabolic situation. All Matches ( pH, PaCO2, and (HCO3))

In a _R_espiratory state, everything is in _R_everse (pH and PaCO2)

  • *pH 7.26 (7.35-7.45), low**
  • PCO2 normal*
  • *HCO3- 16**

Expect metabolic acidosis with resp response ( resp alkalosis, ie rise PH and drop PaCO2)

This patient is suffering severe urosepsis and is acutely unwell. Her pH demonstrates acidosis and a low bicarbonate suggests metabolic acidosis. The anticipated compensation for this derangement is a respiratory alkalosis (i.e. low pCO2) - however this patient’s pCO2 is inappropriately normal.

Its been 4 hours.. expect response
In metabolic acidosis, respiratory compensation can be expected to occur within 30 - 120 minutes. The absence of an appropriate secondary response in a sufficient timeframe indicates an additional co-existing acid-base disorder, often referred to as a ‘mixed picture’.

Check appropriate rate

The expected pCO2 in metabolic acidosis (within a ±2 range) can be estimated using Winter’s formula: Anticipated pCO2 = 1.5 x [bicarb] + 8 (±2)

In this case, the expected pCO2 for her metabolic acidosis would be: 1.5 x 16 + 8 = 32 (±2); versus the patient’s actual pCO2 which is 39. Given sufficient time has elapsed for compensation to occur, the lack of this secondary response indicates that a co-existing disorder.

CO2 too high so must be a respiratory acidosis. The co-existing respiratory acidosis is contributing to her acid-base status and she has a mixed metabolic and respiratory acidosis.

If the patient’s blood gas had been taken within 2 hours of onset of her acidosis, the picture could be characterised as metabolic acidosis with no compensation.

218
Q

Acid base

Name the values looked at for metabolic and respiratory acid base problems

A

Acid base disorders

Normal values:
pH = 7.35-7.45
PCO2 = 35-45
HCO2 = 22-26
Anion gap < 16

Metabolic acid base problem exists if

  • pH is abnormal and pH and PCO2 change in same direction (both up or down)
  • Respiratory compensation is intact if PCO2 resembles last 2 digits of pH (rough gauge for appropriate response vs mixed)

Respiratory acid base problem exists if:

  • PCO2 is abnormal
  • pH and PCO2 change in opposite directions

Mixed acid base problem exists if:

  • PCO2 is abnormal and pH has not changed as expected or normal -Primary Respiratory acid base problem
  • pH is abnormal and PCO2 has not changed as expected or is normal- Primary Metabolic acid base problem
  • Primary Disorder*Expected ChangesMetabolic AcidosisPCO2 = 1.5 x HCO3 + (8 +/- 2)Metabolic AlkalosisPCO2 = 0.7 x HCO3 + (21 +/- 2)Acute Respiratory AcidosisChange in pH = 0.008 x (PCO2 - 40)Chronic Respiratory AcidosisChange in pH = 0.003 x (PCO2 - 40)Acute Respiratory AlkalosisChange in pH = 0.008 X (40 - PCO2)Chronic Respiratory AlkalosisChange in pH = 0.017 x (40 - PCO2)
219
Q
A
220
Q

Acid base

54 yo man is brought to the emergency department by his wife because of high fever and confusion for the past 10 hours. His wife reports that 1 week ago during a trip to Guatemala he underwent an emergency appendectomy. His temperature is 40.1°C (104.2°F), pulse is 132/min, and blood pressure is 74/46 mm Hg. He is oriented only to person. Physical examination shows a surgical wound in the right lower quadrant with purulent discharge. The skin is warm and dry. Serum studies show a sodium concentration of 138 mEq/L, potassium concentration of 3.7 mEq/L, and lactate concentration of 3.5 mEq/L (N = 0.5–2.2). Arterial blood gas analysis on room air shows:

pH7.21PCO236 mm HgHCO3-14 mEq/LO2 saturation87%

Which of the following is the most likely explanation for these laboratory changes?

A
  • pH 7.21- acidosis , PCO2 36 mm Hg-→ (35-45 Normal) , look at HCO3. Metabolica acidosis
  • HCO3- 14 mEq/L-metabolic acidosis, calculated PCO2- 1.5*14 + 8= 29 (27-31)

PCO2 36 mm Hg-→ (too high, so separate resp acidosis)

–causes of resp acidosis: retention of CO2 (retentive or hypoventilate)

O2 saturation 87%-→ hypovetilate

This patient has clinical features of septic shock (e.g., fever, tachycardia, hypotension, warm and dry skin) due to infection of his appendectomy surgical site. His metabolic acidosis is likely due to the accumulation of lactic acid from septic shock. Assess for appropriate compensation using the Winter formula: expected pCO2 = (1.5 x HCO3-) + 8 (± 2).

Respiratory fatigue

Per the Winter formula, a pCO2 of 27–31 mm Hg would reflect an appropriate respiratory compensation for this patient’s metabolic acidosis. Instead, this patient’s pCO2 is 36 mm Hg, which indicates the presence of a respiratory acidosis in addition to the metabolic acidosis. Respiratory acidosis is caused by hypoventilation (e.g., from respiratory fatigue in critically ill patients), which leads to CO2 retention.

Hyperventilation

Hyperventilation results in respiratory alkalosis, which manifests with a lower-than-expected pCO2. This patient’s pCO2 is higher than predicted by the Winter formula.

Salicylate toxicity

Salicylate toxicity typically presents with hyperthermia, altered mental status, and metabolic acidosis. However, salicylates also cause respiratory alkalosis (due to increased respiratory rate resulting from activation of the respiratory center of the medulla), which manifests with a lower-than-expected pCO2 value. This patient’s pCO2, however, is higher than predicted by the Winter formula.

221
Q

Acid base- Resp Acidosis ddx

A 32-year-old man is brought to the emergency department by ambulance 30 minutes after being found unconscious on the street. On arrival, he is unresponsive to painful stimuli. His respirations are 5/min and shallow. Physical examination shows pinpoint pupils. Which of the following sets of findings would be expected on arterial blood analysis?

pH PCO2 (mm Hg) HCO3- (mEq/L)
A pH7.29 pC02 71 HCO3 33
B pH7.52 pC02 51 HCO3 40
C pH7.21 pC02 64 HCO3 25
D pH7.53 pC02 31 HCO3 25
E pH7.26 pC02 36 HCO3 16

A

A pH7.29 pC02 71HCO3 33– 9 mEq increase (full compensation)
B pH7.52 pC02 51 HCO3 40– not resp acidosis (
C pH7.21 pC02 64HCO3 25 – 1mEq increase (more appropriate in acute setting)
D pH7.53 pC02 31 HCO3 25– rep alkalosis
E pH7.26 pC02 36 HCO3 16– mixed metabolic acidosis resp acidosis

key here is identifying reduced resp effort, causing hypoventilation and C02 retention. PCO2 35-45 normal , so values above 45 causing a resp acidosis

This patient’s presentation of decreased respiratory effort, constricted pupils, and unresponsiveness is consistent with opioid overdose.

This ABG analysis suggests acute respiratory acidosis, which is seen in an opioid overdose as a result of hypoventilation. Decreased pH indicates acidosis, increased pCO2 suggests a respiratory source and, due to the acute nature of an opioid overdose, only minimal metabolic compensation from the rapid action of buffers in blood would be expected.

In acute respiratory acidosis, for every 10 mm Hg increase in pCO2 above 40 mm Hg, the HCO3- concentration increases by 1 mEq/L from 24 mEq/L with an expected variability of +/- 3 mEq/L. A slow-onset but more substantial compensation occurs as a result of renal acid-base handling in chronic respiratory acid-base disorders.

pC02 64, so from 40 (50)+1, (60)+1, therefore appropriate acute HCO3 is 24 + 1 +1 = 26 (value of 25)

can use this for resp alkalosis ( appropraite HCO3) calculation too

Differential diagnoses for acute respiratory acidosis in the emergency department setting include

  1. airway obstruction (e.g., foreign body aspiration, acute exacerbations of obstructive lung diseases),
  2. neuromuscular etiologies (e.g., Guillain-Barré syndrome, myasthenic crisis, botulism), and
  3. overdoses with other drugs that can cause CNS depression (e.g., benzodiazepines).
222
Q

NIPT?

A

he NIPT involves a simple blood test and can be done from 10 weeks into the pregnancy.

What does it test for?

The NIPT is a safe and highly effective way of screening for conditions that include:

Some laboratories also test the gender of the baby and look for problems with the sex chromosomes.

The test detects many chromosomal abnormalities, but not as many as a diagnostic test such as amniocentesis. It cannot screen for genetic disorders such as cystic fibrosis, thalassaemia or sickle cell anaemia.

223
Q

delirium

Mx of delirium with failure of deescalation intervention , requiring Pharmacological intervention

A
  • Oral risperidone is the preferred agent for patients experiencing hyperactive delirium.

Pharms Mx in delirium: Options

  1. Haliperiodol ( EPSE)
  2. Risperidone 2 mg (older adults 0.25–0.5 mg) PO; repeat as needed every 2 hours (max. dose: 6 mg/day; older adults 3 mg/day
  3. Olanzapine 5–10 mg (older adults 2.5–5 mg) PO; repeat as needed every 2 hours (max. dose: 20 mg/day) OR Olanzapine 10 mg (older adults 2.5–5 mg) IM; repeat as needed every 20 minutes (max. dose: 30 mg/day; older adults 20 mg/day).

NB NB

Provided she can be persuaded to take it,
oral risperidone 0.5mg
would be a valid option in this scenario as antipsychotics are indicated in the care of patients who have delirium with a severe behavioural or emotional disturbance insufficiently managed by non-pharmacological measures. These should be given in a low dose. Therefore, both olanzapine and risperidone would be appropriate choices. However, involuntary intramuscular administration
for those refusing oral medication
requires a legal treatment order

in most jurisdictions, because of the involuntary nature of sedation and hence chemical restraint.

Benzodiazepines should not be used; these carry heightened risk of complications in the elderly, and may even paradoxically worsen delirium. Morphine is not indicated as there is nothing to suggest that she is in pain; additionally, opioids increase the risk of delirium in surgical patients.